You are on page 1of 189

Recall march 6.

The most common site of


endometriosis is
2015
A. Ovary
By: dr. Mohamed yahia B. Uterosacral ligament
abuelgasem C. Vagina
D. Rectum
E. Oviduct

1. The probability of sarcomatous 7. The posterior fontanelle usually


change occurring in fibroid is closes by what age?

a. 0.1% A. 3 months
b. 1% B. 6 months
c. 5% C. 9 months
d. 10% D. 12 months
e. 15% E. 18 months

2. Midazolam is an effective anxiolytic 8. Frequency of current used in


drug, used for preoperative sedation. electrosurgery unit is
Which receptor is responsible for its
action? A. 1KHz to 1.5MHz
B. 100KHz to 1.5MHz
A. 5-HT 1a C. 100KHZ to 4MHz
B. 5-HT 3 D. 400KHz to 9MHz
C. D2 E. 800KHz to 9MHz
D. GABA
E. N-methyl-D-aspartate (NMDA) 9. The most likely karyotype for a
patient with complete hydatidiform
3. Which one of the following tocolytic mole is
drug is a competitive oxytocin receptor
antagonist ? A. 46, XX
B. 46, XY
A. Ritodrine C. 69, XXX
B. Terbutaline D. 69, XXY
C. Atosiban E. 69, XYY
D. Nifedipine
E. Indomethacin 10. Which one of the following
congenital infections is most
4. What is the mechanism of action of characteristically nassociated with fetal
labetalol ? hydrops?

A. Antimuscranic A. Toxoplasma gondii


B. α blockers B. Parvovirus B 19
C. β blockers C. Rubella
D. Both α and β blockers D. Treponema pallidum
E. Direct-acting smooth-muscle E. Cytomegalovirus
relaxant
11. What is the most common site for
5. What is the contrast dye used in MRI tubal ectopic gestation?
scans?
A. Ampulla
A. Iodine B. Isthmus
B. Barium C. Infundibulum
C. Thallium D. Interstitial portion
D. Gadolinium E. Fimbrial end
E. Tecnisian

1 ‫صفحة‬ Recall march 2015


12. The standard for conventional 17. A 16-year-old girl is diagnosed with
cytogenetic analysis of human transverse vaginal septum. The most
chromosomes is likely embryological mechanism
responsible for this anomaly is
A. FISH
B. G banding A. Absent paramesonepheric ducts
C. M banding B. Absent fusion of
D. DNA microarray paramesonepheric ducts
E. QF-PCR C. Incomplete fusion of
paramesonephric ducts
13. Metabolic abnormality seen in D. Incomplete lateral fusion/septum
congenital hypertrophic pyloric stenosis resorption
is E. Incomplete fusion of sinovaginal
bulb with mullerian system
A. Hypochloremic hypokalemic
metabolic alkalosis 18. A structure found within the adult
B. Hyperchloremic hypokalemic female pelvis formed from the
metabolic alkalosis gubernaculums is the
C. Hypochloremic hypokalemic
metabolic acidosis A. Transverse cervical ligament
D. Hyperchloremic hyperkalemic B. Suspensory ligament of the ovary
metabolic acidosis C. Round ligament of the uterus
E. Hyperchloremic hypokalemic D. Broad ligament
metabolic acidosis E. Uterosacral ligament

14. Which antigen-presenting cells 19. What part of the embryo is the
(APCs) are mainly found in the cervix? origin of primordial germ cells?

A. Plasma cells A. Gonadal ridge


B. Hofbauer cells B. Yolk sac endothelium
C. Langerhans cells C. Coelomic epithelum
D. B cells D. Extraembryonic mesoderm
E. T cells E. Primitive sex cord

15. Mifepristone is a drug used in 20. Misoprostol is a commonly used


medical termination of pregnancy. drug in the medical treatment of
What is the mifepristone mode of miscarriage. What type of drug is
action? misoprostol?

A. Estrogen agonist A. Cyclooxygenase inhibitor


B. Estrogen antagonist B. Prostaglandin f 2α
C. Oxytocin antagonist C. Progesterone antagonist
D. Progesterone agonist D. Prostaglandin E 2
E. Progesterone antagonist E. Prostaglandin E 1

16. In the fetal circulation, which 21. Which artery supplies the distal
structure bypasses the liver, carrying portion of round ligament of the uterus?
oxygenated blood directly into the
inferior vena cava? A. Ovarian artery
B. Uterine artery
A. Ductus venosus C. Vaginal artery
B. Ductus arteriosus D. Superior vesical artery
C. Foramen ovale E. Inferior epigastric artery
D. Right hepatic vein
E. Portal vein 22. While performing laparoscopy, the
lateral umbilical folds can be seen on

2 ‫صفحة‬ Recall march 2015


the anterior abdominal wall in the 28. What type of compound is Histones?
adult. This structure contains :
A. Carbohydrate
A. Inferior epigastric vessels B. Fatty acid
B. Obturator vessels C. Prostaglandin
C. Remnant of urachus D. Protein
D. Obliterated umbilical arteries E. Steroid
E. Ovarian vessels
29. Ovarian reserve is a term that is
23. What is the most abundant used to determine the capacity of the
carbohydrate in the breast milk? ovary to provide egg cells that are
capable of fertilization. Determination
A. Maltose of ovarian reserve is important in the
B. Galactose treatment of infertility. Test for ovarian
C. Glucose reserve
D. Lactose
E. Sucrose A. LH
B. LH/FSH ratio
24. Which of the following structure, C. FSH
when dilated, produces caput medusa? D. Estradiol
E. Anti-mullerian hormone
A. Hepatic veins
B. Splenic veins 30. 28-year-old Asian woman G2P2 at
C. Ductus venosus 26 weeks gestation. Her BMI is 36 kg/m
D. Umbilical arteries 2 Glucose tolerance test report (75 g
E. Umbilical veins load) are as follows:
25. Whenever the intestinal midgut loop  Fasting serum glucose : 4.7
fails to return from the umbilical cord into mmol/l
the abdominal cavity, the defect is known  2-hour serum glucose : 7.4
as which of the following? mmol/l
A. Gastroschisis What is the diagnosis?
B. Exompholos
C. Meckel’s diverticulum A. Normal
D. Vitelline fistula B. Impaired glucose tolerance
E. Left-sided colon C. Impaired fasting glucose
D. Postprandial hyperglycemia
26. Suxamethonium does not readily E. Gestational D.M
cross the placenta because of
31. Peak maternal serum hCG
A. Placental cholinesterase concentrations occur at
B. High protein binding
C. Elongated molecular configuration A. 6 weeks gestation
D. High degree of ionization B. 8 weeks gestation
E. Insufficient maternal C. 10 weeks gestation
concentration D. 14 weeks gestation
E. 20 weeks gestation
27. Nifedipine belongs to which class of
drugs? 32. Forest plot: horizontal lines
represent
A. K + channel blockers
B. Na + channel blockers A. Range
C. Ca ++ channel blockers B. Interquartile range
D. Angiotensin receptor blockers C. 95% CI
E. Angiotensin-converting enzyme D. Standard error
inhibitors E. Study's weight

3 ‫صفحة‬ Recall march 2015


33. Which one of the following bacteria and scant neutrophils. What is the most
produces an exotoxin causing 'toxic likely causative organism?
shock syndrome' associated with
tampon usage? A. Actinomyces israelii
B. Neisseria gonorrhoeae
A. Escherichia coli C. Chlamydia trachomatis
B. Streptococcus agalactiae D. Gardnerella vaginalis
C. Clostridium welchii E. Treponema pallidium
D. Staphylococcus aureus
E. Mycoplasma hominis 38. A 33-year-old woman with an
intrauterine contraceptive device
34. Which of the following most develops symptoms of acute salpingitis.
accurately describes the mode of action On laparoscopy, sulfur granules appear
of letrozole? at the fimbria of the tubes. Which of the
following is the most likely organism?
A. Progesterone-receptor antagonist
B. Estrogen-receptor antagonist A. C. trachomatis
C. Aromatase inhibitor B. Nocardia species
D. Selective estrogen receptor C. N. gonorrhea
modulator D. T. pallidium
E. Selective serotonin reuptake E. Actinomyces species
inhibitor
39. A 32-year-old woman has increasing
35. What is represented by the equation white vaginal discharge. She is 7 weeks
Sensitivity/(1-specificity)? pregnant. Her Chlamydia swab is
positive. All other tests are normal.
A. Accuracy Which is the single most appropriate
B. Positive predictive value treatment?
C. Negative predictive value
D. Positive likelihood ratio A. Amoxicillin
E. Negative likelihood ratio B. Clindamycin
C. Doxycycline
36. 16-year-old has recently become D. Erythromycin
sexually active. She complains of E. Metronidazole
intensely irritating greenish frothy
vaginal discharge. The organism is seen 40. What is the smallest human
under microscope in a drop of saline chromosome?
and PH is 6. What is the most likely
causative organism? A. Chromosome 20
B. Chromosome 21
A. Candida albicans C. Chromosome 22
B. Chlamydia trachomatis D. Chromosome X
C. Gardnerella vaginalis E. Chromosome Y
D. Trichomonas vaginalis
E. Treponema pallidium 41. Which antiemetic drug can cause
uncontrolled muscle movements?
37. A sexually active 18-year-old woman
presents with a fever for the past 24 A. Meclizine
hours and lower abdominal pain and B. Scopolamine
anorexia for the past 5 days. On C. Ondansetron
physical examination, there is D. Granisteron
generalized tenderness of the abdomen E. Metoclopramide
and the cervix appears slightly friable
with motion tenderness. A smear of 42. What is the pathophysiology explain
yellow odorless cervical discharge shows that vitamin A can cause birth defects of
gram negative intracellular diplococci the child during pregnancy?

4 ‫صفحة‬ Recall march 2015


A. Accumulation of free radicals 47. The typical effective dose for
B. Oxidative stress abdominal CT scan is
C. Excessive carotenoids
D. Hypervitaminosis A A. 0.1-0.5 mSv
E. Deficiency of vitamin A B. 1-2 mSv
C. 3-5 mSv
43. Patient has a heavy menstrual D. 10-15 mSv
bleeding. She desires an endometrial E. 20-25 mSv
ablation to stop bleeding after failure of
medical treatment. Which part of the 48. In which circumstance is rhesus
uterus is destructed to stop bleeding? immunisation NOT required in a
rhesus- negative mother?
A. Basal layer of endometrium
B. Functional layer of endometrium A. Amniocentesis
C. Both layer of endometrium B. Chorionic villus sampling
D. Entire myometrium C. External cephalic version
E. Spiral artery D. Sweeping membranes
E. Caesarean section
44. A 26-year-old pregnant woman is
found to have an Hb of 11 g/dL on a 49. Which class of immunoglobulin is
routine blood test , with an MCV of 70. mainly activate the classical
Serum electrophoresis reveals an Hb F complement system?
of 0.5 percent, Hb A 1 of 60 percent, Hb
A 2 of 2 percent and Hb S of 40 percent. A. IgM
B. IgA
Her ferritin levels are normal. The most
likely diagnosis is: C. IgG
D. IgE
A. Iron deficiency anemia E. IgD
B. α - thalassemia
C. β-thalassemia 50. Sample size 30, variance 25 and
D. Sickle cell trait standard error of mean 3 What is the
E. Sickle cell disease data set's Standard deviation?

45. A 36-year-old woman underwent a A. 2


B. 5
total abdominal hysterectomy for
uterine fibroids. Postoperatively she C. 8
complains of loss of flexion of her left D. 12
E. 16
hip and numbness over her left anterior
and medial thigh. The compression of 51. Physiological variation of the
which nerve is likely to be responsible? menstrual cycle between 21-35 days due
A. Common peroneal nerve to
B. Femoral nerve A. Aberrant LH surge
C. Ilioinguinal nerve B. Estrogen fails to produce negative
D. Obturator nerve feedback on pituitary
E. Pudendal nerve C. Estrogen fails to produce positive
feedback on pituitary
46. Which nerve or nerve plexus
D. Luteal phase varies in duration by
provided sensory innervation to the
cervix? time taken for corpus luteum to
degenerate
A. Pudendal nerve E. Loss of progesterone secretion
B. Sacral splanchnic nerve
C. Pelvic splanchnic nerve 52. 23-year-old woman presents with
D. Superior hypogastric plexus fishy foul smelling vaginal odour. 'Clue
E. Inferior hypogastric plexus cells' are found in the smear. Which of

5 ‫صفحة‬ Recall march 2015


the following cell types is belong to C. Quinacrine
'Clue cells'? D. Quetiapine
E. Terguride
A. Neutrophils
B. Natural killer cells 57. Which of the following is commonly
C. Lymphocytes used to determine hormone receptor
D. Macrophages expression 'receptor status' of the breast
E. Epithelial cells cancer?

53. A 27-year-old woman after normal A. Mammogram


vaginal birth develops shortness of birth B. Fine needle aspiration cytology
and seizures. Despite resuscitation she C. Pestron emission computed
dies within 20 minutes. At autopsy tomography
squamous cells and fetal hair are found D. Immunohistochemical staining
in lungs. What is the most likely cause E. fluorescence in situ hybridization
of death?
58. The greater omentum is derived
A. Amniotic fluid embolism from which of the following embryonic
B. Placental abruption structures?
C. Eclampsia
D. Pulmonary embolism A. Dorsal mesoduodenum
E. Postpartum hemorrhage B. Dorsal mesogastrium
C. Pericardioperitoneal canal
54. A 19-year old G2 P1 woman has an D. Pleuropericardial membranes
amniocentesis performed at 18 weeks E. Ventral mesentery
gestation. Fibroblasts recovered from
amniocentesis are grown in culture to 59. A 19-year-old woman gives birth to
assess the karyotype of fetal cells. These her first child. She begins breast feeding
cells are subcultured for additional the infant. She continues breast feeding
experimental work, but the culture is for almost a year with no difficulties
lost after 50 doublings of the cells has and no complications. Which of the
occurred, and the fibroblasts no longer following cellular processes that
grow. Which of the following factors occurred in the breast during
affecting these cells is most likely pregnancy allowed her to nurse the
demonstrated by this phenomenon? infant for this period of time?

A. Nutrition A. Stromal hypertrophy


B. Mutation B. Epithelial dysplasia
C. Apoptosis C. Steatocyte atrophy
D. Aging D. Ductal dilation
E. Oxidation E. Lobular hyperplasia

55. What percentage triiodothyronin (T 60. Which of the following structures


3 ) is free? separates the superficial perineal pouch
from the deep perineal pouch?
A. 0.1%
B. 1% A. Superior levator ani
C. 5% B. Inferior levator ani
D. 15% C. Superior fascia of urogenital
E. 70% diaphragm
D. Perineal membrane
56. Which is non-ergot dopamine E. Colles fascia
agonist drug used to treat
hyperprolactinemia? 61. Which of the following hormones
increases the excretion of calcium in
A. Cabergoline Kidney?
B. Quinagolide

6 ‫صفحة‬ Recall march 2015


A. Insulin 67. Where in the kidney is the majority
B. Cortisol of bicarbonate reabsorbed?
C. Calcitonin
D. parathyroid hormone A. Bowman's capsule
E. Antidiuretic hormone B. Collecting duct
C. Distal convoluted tubule
62. Choriocarcinoma is a malignant D. loop of Henle
form of gestational trophoblastic E. proximal convoluted tubule
disease. When Choriocarcinoma
metastasises, it has a propensity for 68. What drug can be given to reverse
which type of spread? myelosuppression in methotrexate
toxicity?
A. Direct invasion
B. Hematogenous A. Folic acid
C. Lymphatic B. Vitamin B 12
D. Surface implantation C. Pyridoxine
E. Transcoelomic D. Thymidine
E. Folonic acid
63. The epithelial lining of the cervix
exterior to transformation zone is which 69. Vertical transmission of HIV
of the following tissue types? infection mostly occurs

A. Transitional A. In the first trimester


B. Stratified squamous keratinized B. In the second trimester
C. Stratified squamous non- C. In the third trimester
keratinized D. During labour and delivery
D. Stratified columnar E. In the neonatal period through
E. Simple columnar breastfeeding

64. A patient presents with condyloma 70. Which cytokines are secreted by
lata. At what stage of syphilis does this virally infected host cells and Stimulates
feature occur? uninfected neighbouring cells to
synthesize antiviral proteins?
A. Neurosyphilis
B. Primary A. Interleukin 1 (Il-1)
C. Quaternary B. Interleukin 10 (Il-10)
D. Secondary C. Tumor necrosis factor alpha (TNF
E. Tertiary α)
D. Lymphotoxin (LT)
65. What histology constitutes the E. Interferons alpha and beta (IFN α
majority of vulvar cancers? and IFN β)

A. Squamous cell carcinoma 71. Insensible loss of water from skin


B. Basal cell carcinoma and lungs in 70 year old healthy woman
C. Merkel cell carcinoma is
D. Adenocarcinoma
E. Melanoma A. 100 ml/day
B. 450 ml/day
66. What sexually transmitted disease C. 850 ml/day
(STD) is caused by Haemophilus D. 1350 ml/day
ducreyi? E. 2600 ml/day

A. Chancre 72. The vaccine for measles, mumps and


B. Chancroid rubella (MMR) is best characterized as
C. Granuloma inguinale
D. Condyloma acuminatum A. Synthetic peptide vaccine
E. Molluscum contagiosum B. Killed virus vaccine

7 ‫صفحة‬ Recall march 2015


C. Inactivated virus vaccine B. Mitochondria
D. Live virus vaccine C. Nucleus
E. Recombinant viral vaccine D. Rough endoplasmic reticulum
E. Smooth endoplasmic reticulum
73. Which of the following is a
circulatory blood cell that is capable of 78. What is the most common type of
differentiation into plasma cells? episiotomy performed in the United
kingdom (UK)?
A. Neutrophils
B. Basophils A. Mediolateral episiotomy
C. Monocytes B. Median episiotomy
D. B lymphocytes C. Paramedian episiotomy
E. T lymphocytes D. Lateral episiotomy
E. J-shaped episiotomy
74. A 40-year-old woman had bilateral
silicone breast implants placed two 79. What is the embryonic origin of the
years ago. Since that time, she has noted trigone of urinary bladder?
increased firmness with slight deformity
of the breast on the left. The implants A. Mesonephric duct
are removed, and there is evidence for B. Paramesonephric duct
leakage of the implant contents on the C. Ureteric bud
left. Which of the following cell types is D. Urogenital sinus
most likely to be most characteristic of E. Caudal of cloaca
the inflammatory response in this
situation? 80. What is the most common congenital
solid tumour of the newborn?
A. Neutrophils
B. Mast cells A. Astrocytoma
C. Plasma cells B. Willim's tumour
D. Giant cells C. Neuroblastoma
E. T lymphocytes D. Sacrococcygeal teratoma
E. Hepatic hemangioma
75. Which class of immunoglobulin is
primary secreted in breast milk and 81. Which ovarian tumor is associated
protects the infant's intestinal mucosa with Meig's syndrome?
from infection? A. Cystadenoma
A. IgA B. Teratoma
B. IgD C. Fibroma
C. IgE D. Mucinous
D. IgG E. Granulosa cell tumour
E. IgM 82. Which screening test has high
76. Which immunoglobulin class has a negative predictive value for detection
pentameric structure and is unable to of preterm pre-labour rupture of the
cross the placenta? membranes?

A. IgA A. Nitrazine test


B. IgD B. Salivary estriol test
C. IgE C. Fetal fibronectin test
D. IgG D. Spinnbarkeit test
E. IgM E. Cervical length measurement

77. Where does fatty acid synthesis 83. Oxybutynin hydrochloride is a non-
occur within the cell? selective muscarinic receptor antagonist
used to treat bladder overactivity.
A. Cytosole

8 ‫صفحة‬ Recall march 2015


Which of the following diseases are C. Gap phase 0 (G0)
aggravated by use of oxybutynin? D. Gap phase 2 (G2)
E. Mitotic phase (M)
A. Bronchial asthma
B. Closed angel glucoma 89. Which one of the following stages of
C. Diabetes mellitus the cell cycle is preceded by mitosis?
D. Parkinsonism
E. Peptic ulcer A. Synthetic phase (S)
B. Gap phase 1 (G1)
84. How many days does it take from C. Gap phase 0 (G0)
the resting follicle stage to ovulation? D. Gap phase 2 (G2)
E. Gap phase 3 (G3)
A. 14 days
B. 28 days 90. A neoplasm of the female genital
C. 85 days tract occurring in an 18-year-old girl
D. 120 days whose mother was treated with
E. 375 days diethylstilbestrol during the pregnancy
is likely to be a:
85. LH peak precedes ovulation by
A. Brenner cell tumor of ovary
A. 16 hr B. Teratoma of ovary
B. 24 hr C. Sarcoma botryoides of vagina
C. 36 hr D. Clear cell carcinoma of vagina
D. 48 hr E. Squamous cell carcinoma of vulva
E. 72 hr
91. A 19-year-old extremely anxious
86. Which one of the following best parturient requests labor epidural
describes the histological type of analgesia. During epidural placement,
endometrium soon after ovulation? cerebrospinal fluid obtained in the
needle catheter. Which along spinal
A. Decidual reaction cord dose the needle traverse obtaining
B. Atrophic endometrium
CSF leakage from the needle catheter?
C. Proliferative endometrium
D. Secretory endometrium A. Vertebral canal
E. Arias-Stella phenomenon B. Space between wall of vertebral
canal and dura mater
C. Space between arachnoid and dura
87. A 49-year-old female asks about the mater
benefits and risks of hormone D. Space between arachnoid and pia
replacement therapy (HRT) to control mater
the effects of menopause. The use of E. Spinal canal
hormone replacement therapy (HRT) 92. When should the prophylactic dose
will decreases the risk of which cancer? of low molecular weight heparin
A. Breast cancer (LMWH) be stopped before regional
B. Endometrial carcinoma analgesia?
C. Hepatoma A. 6 hr
D. Ovarian cancer B. 12 hr
E. Colorectal cancer C. 24 hr
88. Which one of the following stages of D. 48 hr
the cell cycle is cell prepared for DNA E. 72 hr
synthesis? 93. Monopolar diathermy cause
A. Synthetic phase (S) coagulation effect with sparkling to stop
B. Gap phase 1 (G1) the bleeding vessels by Which of the
following procedures?

9 ‫صفحة‬ Recall march 2015


A. Electrosurgical cutting 99. An inherited metabolic disorder
B. Electrosurgical desiccation called phenylketonuria (PKU) can
C. Electrosection result in serious problems in infancy.
D. Fulguration Via which pattern of inheritance is the
E. Vaporization PKU?

94. Which nerve arises from ventral A. Autosomal dominant


rami of lumbar plexus and emerges B. Autosomal dominant with
from the medial border of psoas major incomplete penetrance
muscle? C. Autosomal recessive
D. X-linked recessive
A. Femoral nerve E. Polygenic inheritance
B. Genitofemoral nerve
C. lateral femoral cutaneous nerve 100. Which of the following is the most
D. Common peroneal nerve active form of vitamin D?
E. Obturator nerve
A. Ergosterol
95. Which of the following eicosanoids B. 7-dehydrocholesterol
are mainly increased before C. 25 hydroxycholecalciferol
menstruation? D. 24,25 dihydroxycholecalciferol
E. 1 alpha 25 hydroxy derivative
A. leukotriene LTB4 calcitriol
B. Thromboxane TXA2
C. Prostacyclin PGI2 101. 'Donovan bodies' are associated
D. Prostaglandin D2 with?
E. Prostaglandin F2α
A. Granuloma inguinale
96. Which enzyme in the renal tubular B. Lymphogranuloma venereum
cells are responsible for production of C. Herpes genitalis
ammonia? D. Congenital syphilis
E. Chancroid
A. Glutaminase
B. Urease 102. What is the main mechanism of
C. Arginase action of the copper-IUD?
D. Glutamate dehydrogenase
E. Carbonic anhydrase A. Blockage of the endometrial
cavity
97. What type of joint is formed at the B. Blockage of the uterine tubes
sacroiliac joint? C. Inhibition of fertilization
D. Local inflammatory reaction
A. Primary cartilaginous preventing implantation
B. Secondary cartilaginous E. Hostility of the cervical mucus
C. Gomphosis
D. Condyloid 103. In the developing foetal testis, the
E. Synovial cells that produce mullerian-inhibiting
substance are the
98. Which substance removed from
proinsulin to release insulin into the A. Testicular mesenchymal cells
circulation? B. Interstitial cells of leydig
C. Sertoli cells
A. Preproinsulin D. Spermatogonia
B. Substance P E. Primordial germ cells
C. A peptide
D. B peptide 104. What kind of cells secrete
E. C peptide Calcitonin hormone?

A. Chief cells

10 ‫صفحة‬ Recall march 2015


B. Parafollicular cells D. Endoderm of midgut
C. Oxyphilic cells E. Urorectal membrane
D. G cells
E. Follicular cells 110. Sonographically, fluid appear

105. Which of the following clotting A. Echogenic


factors are remain unchanged in normal B. Hypoechoic
pregnancy? C. Anechoic
D. Isoechoic
A. Factor VII E. Hyperechoic
B. Factors XI and XIII
C. Factors II, V and IX 111. What is the mode of inheritance of
D. Factors VII, VIII, X and XI Duchenne muscular dystrophy (DMD)?
E. Factors VII, VIII, X, XI and XIII
A. X-linked dominant
106. Peptides cross the placenta by: B. Autosomal recessive
C. Autosomal dominant
A. Simple diffusion D. Mitochondrial inheritance
B. Facilitated diffusion E. X-linked recessive
C. Solvent drag
D. Active transport 112. Which of the following arteries
E. Pinocytosis gives rise to the deep dorsal penile
artery?
107. A 34-year-old woman attends
antenatal clinic for a routine ultrasound A. Internal pudendal
scan. Abnormalities of placentation are B. Testicular
detected and a magnetic resonance C. Inferior vesicle
imaging (MRI) scan is organized by the D. Obturator
fetal medicine consultant. The MRI E. Umbilical
report shows: ‘The placenta is lower
anterior in which invades through the 113. What is the most common cause of
myometrium up to the uterine serosa’. secondary hyperparathyroidism?
What is the most likely diagnosis? A. Parathyroid adenoma
A. Placenta accreta B. Multiple myeloma
B. Placenta percreta C. Sarcoidosis
C. Placenta increta D. Chronic renal failure
D. Placenta praevia E. Multiple endocrine neoplasia type
1
E. Vasa previa

108. The progesterone receptor is an 114. Besides the skull, the axial skeleton
example of a : is derived from which structure?

A. Guanylate cyclase receptor A. Neural crest


B. Nuclear transcription factor B. Neural tube
C. Tyrosine kinase nuclear receptor C. paraxial mesoderm
D. Ligand-gated ion channel in the D. Somatic mesoderm
cell membrane E. Splanchnic mesoderm
E. G protein coupled receptor on the 115. Lithium therapy during pregnancy
Golgi complex increases the risk of congenital
anomalies in which organ?
109. The rectum originates from
posterior division of which structure? A. Lung
A. Allantois B. Heart
B. Primitive Cloaca C. Brain
C. Ectoderm of proctodeum D. Limb

11 ‫صفحة‬ Recall march 2015


E. Kidney A. Adenomyosis
B. Endometriosis
116. Which signaling molecule that C. Leiomyoma
stimulate the renin-angiotensin system D. Leiomyosarcoma
to regulate the sodium concentration? E. Endometrial carcinoma
A. Angiotensin 121. Glucose 6 phosphatase deficiency is
B. Aldosterone a common metabolic disease in Africa.
C. Bradykinine What is the function of the enzyme
D. Norepinephren glucose 6 phosphatase in carbohydrate
E. Oxytocin metabolism?
117. Antidiuretic hormone (ADH) is A. Adds glucose to glycogen ends
released from which part of B. Adds phosphate to glucose
hypothalamus: C. Production of ATP in Aerobic and
Anaerobic oxidation
A. Arcuate nucleus D. Production of NADPH in red
B. Preoptic nucleus blood cells
C. Periventricular nucleus E. Converts glucose 6 phosphate to
D. Suprachiasmatic nucleus glucose
E. Supraoptic and Paraventricular
nuclei 122. Which neurotransmitter is secreted
onto the adrenal medulla?
118. The bifurcation of the abdominal
aorta occurs at the level of which of the A. Acetylcholine
following structures? B. Norepinephrine
C. Epinephrine
A. Lowest point of costal margin D. Dopamine
B. Highest point of iliac crest E. Serotonin
C. pubic tubercle
D. Anterior superior iliac spine 123. An 18-year-old woman who had
E. Posterior superior iliac spine inflammation of the right ovary
complains painful spasms in the muscles
119. Combined oral contraceptive pills and some numbness on the skin of the
are safe to use following evacuation of a medial part of her right thigh. The
complete hydatidiform mole at what following nerves is mostly involved?
stage?
A. Femoral nerve
A. A declining of hCG titers B. Genitofemoral nerve
B. A plateau of hCG titers C. Sciatic nerve
C. hCG titers start to decline D. Obturator nerve
D. Return of hCG titer to normal not E. Nerve to obturator internus
rising
E. Combined oral contraceptive pills 124. Plasma concentration of estrogen
cannot be used at all increases throughout pregnancy. By 40
weeks, the concentration of estriol
120. 46-year-old woman who has increased to estradiol by who much
chronic pelvic discomfort undergo CT times?
scan which shows multiple
circumscribed masses in the A. 100 times
myometrium. Hysterectomy is B. 50 times
performed and histopathological C. 20 times
examination shows spindles shaped cells D. 10 times
in whorled bundles and the mitotic E. 5 times
figures are scarce. The most likely
diagnosis is? 125. If day 0 is the day of fertilization,
implantation of the embryo into the

12 ‫صفحة‬ Recall march 2015


endometrium occurs on what day after a transverse suprabic skin incision is
fertilization? made?

A. day 3 A. Superficial epigastric


B. day 5 B. Superficial circumflex iliac
C. day 7 C. Deep circumflex iliac
D. day 14 D. Superior epigastric
E. day 28 E. Inferior epigastric

126. What percentage of Down occur 130. An Asian woman books in for her
due to Robertsonian Translocation? third pregnancy at 12 weeks of
gestation. She has recently moved to the
A. 1% UK from Thailand to be with her new
B. 3% husband. After pre-test counseling, with
C. 5% the aid of an interpreter, she agrees to
D. 10% hepatitis B virus (HBV) screening. The
E. 90% results return as follows. What is the
significance of these results?
127. A pregnant woman desires
screening test for down's syndrome and  HBsAg; POSITIVE
the following report is obtained :  Anti-HBc; POSITIVE
 Anti-HBc IgM; NEGATIVE
 Age related risk; 1:1000  HBeAg; NEGATIVE
 MoM AFP; 1.05  Anti-HBe; POSITIVE
 MoM β-hCG; 0.85  HBV DNA; 203 iu/ml
 Test risk; 1:10000

What will you counsel this woman? A. Acute HBV infection


B. Chronic infection (immune control
A. The fetus has down's syndrome phase)
B. The fetus does not have down's C. Natural HBV immunity (resolved
syndrome infection)
C. It is high risk for down's D. Occult HBV infection
D. It is low risk for down's E. Post vaccination
E. Ask the patient to go home
131. A 24-year-old woman presents to
128. Which of the following describes her GP following sexual contact with
the correct order of musculature of the her boyfriend. She is concerned about
anal canal from deep to superficial? the development of new lesions on her
vulva. Upon exam, you note pearly-
1. Deep part of external sphincter white, non-tender several dome-shaped
ulcers around 3 mm in diameter. Which
2. Subcutaneous part of external of the following is the most likely cause
sphincter of the patient’s vulvar lesions?
3. Internal sphincter A. Chlamydia trachomatis serotype L
1,L3
4. Superficial part of external sphincter
B. Herpes simplex virus (HSV) type
A. 1,4,2,3 2
B. 2,1,4,3 C. Molluscum contagiosum
C. 3,1,4,2 D. Donovanosis
D. 3,2,1,4 E. Syphilis
E. 2,4,1,3
132. Identify which of the following
129. Which vessels can possibly be gives the correct pathway of piriformis
injured in the subcutaneous tissue when muscle. Origin Exits pelvis via Insertion

13 ‫صفحة‬ Recall march 2015


A. Ventral surface of sacrum Greater A. 50,20,000
siactic notch Greater trochanter of B. 70/20,000
femur C. 120/20,050
B. Ventral surface of sacrum Lesser D. 120/20,070
siactic notch Greater trochanter of E. 120/20,120
femur
C. Ventral surface of sacrum Greater 137. Which option correctly identifies
siactic notch Lesser trochanter of the embryological origin, Beginning and
femur ending of the round ligament of the
D. Dorsal surface of sacrum Greater uterus?
siactic notch Greater trochanter of
femur Embryology
E. Dorsal surface of sacrum Lesser Beginning Ending
siactic notch Lesser trochanter of A. Gubernaculum ovarii
femur Anteroinferior to uterine cornua
labium majus
133. Which of the following potential
B. Gubernaculum ovarii
mechanism is more likely to cause tissue
damage from ultrasound exposure? Posterosuperior to uterine cornua
labium majus
A. Cavitation C. Gubernaculum ovarii
B. Heat Anteroinferior to uterine cornua
C. Scatter labium minus
D. Fractionation D. Paramesonephric duct
E. Ionization Posterosuperior to uterine cornua
labium minus
134. A woman has been diagnosed with E. Paramesonephric duct
carrying the BRCA 1 gene. What is her Anteroinferior to uterine cornua
life time risk of ovarian cancer? labium majus

A. 5% 138. Premenopausal estrogen is mainly


B. 10% produced by which cells?
C. 15%
D. 20% A. Ovarian Stromal cells
E. 40% B. Ovarian Granulosa cells
C. Ovarian Theca cells
135. Warfarin blocks postribosomal D. Adrenal secretory cells
glutamic acid carboxylation of the E. Peripheral fat cells
precursors of which clotting factors?
139. A newborn girl is found to have
A. Factor I multiple midline malformations. Her
B. Factor II physician suspects a chromosomal
C. Factor III disorders and orders a karyotype
D. Factor V (picture below).
E. Factor XI

136. Some-town in the UK, has a


population of 350,000. During 2010,
there were 20,000 live births, 70
stillbirths, and 50 early neonatal deaths.
Among the births that year, there were
20 sets of twins and four sets of triplets,
and four mothers died at the time of
delivery. Which of the following Which of the following is the most likely
represents the perinatal mortality rate diagnosis?
in this town during 2009?

14 ‫صفحة‬ Recall march 2015


A. Cri-du-chat syndrome A. Meta analysis data
B. Down syndrome B. Binary data
C. Turner syndrome C. Bivariate data
D. Edward syndrome D. Digital data
E. Patau syndrome E. Ordinal data

140. Consider the following pedigree: 143. The following image shows assisted
conception technique by
intracytoplasmic sperm injection. What
will you see in the middle at 12'oclock?

Assuming that the solid square indicate


that the person is affected with the
condition in the question, What is the A. Mature oocyte
likely mode of inheritance? B. Sperm
A. Autosomal dominant C. First polar body
B. X-linked dominant D. Second polar body
C. Y-linked dominant E. Zygote
D. Autosomal recessive 144. Remifentanil is an ideal controlled
E. X-linked recessive analgesia during labour because
141. The following is a normal A. It is a strong µ receptor antagonist
spirometry of a preoperative woman. B. It has a long duration of action
C. It is rapidly metabolized by tissue
estrases
D. It is more effective than epidural
analgesia
E. It cannot cross the placenta

145. The following diagram shows the


hormonal level in a normal menstrual
cycle. Which hormone is indicated in
purple in the above diagram?
Which lung volume is indicated by the
arrows on the spirometry tracing?

A. Residual volume
B. Expiratory reserve volume
C. Inspiratory reserve volume
D. Tidal volume
E. Vital capacity
A. Estrogen
142. The following scatter plot is used to B. Progesterone
view what type of data? C. Inhibin
D. luteinizing hormone (LH)
E. Follicle stimulating hormone
(FSH)

146. Which structure blocks the blood


supply to a loop of small intestine at the

15 ‫صفحة‬ Recall march 2015


femoral ring causing strangulated B. Bipolar disorder
femoral hernia? C. Schizophrenia
D. Postnatal depression
A. Inguinal ligament E. Puerperal psychosis
B. Lacunar ligament
C. Pectineal ligament 152. What is the correct formula to
D. Pubic tubercle calculate the specificity? TP = true
E. Pubis symphysis positive; FP = false positive; TN = true
negative; FN = false negative
147. What is the most common cause of
caushing's syndrome? A. TN / (TN + FN)
B. TP / (TP + FP)
A. Administration of synthetic ACTH C. TN / (TN + FP)
B. Paraneoplastic syndrome D. TP / (TP + FN )
C. Pituitary adenoma E. TN/(TP+FP+FN)
D. Iatrogenic steroid administration
E. Adrenal adenoma 153. The Eighth Confidential Enquiry
into maternal deaths in the United
148. The most common cause of delay in Kingdom
puberty in males is:
defines maternal mortality as:
A. Constitutional delay in growth
B. Klinefelter's syndrome A. The number of deaths per 1000
C. Noonan's syndrome pregnancies
D. Primary hypothyroidism B. The number of deaths per 100,000
E. Gonadal dysgenesis pregnancies
C. The number of direct and indirect
149. What electrolyte imbalance can deaths per 100,000 mortalities
cause paralytic ileus? D. The number of direct and indirect
deaths per 10,000 mortalities
A. Hypernatremia E. The number of direct and indirect
B. Hyperkalemia deaths per 100,000 pregnancies
C. Hypocalcemia
D. Hyponatermia 154. You wish to investigate the time it
E. Hypokalemia takes to perform a caesarean section on
women with and without pre-eclampsia.
150. Which one of the following imaging At the end of the study you have two
techniques gives maximum radiation groups of women with the duration of
exposure to the patient during checking caesarean sections recorded for each. It
tubal patency? can't be assumed that these are
A. Robin tube cannulation normally distributed. Which non-
B. Hysterosalpingography parametric statistic test would you use?
C. hystero contrast sonography A. Chi-squared test
D. Hysteroscopic tube visualization B. Linear regression analysis
E. Laparoscopy and dye intubation
C. Mann-Whitney U test
151. About 2 weeks postnatally, a 36 D. Pearson’s R test
year old multiparous woman starts to E. Student’s t-test
worry that her partner is spying on her 155. Nitrogenous bases make up cellular
as she cares for her baby. She begins to nucleic acids including DNA and RNA.
think she can hear someone telling she is Which base pairs with Adenine in the
doing tasks incorrectly. Her partner standard DNA helix?
calls the health visitor who suspects that
the most likely diagnosis is: A. Thymine
B. Guanine
A. Baby blues C. Inosine

16 ‫صفحة‬ Recall march 2015


D. Uracil measured 34 x 18 x 12 cm. Which of the
E. Uranine following cellular processes was the
major reason for the increase in the size
156. Which of the following human of the uterus?
papilloma virus (HPV) subtypes is high-
risk for the development of genital A. Endometrial glandular hyperplasia
warts? B. Myometrial fibroblast
proliferation
A. HPV serotype 2, 63 C. Endometrial stromal hypertrophy
B. HPV serotype 5, 8 D. Myometrial smooth muscle
C. HPV serotype 6, 11 hypertrophy
D. HPV serotype 16,18 E. Vascular endothelial hyperplasia
E. HPV serotype 16, 18, 31
162. Which biochemical technique is
157. Lancefield grouping of streptococci used to prenatal identification of cystic
is based on the presence of fibrosis by using specifically amplifying
predetermined DNA sequences?
A. Carbohydrate antigen on cell wall
B. Peptidoglycan antigen on cell wall A. karyotyping
C. M protein B. Sweat test
D. Color of blood agar C. Southern blotting
E. Sugar fermentation D. Polymerase chain reaction (PCR)
E. Fluorescent in situ hybridization
158. The deep circumflex iliac artery is (FISH)
a branch of which artery?
163. What is the function of DNA
A. Internal iliac artery polymerase?
B. Internal pudendal artery
C. External iliac artery A. To unwind DNA helix during
D. Common iliac artery replication
E. Femoral artery B. To add nucleotides to the end of a
template DNA strand
159. Which of the following metabolic C. To synthesis the nucleotides of
processes produces the most energy DNA strands
needed for uterine contraction during D. To seal together the broken ends
parturition? of DNA strands
A. Aerobic glycolysis E. To proofread the growing DNA
B. Anaerobic glycolysis strands and replace mismatched
C. krebs cycle nucleotides
D. Oxidative phosphorylation 164. Two days after undergoing total
E. Beta oxidation abdominal hysterectomy and bilateral
160. Which hormone is secreted by the salpingo- oophorectomy a 60-year-old
placenta and fetal adrenal gland, woman develops high temperature and
promotes maturation of the fetal lungs? confusion. On examination; She has a
temperature of 38.2°C, a respiratory
A. Androstenedione rate of 28/min, a pulse of 103 beats/min,
B. Dehydroepiandrosterone a blood pressure of 130/80 and a urine
C. Corticosterone output of 10 ml/hr after catheterization.
D. Cortisol Which of the following is the most likely
E. Cortisone diagnosis?

161. The non pregnant uterus of a 20- A. Sepsis


year-old female measured 7 x 4 x 3 cm. B. Septic shock
She became pregnant, and just before C. Systemic inflammatory response
delivery of a term infant, the uterus syndrome

17 ‫صفحة‬ Recall march 2015


D. Septicemia A. Nitrofurantoin
E. Severe sepsis B. Gentamicin
C. Trimethoprim
165. Six hours after undergoing a D. Co-amoxiclav
laparoscopic tubal ligation, a 32-year- E. Cefuroxime
old woman complains of suprapubic
pain and oozing. On examination there 169. A 28-year-old primiparous woman
is a tender palpable suprapubic mass up is 34 weeks pregnant. Which of the
to the umbilicus that is dull to following complains of pregnancy, you
percussion. Her Hb level is 11 g/dl. will give her a further investigations?
What is the most likely diagnosis?
A. Backache
A. Bladder injury B. Breast tenderness
B. Intra-abdominal bleeding C. Nausea
C. Ureteric trauma D. Tiredness
D. Urinary retention E. Dysuria
E. Fluid overload
170. A 43-year-old woman presents to
166. Which enzyme found in red blood her GP with a 2-month history of
cells that buffers blood CO 2 ? galactorrhoea. She is otherwise healthy
and is not taking any medications.
A. Cytochrome-b5 reductase Nothing abnormal is found in the
B. 5-nucleotidase examination. Her hormonal profile was
C. Glucose-6-phosphate a follows :
dehydrogenase
D. Carbonic anhydrase  FSH 8.7 iU/l
E. Pyruvate kinase  LH 4.5 iU/l
 Oestradiol 100 pmol/l
167. Which of the following is the  Prolactin 700 mU/l
genotype of Edward's syndrome?  TSH 15 mU/l
 Free T 4 9 iU/l
A. 45, XO
B. 46, XX What is the most likely cause ?
C. 47, XXY
D. 47, XX A. Prolactinoma
E. 46, XX+18 B. Non-functioning pituitary tumour
C. Addison's disease
168. A 30-year-old woman who is 37 D. Hypothyroidism
weeks pregnant presents with dysuria E. Hyperthyroidism
and urinary frequency. She has been
started on cefuroxime that was 171. An 18-year-old woman presented
prescribed for a previous urinary tract with a history of 15 kg weight loss in the
infection. Mid stream urine specimen previous four months. She has been
report result shows; amenorrheic for some months. Her
examination is completely normal. Her
 Leukocytes; 50/hpf urine pregnancy test is negative. Her
 100,000 colony of Pure E. coli laboratory results shows;
growth
 Co-amoxiclav; sensitive  Luteinizing hormone; 0.3 mIU/l
 Nitrofurantoin; sensitive  Follicle-stimulating hormone; 0.2
 Trimethoprim; sensitive iu/l
 Gentamicin; sensitive  Prolactin 400 mU/l
 Cefuroxime; resistant  Thyroid stimulating hormone; 1.5
mIU/L
What is the most suitable antibiotic to  Total T 4 ; 200 nmol/L
use?  Total T 3 ; 2 nmol/l

18 ‫صفحة‬ Recall march 2015


What is the most likely diagnosis? D. Respiratory acidosis
E. Mixed metabolic/respiratory
A. Hypothalamic dysfunction acidosis
B. Pituitary adenoma
C. Ovarian dysgenesis 175. Diagnostic studies frequently use a
D. Polycystic ovary syndrome receiver operating characteristic curve.
E. Hyperthyroidism What do the two axes of the curve
represent?
172. A 70-year-old lady had an
ultrasound because she was feeling
bloated which showed bilateral ovarian
masses with no solid areas associated
with abdominal ascites. She is post
menopausal and otherwise well. She was
sent for a CA 125 which comes back as
50 U/mL. What is her risk of
malignancy index score (RMI)? A. Accuracy versus 1/sensitivity
B. Positive likelihood ratio versus
A. 5 negative likelihood ratio
B. 10 C. Positive predictive value versus
C. 150 negative predictive value
D. 300 D. Sensitivity versus 1 - specificity
E. 450 E. Sensitivity versus positive
predictive value
173. A 28-year-old woman with
complete hydatidiform mole is found to 176. Among hundred women during
have bilateral multiple tiny ovarian laparoscopic diagnosis of endometriosis.
cysts along with an enlarged uterus at Four women were found with
12 weeks’ gestational age. The endometriosis. Three out of the four
quantitative β-hCG level of 300,000 women were found to be true positive.
mIU/mL. What is the most likely type of Seven women were found to be healthy
these ovarian cysts? i.e. without disease. What is the
sensitivity of this test?
A. Polycystic ovaries
B. Granulosa cell cyst A. 10%
C. Follicular cyst B. 30%
D. Theca lutein cyst C. 43%
E. Corpus luteum cyst D. 75%
E. 85%
174. A 35-year-old female presents in
the emergency department (ED) with 177. A 45-year-old woman with type 2
severe tachypnea. The following blood D.M undergoes an abdominal
gases are obtained on room air: hysterectomy for fibroids and excess
vaginal bleeding. Postoperatively, she
 pH 7.25 develops shortness of breath. A chest X
 pCO 2 7.8 kPa ray reveals bilateral hilar vascular
 pO 2 10 kPa marks with mild pleural effusion. Her
 HCO 3 - 18 mEq/L blood investigation shows:
 Saturations 84 per cent
 Sodium 110 mmol/L
What does the blood gas show?  Potassium 4.0 mmol/L
 Bicarbonate 24 mmol/L
A. Metabolic acidosis with
 Urea 3.0 mmol/L
respiratory compensation
 Creatinine 80 µmol/L
B. Metabolic acidosis
C. Respiratory acidosis with What is the most likely diagnosis?
metabolic compensation

19 ‫صفحة‬ Recall march 2015


A. Adrenal insufficiency D. Substrate level phosphorylation
B. Syndrome of inappropriate E. Oxidative phosphorylation
antidiuretic hormone (SIADH)
C. Pulmonary edema 182. The level of the umbilicus varies in
D. Peumonia obese women. What reference point can
E. Renal failure be used instead?

178. Which of the following is the most A. Transpyloric plane


related to endometrial carcinoma? B. Subcostal plane
C. Supracristal plane
A. Simple hyperplasia D. Level of anterior superior iliac
B. Complex atypical hyperplasia spine
C. Complex hyperplasia E. Transtubercular line
D. Simple atypical hyperplasia
E. Polypoid adenoma 183. Which of the following nerves
pierces the internal oblique muscle and
179. A 20-year-old, gravida 2, para 1, at passes through the inguinal canal?
34 weeks of gestation by her last
menstrual period presents to the A. Iliohypogastric
emergency department complaining of B. Ilioinguinal
mild, painless vaginal bleeding. On C. Genital branch of the
examination, her vital signs are: T 37 0 genitofemoral
C, HR 100 bpm, BP 120/85 mmHg, RR D. Obturator
18/min and the fundal height 34 cm with E. Lateral femoral cutaneous
a normal CTG tracing. What is the next
course of action? 184. The chance of malignant
transformation occurring in
A. Perform a sterile vaginal premenopausal benign ovarian cysts is
examination
B. Perform an amniocentesis to rule A. 0.1%
out infection B. 3%
C. Perform an ultrasound C. 10%
examination D. 15%
D. Order FBC and cross matching E. 25%
E. Immediate delivery 185. The following ECG trace shows :
180. 32 year old 3 rd gravida presents at Regular rhythm, normal rate, multiple
32 weeks gestation with a history of P wave before one NN QRS complex
ruptured membranes. Which of the (4:1)
following procedures should be avoided in
this patient?

A. Vital sign assessment


B. Electronic fetal monitoring What is the correct interpretation of
C. Digital examination this ECG?
D. High vaginal swabs
A. Sinus bradycardia
E. Ultrasound examination
B. Atrial flutter
181. Which process occurs in the C. 1 st degree heart block
mitochondria where uses electrons D. 2 nd degree heart block
transport to generate adenosine E. 3 rd degree heart block
triphosphate (ATP)?
186. Which of the following depression
A. Krebs cycle screening questionnaire are frequent
B. Cori's cycle normally used during pregnancy?
C. Glycolysis

20 ‫صفحة‬ Recall march 2015


A. Edinburgh Postnatal Depression C. Estrogenic steroid
Scale D. Progestogenic steroid
B. General Health Questionnaire E. Selective estrogen receptor
C. Hamilton Depression Rating Scale modulator
D. Beck Depression Inventory
E. Hospital Anxiety and Depression 191. Mechanism of action of
Scale Neostigmine is

187. A-24-year old woman complains of A. Choline ester


increasing perineal pain after normal B. Cholinomimetic
vaginal birth. Her labs reveals, Hb of 9 C. Muscarinic antagonist
g/dl and white blood cell count of D. Anticholinesterase
17,000/cmm. At discharge what should E. Nicotinic antagonist
be prescribed for this patient?
192. Without treatment, asymptomatic
A. I.V antibiotic, oral ferrous sulfate bacteriuria at 32 weeks gestation is at
then discharge risk of what pregnancy complication?
B. Oral antibiotic, oral ferrous sulfate
then discharge A. Eclampsia
C. Oral antibiotic, oral ferrous sulfate B. Spontaneous preterm labour
and wait till full blood profile C. Renal failure
D. Oral ferrous sulfate and wait till D. Polyhydramnios
full blood profile E. Post term infant
E. Oral ferrous sulfate then discharge 193. At a significant level of P < 0.05,
the null hypothesis H 0 states that
188. The investigation that should be
carried out in every cases with heavy A. There is a insignificant difference
menstrual bleeding is between soya milk feeding and
A. Pelvic ultrasound scan allergy
B. Full blood count B. There is a difference between soya
C. Vaginal swaps for microscopy and milk feeding and allergy
culture C. There is no difference between
D. Liver function test soya milk feeding and allergy 5%
E. Endometrial biopsy of the time
D. There is a difference between soya
189. A-19-year-old woman who is 28 milk feeding and allergy 95% of
weeks pregnant requests treatment for the time
acne and is prescribed antibiotic by her E. There is no difference between
general practitioner (GP). She goes on soya milk feeding and allergy 95%
to deliver a healthy baby girl at term. of the time
Two years later her daughter is noted to
have unusually gray discoloration of 194. A 36-year-old woman had
teeth. Which treatment for acne did her intermenstrual and heavy menstrual
GP prescribed for acne? bleeding for the past 2 months. She had
a history of removal of cervical polyp 3
A. Chloramphenicol years ago and her cervical smears are
B. Sulphonamide normal. On physical examination, there
C. Quinolone are no remarkable findings. Her Hb
D. Aminoglycosides level is normal. A pelvic ultrasound scan
E. Tetracycline shows endometrial thickness of 19 mm
and a round hyperechoic mass in the
190. Clomiphene citrate is endometrial cavity measuring 10X8mm.
What is the most likely diagnosis?
A. Gonadotropin analogue
B. Androgenic steroid A. Adenomyosis

21 ‫صفحة‬ Recall march 2015


B. Endometriosis following blood results. What do these
C. Endometrial hyperplasia results suggest?
D. Endometrial polyp
E. Submucous fibroid  Urea 2.8 mmol/l
 Creatinine 67 micromol/l
195. An 18-year-old woman has had  Sodium 138 mmol/l
pelvic discomfort for several months.  Potassium 4.2 mmol/l
On pelvic examination, there is a right  Urate 0.37 mmol/l
adnexal mass. Ultrasound picture of the  Albumin 32 g/l
right ovary shows cystic lesion  Alkaline phosphatase 198 iu/l
measuring 3.3x 2.5 cm with echogenic  Alanine transferase 33 iu/l
septea. What is the most likely finding?  Bilirubin 5 mmol/l
A. Acute fatty liver of pregnancy
A. Hemorrhagic cyst B. Cholestasis of pregnancy
B. Benign teratoma C. HELLP syndrome
C. Ovarian carcinoma D. Normal blood results for 36 weeks
D. Serous cystadenoma of pregnancy
E. Follicular cyst E. Cholecystitis
196. What is the major estrogen 200. A-27-year old woman complains of
produced by the placenta during amenorrhea for 6 months. For the past
pregnancy? few months she has been feeling
generally tired. Prior to this she had a
A. Dehydroepiandrosterone regular 30 day cycle. Her urine
B. Estradiol
pregnancy test is negative. Pelvic
C. Estriol examination is normal and routine
D. Estrone bloods are ordered:
E. Ethinylestradiol
 Luteinizing hormone; 33 mIU/l
197. From what parent compound are  Follicle-stimulating hormone; 51 iu/l
most eicosanoids derived?  Prolactin; 300 mU/l
A. Arachidonic acids  Thyroid stimulating hormone; 3.1
B. Oleic acids mIU/L (0.2-5.5)
C. Linoleic acids  Total T 4 ; 100 nmol/L (50-500)
D. Steroids  Total T 3 ; 1.5 nmol/l (0.9-2.8)
E. Cholesterols  Oestradiol; 70 pmol/l (100-500)

198. What percentage of women with What is the most likely diagnosis?
over active bladder have urinary A. Hypothalamic dysfunction
incontinence? B. Gonadotropin-producing pituitary
A. 1:2 adenoma
B. 1:3 C. Premature ovarian failure
C. 1:4 D. Polycystic ovary syndrome
D. 1:5 E. Primary hypothyroidism
E. 1:6

199. A 32-year-old Asian woman


presents at 36 weeks of gestation with
abdominal discomfort, 2+ proteinuria
and a blood pressure of 140/90 mmHg.
She has blood tests in accordance with
the NICE guideline for the management
of hypertension in pregnancy. The
midwife asks you to review the

22 ‫صفحة‬ Recall march 2015


Recall September D. internal iliac artery
E. External iliac artery
2015
6. What is the total blood volume in a
By: dr. Mohamed yahia term newborn?
abuelgasem
A. 120 ml/kg
B. 100 ml/kg
C. 85 ml/kg
1. At what lumbar vertebrae does the D. 70 ml/kg
abdominal aorta bifurcate into the two E. 50 ml/kg
common iliac arteries?
7. Which group of viruses causes
A. L1 molluscum contagiosum?
B. L2
C. L3 A. Adenovirus
D. L4 B. Herpes virus
E. L5 C. Papovavirus
D. Parovirus
2. The level at which the ovarian artery E. Pox virus
arises from the abdominal aorta is
8. What is the most common type of
A. T 10 nosocomial infection in the UK
B. T 12 hospitals?
C. L1
D. L2 A. Pneumonia
E. L3 B. Urinary tract infection
C. Surgical wound infection
3. A 25-year-old diabetic woman has D. Bacteraemia
been morbidly obese for the past five E. Infectious diarrhea
years. In this patient, which one of the
following hormones would decrease the 9. What percent of untreated
appetite as levels increase? individuals move on to the third stage of
syphilis?
A. Leptin
B. Thyroxine A. 5%
C. Ghrelin B. 10 %
D. Adiponectin C. 15 %
E. Insulin D. 25 %
E. 35 %
4. From which organ / system does the
Krukenberg tumor of the ovaries 10. What is the mechanism of action of
originate from? trimethoprim?

A. Lung A. Inhibits cross linkage of the


B. Brain peptidoglycan polymer chains
C. Kidney B. Inhibits dihydrofolate reductase
D. Uterus C. Inhibits dihydropteroate
E. GIT synthetase
D. Inhibits DNA gyrase
5. Which of the following arteries E. Inhibits transpeptidation
supplies the anorectal canal superior to
the pectinate line? 11. What is the SI unit for the activity of
radioactive decay?
A. Celiac trunk artery
B. Superior mesenteric artery A. Gray
C. Inferior mesenteric artery B. Sievert

1 ‫صفحة‬ Recall September 2015


C. Rad A. Bacteroides
D. Roentgen B. Escherichia coli
E. Becquerel C. Proteus mirabilis
D. Pseudomonas aeruginosa
12. An 18 year-old woman is found to E. Staphylococcus saprophyticus
have a largely solid ovarian neoplasm.
Tumor markers were obtained and 16. What is the most common
demonstrated a markedly elevated LDH microorganism in the vaginal mucosa?
value. The hCG and a-Fetoprotein were
negative. Which of the following tumors A. Mycobacterium
is the most likely histology for this B. Candida
neoplasm? C. Doderlin lactobacilli
D. Gardenrella vaginalis
A. Theca cell tumour E. Staph. Epidermidis
B. Dysgerminoma
C. Immature teratoma 17. What day of a regular 25-day
D. Brenner cell tumour menstrual cycle is a woman likely to
E. Endodermal sinus tumour ovulate?

13. An obstetrician performs a A. 4


mediolateral episiotomy to expand the B. 11
birth canal during a child birth. Which C. 14
of the following muscles is typically D. 22
incised during this procedure? E. 25

A. Bulbospongiosus and superficial 18. Which antibody (immunoglobulin)


transverse perineal muscles class is involved in the allergic
B. Bulbospongiosus and deep reactions?
transverse perineal muscles
C. Bulbospongiosus and A. IgA
ischiocavernosu muscles B. IgD
D. Ischiocavernosus and levator ani C. IgE
muscles D. IgG
E. Bulbospongiosus and levator ani E. IgM
muscles 19. Which class of immunoglobulin is
14. A premature infant develops primarily secreted in breast milk and
progressive difficulty breathing over the protects the infant’s intestinal mucosa
from infection?
first few days of life. Deficient
surfactant synthesis by which of the A. IgA
following cell types may have B. IgD
contributed to the baby's respiratory C. IgE
problems? D. IgG
A. Alveolar capillary endothelial E. IgM
cells 20. In which part of the cell does
B. Bronchial mucous cells glycolysis take place?
C. Bronchial respiratory epithelium
D. Type I pneumocytes A. Cytosol
E. Type II pneumocytes B. Mitochondrial matrix
C. Mitochondrial cristae
15. Regarding urinary tract infections, D. Rough endoplasmic reticulum
which micro-organism is most likely to E. Smooth endoplasmic reticulum
be associated with bladder
catheterisation?

2 ‫صفحة‬ Recall September 2015


21. What is the most likely histological C. Exocytosis
subtype of ovarian cancer associated D. Facilitated diffusion
with endometriosis? E. Passive diffusion

A. Clear cell carcinoma 27. Which one of the following stages of


B. Serous adenocarcinoma the cell cycle is a mitosis proceeds?
C. Borderline serous tumour
D. Muscinous cystadenocarcinoma A. Synthetic phase (S)
E. Granulosa cell tumour B. Gap phase 1 (G1)
C. Gap phase 0 (G0)
22. A 35 year old woman who is 26 wks D. Gap phase 2 (G2)
gestation goes for walking in a hill and E. Gap phase 3 (G3)
when she is on the extreme of the hill
she becomes little breathless and has 28. What is the name of structure that
paraesthia in her hands . What is the shunts oxygenated blood from the right
appropriate acid base imbalance? to left atrium in the fetus?

A. Metabolic acidosis A. Ductus arteriosus


B. Metabolic alkalosis B. Ductus venosus
C. Mixed metabolic and respiratory C. Foramen ovale
acidosis D. Ligamentum venosum
D. Respiratory alkalosis E. Pulmonary trunk
E. Respiratory acidosis
29. Infection with which one of the
23. What type of acid–base disturbance following human papillomaviruses
result in a case of Conn's syndrome? (HPVs) is associated with an increased
risk of cervical cancer
A. Metabolic acidosis
B. Metabolic alkalosis A. HPV 6
C. No effect B. HPV 11
D. Respiratory acidosis C. HPV 17
E. Respiratory alkalosis D. HPV 30
E. HPV 33
24. Finasteride exert its action by
inhibition of which enzyme? 30. A 34 yr old woman has just returned
to the UK following missionary work in
A. 21-hydroxlase the northern part of South America.
B. 5-alpha reductase She is delighted to be pregnant and
C. Aromatase presents for booking at 12 wks of
D. Phosphodiesterase gestation. After counseling she agrees to
E. Desmolase have routine blood investigations
including screening for syphilis. The
25. In which subsites does vulvar cancer results are returned as following:
most commonly arise?
 Venereal Disease Research Lab.
A. Mons pubis (VDRL) test : reactive
B. Labia majora  Traponema Pallidum particle
C. Labia minora agglutination (TP-PA) : non reactive
D. Clitoris  Fluorescent Traponemal Ab
E. Posterior fourchette absorption (FTA-Abs) : non reactive
26. Urea is transferred from fetal What is the most likely interpretation
circulation to maternal circulation via for theses result?
which transport mechanism?
A. Consistent with traponemal
A. Active transport infection at sometime
B. Endocytosis B. Diagnostic of latent syphilis

3 ‫صفحة‬ Recall September 2015


C. Indicative of early syphilis 36. Male infertility in a patient with
D. Likely biological false positive cystic fibrosis is likely to be due to
E. Suggests a past treated infection which condition?

31. 23-year-old woman presents with A. Congenital absence of testes


fishy foul smelling vaginal odour. 'Clue B. Congenital absence of vas
cells' are found in the smear. What is deference
the most likely causative organism? C. Hypothalamic failure
D. Oligospermia
A. Trichomonas viginalis E. Testicular failure
B. Candida albicans
C. Gardnerella vaginalis 37. Forms the lateral wall of the ischio-
D. Chlamydia trachomatis rectal fossa
E. Treponema pallidum
A. Skin
32. What kind of cells produces B. Anal canal
parathyroid hormone (PTH)? C. Obturator internus fascia
D. Vagina
A. Chief cells E. Rectum
B. Parathynotic cells
C. Oxyphilic cells 38. A cystic structure is noted lateral to
D. C cells the vagina. This is believed to be a
E. Follicular cells Gartner duct. What is the
embryological origin of this structure?
33. Which one of the following stages of
the cell cycle is cell become quiescent? A. Cloaca
B. Genital tubercle
A. Synthetic phase (S) C. Mesonephric duct
B. Gap phase 1 (G1) D. Paramesonephic duct
C. Gap phase 0 (G0) E. Urogenital septum
D. Gap phase 2 (G2)
E. Mitotic phase (M) 39. What percentage thyroxine hormone
(T 4 ) is free?
34. Which of these chromosomes have
centromere located at one side? A. 0.1%
B. 1%
A. Telocentric chromosome C. 5%
B. Metacentric chromosome D. 15%
C. Dicenteric chromosome E. 70%
D. Submetacentric chromosome
E. Acrocentric chromosome 40. What is the second common cause of
congenital adrenal hyperplasia (CAH)?
35. Twenty-four hours after an
abdominal hysterectomy, a 45-year-old A. 17-hydroxylase deficiency
woman complains of vomiting and B. 21-hydroxylase deficiency
severe gas pain. On examination her C. 11-Beta hydroxylase deficiency
abdomen is distended but nontender. D. 3 Beta- hydroxysteroid
Bowel sounds are absent. What is the dehydrogenase deficiency
most likely electrolyte imbalance? E. 17 Beta-hydroxysteroid
dehydrogenase deficiency
A. Hypercalcaemia
B. Hyperkalaemia 41. A 30-year-old P1 + 0 woman
C. Hypernatraemia presents with painful swelling in right
D. Hypokalaemia posterolateral part of vagina and fever
E. Hyponatraemia for 3 days. Diagnosis of Bartholin’s
abscess is made. Incision and drainage

4 ‫صفحة‬ Recall September 2015


is planned. What is the anatomical A. 240
location of Bartholin’s gland? B. 320
C. 400
A. Deep perineal pouch D. 480
B. Ischiorectal fossa E. 720
C. Pudendal canal
D. Superficial perineal pouch 46. Which is the earliest physical sign of
E. Urogenital diaphragm puberty in females?

42. Sympathetic supply to the urinary A. Growth spurt


bladder is derived from which lumbar B. Thelarche
segments? C. Adrenarche
D. Pubarche
A. L1 and L2 E. Menarche
B. L2 and L3
C. L3 and L4 47. What is the most common cause of
D. L4 and L5 postmenopausal bleeding?
E. S2, S3 and S4
A. Atrophic vaginitis
43. A 68-year-old woman developed left- B. Endometrial cancer
sided chest pain a day after she has had C. Endometrial hyperplasia
total abdominal hysterectomy and D. Endometrial polyp
bilateral-salpingo-oophorectomy. A 12- E. Hormone replacement therapy
lead electrocardiogram is performed
and is suggestive of an inferior ST 48. A woman is admitted with
elevation myocardial infarction. Which threatened preterm labour and is
ECG leads represent the inferior commenced on atosiban. What is the
myocardial infarction? mechanism of action of atosiban?

A. V1, V2 A. Calcium channel blocker


B. V3, V4 B. GnRH analogue
C. aVR C. GnRH antagonist
D. I, aVL D. Oxytocin agonist
E. II, III, aVF E. Oxytocin antagonist

44. A 30-year-old woman with bipolar 49. What is the mechanism of action of
disorder is 12-week pregnant. She has vincristine?
been taken her lithium every night.
A. Inhibits topoisomerase II
Which of the following abnormality
may occur if she continues to take B. Cross links DNA
lithium? C. Inhibits function of microtubules
D. Inhibits DNA polymerase
A. Tetralogy of Fallot's E. Alkylates nucleophilic groups on
B. Dandy walker syndrome DNA bases
C. Ebstein's anomaly
D. Limb reduction deformity 50. A 42-year-old woman has frequency,
E. Renal agenesis urgency, and urges incontinence.
Examination is unremarkable and a
45. A 57-year-old postmenopausal midstream specimen of urine is sterile.
woman is referred to clinic with a six- She is treated for detrusor overactivity
month history of bloating. Ultrasound with Mirabegron. Which is the single
reveals a 7 cm multilocular unilateral mechanism of action for this drug?
ovarian cyst with solid areas. There is
A. Selective serotonin re-uptake
no ascites. Her CA-125 is 80 IU/ml.
What is her risk of malignancy index inhibitor
score (RMI)? B. Muscarinic receptors agonist
C. Selective Beta-1 agonist

5 ‫صفحة‬ Recall September 2015


D. Selective Beta-3 agonist hysterectomy is performed, and
E. Selective Beta-3 antagonist pathologic examination of the removed
uterus reveals heterologous components
51. Which of the following ligaments of a malignant tumor of the endometrial
allows us to stand upright with a glands and metaplastic cartilage. Which
minimum of muscular support? of the following is the most likely
diagnosis?
A. Sacrospinous ligament
B. Sacrotuberous ligament A. Endolymphatic stromal myosis
C. Iliolumbar ligament B. Endometrial stromal sarcoma
D. Iliofemoral ligament C. Endometrial carcinosarcoma
E. Pubofemoral ligament D. Endometrial adenosarcoma
E. Endometrioid carcinoma
52. An MRI examination is NOT
allowed under any circumstances when 56. Which of the following is an
the following is present objective of a Phase 3 Clinical testing?
A. A pacemaker A. Assess tolerance
B. A hip/knee joint replacement B. Identify less common adverse
C. An intracranial aneurysm clip reactions
D. A metallic heart valve C. Demonstrate/confirm efficacy
E. A first-trimester pregnancy D. Estimate dose for subsequent
studies
53. Which one of the following E. Estimate activity
statements best describes a type I
statistical (α error)? 57. Primitive gonads develop from
which cell layer?
A. Rejecting a null hypothesis when
there is a difference A. Endoderm
B. Rejecting a null hypothesis when B. Intermediate mesoderm
there is no difference C. Lateral mesoderm
C. Rejecting the alternative D. Paraxial mesoderm
hypothesis when there is a E. Surface ectoderm
difference
D. Accepting a null hypothesis when 58. What cellular organelle is especially
there is a difference abundant in cells that synthesize
E. Accepting the alternative antibodies?
hypothesis when there is a
difference A. Peroxisomes
B. Free ribosomes
54. Oxytocin cannot be administrated C. Rough endoplasmic reticulum
by oral route because D. Smooth endoplasmic reticulum
E. Lysosomes
A. It is resistant to destruction by
gastric juice 59. Psammoma bodies are a histologic
B. It is destroyed by gastric juice feature of ovarian papillary serous
C. It is eliminated very quickly cystadenoma. ' Psammoma bodies' are
D. It is needed in large doses if taken composed of
orally
E. It causes nausea and vomiting A. Collection of calcium
orally B. iron
C. Fibrin
55. A 65-year-old nulliparous women D. Mucin
presents with vaginal spotting. She has E. Lipid
been postmenopausal for 15 years and
does not take hormones. An ultrasound 60. A 27-year-old woman at 30 weeks of
shows a mass in the uterine fundus. A gestation has gross hematuria.

6 ‫صفحة‬ Recall September 2015


Ultrasound shows a normal pregnancy A. Alpha
and a maternal urinary bladder lesion. B. Gamma
Cystoscopy reveals a 3 cm multifocal C. Beta 1
pedunculated papillary lesion. The D. Beta 2
remainder of the urinary bladder is E. Dopaminergic
normal. The most likely diagnosis of this
bladder lesion is 66. What is the most common cause of
the female urethral caruncle?
A. Transitional cell carcinoma
B. Squamous cell carcinoma A. Trauma
C. Adenocarinoma B. Tumour
D. Small cell carcinoma C. Infection
E. Bladder calculi D. Hypoestrogenism
E. Congenital
61. The shelf-life of stored whole blood
is approximately 67. Gonadotropin Releasing Hormone
(GnRH) is what type of Biochemical?
A. 3 weeks
B. 5 weeks A. Tripeptide
C. 7 weeks B. Octapeptide
D. 9 weeks C. Nanopeptide
E. 12 weeks D. Decapeptide
E. Polypeptide
62. Which type of vaccines is absolutely
contraindicated during pregnancy? 68. What type of compound is thyroid
releasing hormone (TRH)?
A. Killed vaccine
B. Toxoid vaccine A. Carbohydrate
C. Conjugate vaccine B. Protein
D. Live attenuated vaccine C. Glycoprotein
E. Recombinant DNA vaccine D. Polypepetide
E. Steroid
63. In what part of the female
urogenital system describes the 69. The standard chest X-ray is
Vestibule? equivalent to what duration of natural
background radiation
A. Space between labium minus
B. Space between labium majus A. 3 days
C. Space between labia minora and B. 3 weeks
majora C. 3 months
D. Area around hymen D. 18 months
E. Area between hymen and labia E. 5 years
minora

64. How much percentage of the cardiac


output passes through the kidneys at
term? 70. The immediate layer around the
A. 5% oocyte of the tertiary follicle is known as
B. 10% the
C. 25% A. Lamina propria
D. 50% B. Cumulus oophorus
E. 75% C. Zona pellucida
65. Terbutaline has a preference for D. Zona granulosa
E. Theca interna
stimulation of which of the following
receptors?

7 ‫صفحة‬ Recall September 2015


71. Nitrogenous bases make up cellular 74. Following a water birth, a woman
nucleic acids including DNA and RNA. elects not to have oxytocics for the
Which of these nitrogen bases is found management of the third stage of
in RNA but not in DNA? labour. Thirty minutes later, she is
brought to the consultant unit with a
A. Thymine postpartum haemorrhage owing to an
B. Cytosine atonic uterus. If she had received
C. Uracil standard oxytocic management for the
D. Guanine third stage of labour, by what amount
E. Adenine would she have reduced her risk of a
postpartum haemorrhage?
72. The number of hours worked per
week for a sample of ten students is A. 10%
shown B. 20%
C. 30%
below D. 60%
E. 90%
Student Hours 75. Which of the following drugs is used
1 20 to suppress lactation postpartum in
mother who suffers from stillbirth?
2 0
A. Quetiapine
3 18 B. Metoclopramide
C. Loperamide
4 16 D. Leuprolide
E. Cabergoline
5 22
76. What is the term used for the
6 40 anticancer agent methotrexate?
7 8 A. plant alkaloid
B. Antimetabolite
8 6
C. Alkylating agent
9 30 D. Chain cutting agent
E. Intercalating agent
10 40
77. In the kidneys, the largest volume of
What is the mode of the above data? glucose reabsorption occurs in which of
the following?
A. 5
B. 10 A. Glomerular capillaries
C. 20 B. Proximal tubules
D. 30 C. Loop of henle
E. 40 D. Distal tubules
E. Collecting duct
73. Which adverse reaction is common
and usually dose related in patients 78. Where in the kidney is potassium
taking misoprostol? completely reabsorbed?

A. Diarrhea A. Bowman's capsule


B. Nausea B. Distal convoluted tubule and loop
C. Vomiting of Henle
D. Bloating C. Proximal convoluted tubule and
E. Dyspepsia loop of Henle
D. Distal convoluted tubule and
Proximal convoluted tubule

8 ‫صفحة‬ Recall September 2015


E. Collecting duct C. Ceftazidime
D. Clindamycins
79. A 64-year-old woman undergoes a E. linezolid
total abdominal hysterectomy and
bilateral salpingo-oophorectomy for 84. The ureteric bud is derived from
endometrial carcinoma. This procedure which of the following embryonic
will be placed in which of the following structures?
wound classifications?
A. Allantois
A. Clean B. Pronephros
B. Clean/contaminated C. Mesonephros
C. Contaminated D. Mesonephric duct
D. Infected E. Metanephric blastema
E. Dirty
85. Twenty percent of estrogen hormone
80. The process by which mRNA is bound to
made from DNA is
A. Sex hormone-binding globulin
A. Synthesis (SHBG)
B. Translation B. Corticosteroid-binding globulin
C. Transcription (CBG)
D. Reverse transcription C. Thyroid-binding globulin (TBG)
E. Replication D. Albumin
E. Pre-albumin (transthyretin)
81. What is the most important cation in
the extracellular fluid? 86. The non-invasive, cell-free fetal
DNA in maternal blood of prenatal
A. Ca 2+ diagnosis would best be used on
B. Cl - l
C. HCO 3 - A. Fetal RBCs
D. K+ B. Fetal WBCs
E. Na + C. Fetal fibroblasts
D. Amniotic cells
82. To perform an elective lower E. Chorionic cells
segment caesarean section, the
obstetrician makes a transverse 87. A 35-year-old diabetic woman
suprapubic incision. Which of the underwent vaginal surgery.
following abdominal wall layers will not Postoperatively, she has Loss of
be encountered transected during this cutanous sensation over the anterior
incision? and lateral surface of the thigh. The
compression of which nerve is likely to
A. Anterior rectus sheath be responsible?
B. Posterior rectus sheath
C. Rectus abdominis muscle A. Superior gluteal nerve
D. Skin and subcutaneous tissue B. Lateral cutanous nerve
E. Transversalis fascia and parietal C. Sciatic nerve
peritoneum D. Femoral nerve
E. Obturator nerve
83. Laboratory results of a clinical
specimen from a patient with hospital- 88. Magnesium is secreted in which of
acquired pneumonia reveal the presence the following nephron segments?
of methicillin-resistant Staphylococcus
aureus (MRSA). MRSA infection best A. Proximal tubule
respond to which antibiotics? B. Thin descending loop of Henle
C. Thick ascending loop of Henle
A. Piperacillin/tazobactam D. Distal tubule
B. Co-amoxiclav E. Collecting duct

9 ‫صفحة‬ Recall September 2015


89. Which artery supplies the structures 94. What is the mechanism of action of
derived from the midgut of the embryo? hydralazine?

A. Coeliac trunk A. α2 agonist


B. Inferior mesenteric B. Angiotensin-converting enzyme
C. Middle rectal inhibitor
D. Renal C. Antimuscarinic
E. Superior mesenteric D. β2 agonist
E. Direct-acting smooth-muscle
90. The vertebral column is developed relaxant
from which germ layer?
95. Which one of the following class of
A. Paraxial mesoderm drugs causes neonatal kernicterus?
B. Somatic mesoderm
C. Splanchnic mesoderm A. Penicillin
D. Extraembryonic mesoderm B. Aminoglycoside
E. Lateral plate mesoderm C. Sulphonamide
D. Chloramphenicol
91. While performing laparoscopy the E. Macrolide
surgeon identified the medial umbilical
folds on the deep surface of the anterior 96. What is the most common cause of
abdominal wall. The two medial caushing's syndrome?
umbilical folds represent remnants of
which of the following structures? A. Administration of synthetic
adrenocorticotropic (ACTH)
A. Urachus B. Paraneoplastic syndrome
B. Inferior epigastric vessels C. Pituitary adenoma
C. Obliterated umbilical veins D. Iatrogenic steroid administration
D. Obliterated umbilical arteries E. Adrenal carcinoma
E. Round ligaments of the uterus
97. Motor fibers to detrusor muscle of
92. The Haldane effect refers to which the urinary bladder are derived from
of the following options? which of the following nerves?

A. The inhibitory effect of oxygen on A. Pelvic splanchnic


fermentation B. Greater splanchnic
B. The chloride shift that maintains C. T11-L2
electrical neutrality D. Superior hypogastric plexus
C. The dissociation constant for the E. Sacral plexus
bicarbonate buffer system
D. The increased capacity for 98. Which of the following muscles
deoxygenated blood to carry CO 2 leaves the lesser pelvis through the
E. The reduction in affinity of greater sciatic foramen?
hemoglobin for oxygen in active
tissues in response to a drop in pH A. pubococcygeus
caused by increased CO 2 B. Iliococcygeus
production C. Puborectalis
D. Piriformis
93. What percentage of leydig-cell E. Coccygeus
tumours is malignant in females?
99. To reduce postoperative pain after
A. < 1% caesarean section, an obstetrician
B. 5-10% performs ilioinginal-iliohypogastric
C. 25% nerve block (IINB) on his 25-year-old
D. 50% patient. What is the nerve root of
E. 70% ilioinguinal and iliohypogastric nerve?

10 ‫صفحة‬ Recall September 2015


A. T 12 C. Stratified squamous non-
B. T 12 and L 1 keratinized
C. L1 D. Stratified columnar
D. L2 E. Simple columnar
E. L 1 and L2
104. Which of the following ovarian
100. A woman experiences a tumor is most prone to undergo torsion
postpartum hemorrhage after delivery during pregnancy?
and is administrated carboprost to
promote uterine contraction. What type A. Serous cystadenoma
of drug is carboprost? B. Mucinous cystadenoma
C. Dermoid cyst
A. Oxytocin agonist D. Theca lutein cyst
B. Progestogen E. Corpus luteium cyst
C. Synthetic prostaglandin E 1
D. Synthetic prostaglandin E 2 105. Red degeneration of uterine fibroid
E. Synthetic prostaglandin F 2 α during pregnancy is due to

101. What is the most appropriate A. Ischemia


treatment for pelvic inflammatory B. Calcification
disease (PID)? C. Infection
D. Gangrene
A. Ofloxacin 400 mg + Doxycycline E. Rupture of capsule
100 mg for 14 days
B. Metronidazole 400 mg + 106. A 14-year-old girl is treated with
Doxycycline 100 mg for 14 days radiotherapy. Which of following tissues
C. Augmentin (co-amoxiclav) 1g + is most radioresistant?
Doxycycline 100 mg for 14 days
D. Ceftriaxone 500 mg + A. Intestinal epithelium
Doxycycline 100 mg + B. Cerebral cortex
metronidazole 400 mg for 14 days C. Respiratory epithelium
E. Augmentin (co-amoxiclav) 1g + D. Bone marrow
metronidazole 400 mg for 14 days E. Skin

102. A 24-year-old woman attends her 107. When does the odds ratio (OR)
obstetrics ultrasound appointment. Her approximate the risk ratio (RR)?
medical history noted mechanical A. When the disease being studied is
prosthetic valves. The scan reveals the occur frequently
presence of depressed nasal bridge, B. When the disease being studied is
stippling of non-calcified epiphysis and rare
microcephaly. No other abnormalities C. When the sample size is large
were noted. A history of taking which D. When there is a negative
drug is likely to explain the aetiology? association between exposure and
A. Thalidomide disease
B. Labetalol E. When there is no association
C. Phenytoin between exposure and disease
D. Sodium valproate 108. A 42-year-old perimenopausal
E. Warfarin woman complains of 12 months history
103. Which type of epithelium lines the of irregular vaginal bleeding. Her
distal urethra near the external urethral husband had vasectomy since 2 years.
orifice? Her family is complete. Which action is
the most appropriate?
A. Transitional
B. Stratified squamous keratinized A. Commence tranexamic acid

11 ‫صفحة‬ Recall September 2015


B. Insertion of a levonorgestrel- E. Prolactin
releasing intrauterine system
(IUS) 114. A healthy 34-year-old woman had a
C. Arrange for transvaginal total abdominal hysterectomy for
ultrasound cervical disease. Routine urea and
D. Perform an endometrial biopsy electrolytes are measured one day after
E. Hysterectomy surgery and are as follows :

109. The bacteria Neisseria gonorrhoeae  Sodium 121 (reference range


can be described as what? 135-145 mmol/l)
 Potassium 4.3 (reference range
A. Gram-negative aerobic cocci 3.5-5.0 mmol/l)
B. Gram-negative aerobic bacilli  Urea 2.8 (reference range
C. Gram-positive anaerobic cocci 2.5-6.7 mmol/l)
D. Gram-positive aerobic bacilli  Creatinine 74 (reference range
E. Gram-negative anaerobic bacilli 70-150 micromol/l)
110. Meckel's diverticulum is an adult What is the most likely cause to explain
remnant of the vitelline duct. Which of the blood test results?
the following embryonic structures are
joined by the vitelline duct? A. Excessive intravenous dextrose
B. Nephrogenic diabetes insipidus
A. Foregut and yolk sac C. Primary aldosteronism
B. Midgut and yolk sac D. Undiagnosed diabetes mellitus
C. Hindgut and yolk sac E. Ureteric damage
D. Foregut and allantois
E. Midgut and allantois 115. Which of the following fungi is
most commonly found in catheter-
111. When should the last therapeutic related infection?
dose of low molecular weight heparin
(LMWH) be stopped before regional A. Cryptococci
anesthesia? B. Candida spp.
C. Aspergillus spp.
A. 6 hrs D. Coccidioides spp.
B. 12 hrs E. Pneumocystis jiroveci
C. 24 hrs
D. 48 hrs 116. A 60-year-old woman with stage
E. 72 hrs IIB cervical cancer is currently
undergoing radiotherapy by impact of
112. Which organelles do type II gamma-emitting isotopes on her vagina.
pneumocytes utilize for surfactant This type of therapy is best described by
release? which of the following terms?
A. Golgi complex A. Adjuvant
B. Lysosomes B. Neoadjuvant
C. Endosomes C. Radical
D. Lamellar Bodies D. Brachytherapy
E. Rough endoplasmic reticulum E. Teletherapy
113. Human placental lactogen (hPL) is 117. A woman who is 14 weeks pregnant
structurally MOST similar to which of presents as she came into contact with a
the following hormones? child who has chickenpox around 4 days
ago. She is unsure if she had the
A. Anti mullerian hormone
condition herself as a child. Blood tests
B. Cortisol
show the following:
C. Follicle-stimulating hormone
D. Growth hormone VZV IgG : Negative

12 ‫صفحة‬ Recall September 2015


VZV IgM : Negative D. Incomplete miscarriage
E. Threatened miscarriage
What is the most likely diagnosis?

A. Non-immunity to measles
B. Non-immunity to chickenpox 121. Following Ectopic pregnancy, what
C. She is immune to chickenpox percentage of medically treated women
D. Recent infection to measles with single dose regimen of
E. Recent infection to chickenpox methotrexate will require surgical
intervention?
118. A 26-year-old second gravid who is
work in a nursery school attends the A. 10%
GP’s surgery as she noticed vesicular B. 15%
rash on her back and abdomen. She is C. 20%
25 weeks pregnant. Her GP confirms D. 30%
the diagnosis of chickenpox. What is the E. 40%
most appropriate action?
122. Two months postoperatively, a 55-
A. Reassurance year-old woman who is being treated
B. Give a single dose of varicella with a diagnosis of small lung cancer
zoster immunoglobulin presents with excessive polyuria and
C. Advise serial ultrasound scans depression. You should recognize that
D. Commence oral acyclovir this patient may be experiencing what
E. Commence intravenous acyclovir electrolyte imbalance?

119. A 25-year-old female presents with A. Hypernetremia


chronic right lower quadrant pain for 3 B. Hyponatremia
months. She is previously received a C. Hyperkalemia
treatment for Chlamydia. Her D. Hypokalemia
ultrasound revealed; a 4-cm diffuse E. Hypercalcemia
homogeneous hypoechoic focal lesion
with low-level internal echoes with little 123. During childbirth, anesthesia is
fluid in the douglas pouch, empty uterus administered into the epidural space of
with ET 4 mm and normal adnexa are the spinal column. Where is the
seen bilaterally. What is the most likely epidural space located?
diagnosis?
A. Between supraspinous and
A. Para-ovarian cyst interspinous ligaments
B. Hydrosalpinx B. Between the wall of vertebral
C. Mucocele appendix cavity and dura mater
D. Cornual ectopic pregnancy C. Between arachnoid and dura mater
E. Endometrioma D. Between arachnoid and pia mater
E. Between pia mater and spinal
120. A 25-year-old woman presented to canal
an early pregnancy unites with mild
vaginal bleeding after 5 week's 124. During a difficult delivery an
amenorrhea. There is history of obstetrician uses forceps to extract the
irregular menses. Ultrasound shows no infant. Upon examining the baby you
evidence of intrauterine pregnancy. notice forceps impressions
Beta hCG is 400 IU/l and beta hCG posteroinferior to the ear. Which of the
after 48hrs later is 700 IU/l. What is the following is very likely to be damaged?
most likely diagnosis?
A. Trigeminal nerve
A. Pregnancy of uncertain viability B. Facial nerve
B. Early normal pregnancy C. Abducens nerve
C. Ectopic pregnancy D. Brachial plexus
E. C 8 T 1 nerve

13 ‫صفحة‬ Recall September 2015


125. What do the bottom and top What is the most likely electrolyte
represent on the box-and-whisker plot? imbalance?

A. Hypercalcaemia
B. Hyperkalaemia
C. Hypernatraemia
D. Hypokalaemia
E. Hyponatraemia
A. Mean and mode
B. Mean and median 129. In a sample of 100 women the age
C. Standard deviation and mean at the delivery of their first child
D. Least and greatest value showed a normal distribution. The
E. Lower and upper quartile mean age at delivery of their first child
was 30 years old, with a standard
126. A 52-year-old woman presents to deviation of 5 years. What is the 95%
her physician for a check-up. She confidence interval?
became menopausal at age 50 and did
not begin hormone replacement therapy A. 20 – 30 years
because of a strong family history of B. 20 – 40 years
breast and endometrial cancer. She now C. 25 – 35 years
fears future menopausal symptoms and D. 29 – 31 years
would like to begin a replacement E. 15 – 45 years
regimen. Which of the following
pharmaceutical agents is most 130. An 18 yr old woman presents to an
appropriate for this patient? early pregnancy unit with vaginal
bleeding after 5 weeks of amenorrhea.
A. Oral conjugated estrogen She had taken a urine pregnancy test
B. Transdermal estrogen that had tested positive. A transvaginal
C. Tamoxifen ultrasound scan showed CRL of 6 mm
D. Raloxifene with no visible fetal heart. What is the
E. Trastuzumab next appropriate step?

127. An undifferentiated malignant A. Check the serum progesterone


tumour on immunohistochemical stain level
shows cytoplasmic positivity of most of B. Measure the serum beta hCG 0 -
the tumor cells for cytokeratin. The 48 hours
most probable diagnosis of the tumor is C. Repeat U/S after 7-10 days
D. Induction with mifepristone
A. Sarcoma E. Intramuscular methotrexate
B. Lymphoma
C. Carcinoma 131. Which pattern of endometrim do
D. Melanoma simple endometrial hyperplasia most
E. Hamartoma likely resemble?

128. A 67-year-old woman attends for a A. Proliferative endometrim


preoperative assessment. She has been B. Secretory endometrim
taking bendroflumethiazide for several C. Endometritis
years for hypertension. Her ECG D. Endometrial polyp
shows: E. Endometrial carcinoma

132. A woman has been diagnosed with


carrying the BRCA1 gene. What is her
life time risk of breast cancer and
ovarian cancer? Breast cancer Ovarian
cancer

A. 30% 10%

14 ‫صفحة‬ Recall September 2015


B. 50% 30% A. Sever gastroenteritis
C. 70% 30% B. Atelectasis
D. 80% 40% C. Pneumonia
E. 95% 60% D. Septic shock
E. Hypovolemic shock
133. The following diagram describes a
structural chromosomal abnormality. 137. What type of epithelial tissue
What is the name of this structural undergoes malignant change in the
abnormality? majority of bladder cancers?

A. Columnar
B. Pseudostratified
C. Squamous
D. Stratified
E. Transitional

138. Which of the following is in direct


contact with maternal blood in lacunae
A. Deletion of the placenta?
B. Insertion
C. Inversion A. Chorion
D. Robertsonian translocation B. Amnion
E. Reciprocal translocation C. Fetal capillaries
D. Fetal endothelium
134. A 36-year-old woman who had E. Maternal epithelium
supraspinous ligament fixation 2 days
ago complains of pain over right mons
pubis, right labia, and the perineum.
Which nerve is most likely to be 139. A 27-year-old woman complains of
injured? amenorrhoea for 12 months with
increased facial hair growth and weight
A. Iliohypogastric nerve gain since she quit smoking. Her urine
B. Ilioinguinal nerve pregnancy test is negative. Her serum
C. Genitofemoral nerve hormonal levels are as follows:
D. Pudendal nerve
E. Posterior femoral cutaneous nerve  FSH 5 lU/L
 LH 15 IU/L
135. What percentage of total body  Prolactin 300 mmol/l
water is intracellular?  Serum oestradiol 350 pmol/L

A. 66% What is the most likely diagnosis?


B. 40%
A. Gonadotropin-producing pituitary
C. 33%
D. 20% adenoma
E. 10% B. Congenital adrenal hyperplasia
C. Polycystic ovarian disease
136. A 30-year-old woman underwent D. Premature ovarian failure
lower segment caesarean section. Two E. Ovarian neoplasm
weeks into the postpartum period, she
has experienced nausea and vomiting 140. In a city having a population of
for three days. On examination, she has 1,000,000 there are 300,000 women of
a temperature of 39 0 C, a respiratory childbearing age. The following
rate of 30/min, and a blood pressure of statistics are reported for the city in the
year 2000:
90/50 mmHg. The patient has warm
peripheries with normal capillary refill Fetal deaths: 200
time. What is the most likely diagnosis?

15 ‫صفحة‬ Recall September 2015


Live births: 25,000 A. Theca externa cells
B. Theca interna cells
Maternal deaths: 75 C. Ovarian Stromal cells
D. Sustentacular cells
Which of the following is the best E. Granulosa cells
estimate of the maternal mortality ratio
in the city in the year 2000? 146. Iodine accumulation in thyroid
cells involves symport with
A. 150
B. 200 A. Chloride
C. 250 B. Potassium
D. 300 C. Hydrogen
E. 350 D. Sodium
E. Calcium
141. Which of the following cell types
lyses cells that have been infected with 147. Which types of cells are phagocytic
viruses? for residual bodies left over from the
process of spermiogenesis?
A. CD4+ T cells
B. B cells A. Testicular mesenchymal cells
C. CD8+ T cells B. Testicular macrophages
D. Killer cells C. Fibroblast cells
E. Macrophages D. Interstitial cells of leydig
E. Sertoli cells
142. What is the single best description
for the histological appearance of the 148. A 63-year-old woman develops
secretory endometrium? cough with bloody sputum and night
sweats. Exposure to Mycobacterium
A. Simple columnar tuberculosis was documented 20 years
B. Simple columnar with subnuclear ago, and M. tuberculosis is identified in
vacuolation her sputum. A chest X-ray reveals
C. Stratified columnar
bilateral apical granulomas. Which
D. Stratified cuboidal
prominent cells are found in the
E. Simple cuboidal epithelium tuberculosis granuloma?
143. The tissue in which most A. Neutrophils
lymphocytes are produced is the B. Eosinophils
A. Liver C. Plasma cells
B. Spleen D. Lymphocytes
C. Thymus E. Macrophages
D. Bones marrow 149. Following radical hysterectomy
E. Lymph nodes
and pelvic lymphadenectomy for Stage I
144. What is the most common adverse carcinoma of the cervix, 63-year-old
effect experienced by women taking the woman reports numbness along the
progestogen-only pill? medial thigh as well as weakness of hip
adduction. Which nerve most likely to
A. Acne have been injured during the operation?
B. Weight gain
C. Erratic bleeding A. Obturator nerve
D. Reduced libido B. Femoral nerve
E. Mood swings C. Inferior gluteal nerve
D. Superior gluteal nerve
145. Inhibin is secreted by which cell E. Sciatic nerve
type of the ovary?
150. Pregnant women, in the second or
third trimester, may develop diabetes

16 ‫صفحة‬ Recall September 2015


insipidus due to excessive placental 155. Which of the following nerves
elaboration of enters the thigh by passing beneath the
inguinal ligament, just medial to the
A. Antidiuretic hormone anterior superior iliac spine?
B. Arginine-vasopressin
C. Vasopressinase A. Femoral nerve
D. Renin B. Iliohypogastric nerve
E. Neurophysin C. Ilioinguinal nerve
D. Genitofemoral nerve
151. A 30-year-old para 1 woman with E. Lateral cutaneous nerve of thigh
previous normal delivery went into
spontaneous labour at term and was 156. What is the volume for total blood
fully dilated for 3 hours. After pushing volume that rises in normal pregnancy?
for 90 minutes, ventouse delivery was
performed for maternal exhaustion. At A. 5 ml/kg
the ventouse cup application area, the B. 10 ml/kg
baby's head is left with a "chignon". In C. 25 ml/kg
which layer of the skull chignon is D. 50 ml/kg
found? E. 100 ml/kg

A. Subcutaneous 157. What is the most common side


B. Subaponeurotic effect with etonogestrel implant?
C. Subperiosteal
D. Subdural A. Amenorrhea
E. Extradural B. Infrequent bleeding
C. Weight gain
152. A woman who had a total D. Acne
abdominal hysterectomy, according to E. Headache
recent protocols, urinary catheter
should be removed for at least 158. Which of the following agents can
cause premature closure of the patent
A. 6 hrs ductus arteriosus?
B. 12 hrs
C. 24 hrs A. Prostaglandin E1
D. 48 hrs B. Nitric oxide
E. 72 hrs C. Cortisone
D. Acetaminophen
153. When the body is in the upright E. Indomethacin
position, the expected angle between the
brim of the pelvis to the horizontal 159. Use of which of the following
plane of approximately: agents during pregnancy is associated
with "floppy infant syndrome?"
A. 15 degrees
B. 40 degrees A. Naproxen
C. 60 degrees B. Diazepam
D. 90 degrees C. Nystatin
E. 120 degrees D. Warfarin
E. Phenytoin
154. Which chromosomes are capable of
robertsonian translocations? 160. What landmark marks where the
external iliac artery becomes the
A. X, Y femoral artery?
B. 15, 17, 19
C. 16, 18, 20 A. Crest of the ilium
D. 9, 10, 11, 12 B. Inguinal ligament
E. 13, 14, 15, 21, 22 C. Pubic tubercle
D. Symphysis pubis

17 ‫صفحة‬ Recall September 2015


E. Obturator foramen A. 1
B. 2
161. Androgen is secreted from which C. 4
region of the adrenal gland? D. 6
E. 8
A. Adrenal capsule
B. Zona reticularis 167. Chromosomal pattern of turner
C. Zona glomerulosa syndrome is
D. Zona fasciculata
E. Adrenal medulla A. 47/XX
B. 46/XXY
162. A nineteen year old girl presents to C. 47/XXY
the clinic and reports that she never had D. 46/X0
a menstrual period. On examination she E. 45/X0
is short, wide spaced nipples, webbed
neck with coarctation of aorta. What is 168. One researcher wishes to assess the
the most likely diagnosis? postoperative recover for a group of
postmenopausal women who had
A. Mullerian agenesis hysterectomy. He classified the patients
B. 5-alpha reductase deficiency as fell "worse," "same," or "improve".
C. Turner syndrome Which level of measurement is this
D. Anorexia nervosa classification?
E. Primary hypothyroidism
A. Nominal
163. Which of the following placental B. Ordinal
hormones helps ensure adequate fetal C. Continuous
nutrition? D. Interval
E. Ratio
A. Estriol
B. Progesterone 169. Kolmogorov smirnov (K-S) test is
C. Human placental lactogen
D. Human chorionic gonadotropin A. Parametric test compare two
E. Human chorionic related, or dependant samples
adrenocorticotropin B. Non-parametric test compare two
related, or dependant samples
164. Chromosomes arrange themselves C. Parametric test compare two non-
on equatorial plate in which phase of related, or independent samples
the cell cycle? D. Non-parametric test compare two
non-related, or independent
A. Anaphase samples
B. Metaphase E. Non-parametric test compare more
C. Prophase than two samples with unpaired
D. Synthetic phase data
E. Telophase
170. Lymphatics of the sigmoid colon
165. Which intracellular organelle is drain into
concerned with processes, packages and
transports proteins out of the cell? A. Superior mesenteric and Pre aortic
lymph nodes
A. Smooth endoplasmic reticulum B. Inferior mesenteric and Pre aortic
B. Rough endoplasmic reticulum lymph nodes
C. Free ribosomes C. Internal iliac lymph nodes
D. Lysosomes D. External iliac lymph nodes
E. Golgi apparatus E. Obturator lymph nodes
166. How many telomeres are in the cell 171. Which neoplasm is
in metaphase of mitosis? characteristically associated with

18 ‫صفحة‬ Recall September 2015


paraneoplastic syndrome of 176. A 37-year-old woman presents to
inappropriate antidiuretic hormone clinic complaining of lower abdominal
(SIADH)? discomfort. Exam of uterus is enlarged.
Biopsy shows normal appearing glands
A. Renal cell carcinoma in myometrium. What is the most likely
B. Breast carcinoma diagnosis?
C. Small cell lung carcinoma
D. Papillary thyroid carcinoma A. Adenocarcinoma
E. Cancer colon B. Basal cell carcinoma
C. Leiomyosarcoma
172. What is the basic principle of the D. Adenomyosis
Kleihauer-Betke test? E. Endometriosis
A. Detection the presence of 177. Consider the following pedigree:
immunoglobulin on RBCs in vivo
B. Migration of Adult hemoglobins
from anode to cathode
C. Fetal hemoglobin is resistant to
acid elution
D. Fetal hemoglobin is resistant to
denaturation by alkali
E. Fetal hemoglobin is resistant to
denaturation by heat

173. Glucagon works by which of the What is the mode of inheritance?


following mechanisms to reverse the A. Autosomal dominant
hypoglycemia? B. Autosomal recessive
C. Mitochondrial inheritance
A. Glycolysis D. X-linked recessive
B. Glycogenesis E. X-linked dominant
C. Gluconeogenesis
D. krebs cycle 178. What type of Laser is Neodymium:
E. Oxidative phosphorylation YAG laser?

174. A 19-year-old married female A. Semiconductor laser


complains of vaginal discharge, odor, B. Excimer Laser
and itching. Speculum examination C. Dye Laser
reveals a homogeneous grey discharge, D. Gas Laser
vulvar and vaginal erythema, and a E. Solid Laser
"strawberry" cervix. The most likely 179. Via which pattern of inheritance is
causative organism is a positive father will give the condition
A. Candida albicans to all of his daughters, but not his sons,
B. Gardnerella vaginalis whereas a positive female will transmit
C. Trichomonas vaginalis the trait to half of her sons and half of
D. Chlamydia trachomatis her daughters.
E. Herpes simplex type 2 A. Autosomal recessive
175. Which of the following laboratory B. X-linked recessive
techniques is used for RNA analysis? C. X-linked dominant
D. Mitochondrial inheritance
A. Northern blotting E. Y-linked dominant
B. Southern blotting
C. Southwestern blotting 180. A G1P0 patient with a dilated
D. Western blotting cardiomyopathy and an ejection
E. Eastern blotting fraction (EF) of 28% presents for a

19 ‫صفحة‬ Recall September 2015


caesarean section. What does EF 28%  Serum T4 178 mmol/l
mean?  Serum TSH 0.05 mU/L
 Prolactin 400 mmol/l
A. 72% of the total amount of blood
pumped into the ventricles prior to What is the most likely diagnosis?
systole
B. 72% of the total amount of blood A. Primary hypothyroidism
left over in the ventricles B. Graves' disease
following systole C. Hashimoto's thyroiditis
C. 28% of the total amount of blood D. Multi-nodular goitre
left over in the ventricles E. DeQuervain's thyroiditis
following systole
D. 28% the total amount of blood 184. A 30-year-old second gravida at 10
pumped by the heart during a 1- weeks pregnant wants to know about
minute period antenatal Group B streptococcus
E. 72% of the total amount of blood measures. Which of the following
pumped by the heart during a 1- circumstances is an indication for GBS
minute period screening?

181. What are the cells in the region A. Previous baby with invasive GBS
represented by Arrow X in the picture infection
called? B. Vaginal swab positive for GBS in
current pregnancy
C. Woman carriage GBS in previous
pregnancy
D. Routine antenatal screening for
GBS carriage
E. No need for routine antenatal
screening for GBS carriage
A. Decidua 185. A 24-year-old pregnant woman
B. Blastocele presents with persistent vomiting. She
C. Inner cell mass becomes tachypanic. Which is the most
D. Cytotrophoblast cells likely acid-base disorder in this patient?
E. Syncytiotrophoblast cells
A. Metabolic acidosis
182. A 23-year-old woman attends the B. Metabolic alkalosis
antenatal clinic in her first pregnancy. C. Mixed metabolic alkalosis and
Booking bloods reveal her Blood group respiratory acidosis
O and her rhesus status to be cde/cde; D. Respiratory acidosis
her long-standing partner is also tested E. Respiratory alkalosis
at her request and his blood group A
with rhesus status CDe/CDe. What is 186. A 25-year old primigravida with
the expected blood group of her fetus? underlying sickle cell disease presents at
32 weeks of pregnancy with intense
A. A positive itching. A set of liver function tests is
B. A negative requested, with the following results:
C. O positive ALT normal, bilirubin normal, albumin
D. O negative slightly low, bile acid high and ALP
E. AB positive slightly high. What is the most likely
diagnosis?
183. A 32-year-old female presents with
a two month history of breathlessness, A. Acute fatty liver
tachycardia and menstrual irregularity. B. Obstetric cholestasis
Her Investigations revealed: C. Haemolytic jaundice
D. Obstructive jaundice
 Serum hCG negative

20 ‫صفحة‬ Recall September 2015


E. Sickle cell crisis 192. Which statistical test requires a
normal population distribution?
187. What type of necrosis is seen in the
heart after an acute myocardial A. Chi-square test
infarction? B. Mann-Whitney U test
C. Spearman's rank correlation
A. Liquefactive necrosis D. Variance estimation test
B. Coagulative necrosis E. Wilcoxon rank sum test
C. Caseous necrosis
D. Gangrenous necrosis 193. Maintenance of the corpus luteum
E. Fibrinoid necrosis during the first trimester of pregnancy
is accomplished principally by the
188. A normal woman is considering secretion of
having a child with her partner who is
carrier for cystic fibrosis. What is the A. Antidiuretic hormone (ADH)
probability of them having an affected B. Follicle stimulating hormone
child? (FSH)
C. Human chorionic gonadotropin
A. 0% (hCG)
B. 25% D. Luteinizing hormone (LH)
C. 50% E. Progesterone
D. 75%
E. 100% 194. What is the blood supply of
sigmoid colon?
189. Which of the following
antihypertensive drugs induced A. Celiac trunk
postnatal depression? B. Superior mesenteric artery
C. Inferior mesenteric artery
A. Alpha methyl dopa D. External iliac artery
B. Hydralazine E. Internal iliac artery
C. Labetalol
D. Amlodipine 195. The following intrauterine growth
E. Captopril chart shows the 10th, 50th and 90th
weight percentiles by gestational ages.
190. What level of measurement uses a
verbal scale to measure data; slight,
moderate and severe?

A. Continuous
B. Nominal
C. Ordinal
D. Interval
E. Ratio

191. Mean hemoglobin of a sample of What is the correct interpretation of


100 pregnant women was found to be 10 this chart?
mg% with a variance of 1.0 mg%. The
standard error of the estimate would A. Appropriate for gestational age
be: (AGA)
B. Small for gestational age (SGA)
A. 0.01 C. Symmetrical intrauterine growth
B. 0.1 restriction (IUGR)
C. 1.0 D. Asymmetrical intrauterine growth
D. 10.0 restriction (IUGR)
E. 100 E. Large for gestational age (LGA)

21 ‫صفحة‬ Recall September 2015


196. A primigravida is in labour at 38 B. interstitial cystitis
weeks of gestation. Midwife has called C. Urinary stress incontinence
you as the mother is exhausted and D. Bladder overactivity
CTG is showing prolonged late E. Sensory neurogenic bladder
decelerations, a baseline of 170 bpm and
an adequate variability. On 198. A 29-year-old woman (gravida 2,
examination, none of the presenting para 1) is admitted to delivery suite at
part is palpable per abdomen. The term. Few minutes after admission the
cervix is fully dilated, vertex is at +1 membranes rupture. On examination,
station and position is occipto posterior. thick meconium is noted. Her cervix is 5
What is the most appropriate cm dilated with vertex at 0 station. The
management? midwife commences a cardiotocograph
trace, which shows one variable
A. Wait for spontaneous vaginal deceleration, one acceleration, a
delivery baseline of 120 bpm and an adequate
B. Perform fetal blood sample variability. Contractions are regular, 3
C. Instrumental delivery in the labour every 10 minutes. What would be your
ward initial management?
D. Instrumental delivery in the
operating theater A. Continuous fetal monitoring
E. Emergency caesarean section B. Perform fetal blood sample
C. Assisted vaginal delivery
197. Illustrates the bladder diary of 48 D. Deliver immediately by caesarean
year old woman who is a teacher section (category 1)
complains of urinary urgency, and leaks E. Deliver by caesarean section
urine if she is unable to reach the toilet (category 2)
quickly. Since she was unable to
measure her voids, she was asked to put 199. What is the correct formula to
a tick in the column each time she calculate the positive predictive value?
passed urine. One day of three-day If TP = true positive; FP = false
charting hows:s positive; TN = true negative; FN = false
negative

A. TN / (TN + FN)
B. TP / (TP + FP)
C. TN / (TN + FP)
D. TP / (TP + FN)
E. TN / (TP+FP+FN)

200. A revolutionary tumour marker


has been trialed for detecting
endometrial carcinoma. 140 women
» Daytime frequency: 12 times in 24 were included in the trial; the result
hours shows that 90 women had a positive
tumour marker however only 30 had
» Nocturia: twice in 24 hours endometrial cancer. Five women who
had endometrial cancer were negative
» Leakage with urgency occurred once
for the tumour marker. The sensitivity
in 24 hours
of the new tumour marker is
» Fluid intake: 1,450ml, made up of six
A. 40
cups of coffee and a pint of lager
B. 45
What is the correct interpretation of C. 60
this urinary diary? D. 75
E. 85
A. Excessive fluid intake

22 ‫صفحة‬ Recall September 2015


MRCOG
RECALLS

2016 – 2017 – 2018 - 2019

WRITTEN BY: Dr. MOHAMED YAHIA ABUELGASEM KHATEM


MRCOG RECALLS

EXAM 1

2 ‫صفحة‬ Dr. MOHAMED YAHIA 0900988745


MRCOG RECALLS

1. What percentage of lymphocytes is E. HLA-G


present in peripheral blood?
6. How many sperm are produced in the
A. 2-8% testes each day?
B. 15-20%
C. 20-40% A. 30,000
D. 50-70% B. 300,000
E. 70-80% C. 3,000,000
D. 30,000,000
2. A 65-year-old woman has a sudden E. 300,000,000
onset loss of speech and hemiparesis. On
arrival in hospital her symptoms and 7. Clomifene citrate is a commonly used
neurological examination is suggestive drug for the induction of ovulation.
of a cerebrovascular incident. What type of drug is clomifene citrate?
Subsequent imaging supports the
A. Androgenic steroid
diagnosis of an ischaemic stroke,
B. Estrogenic steroid
affecting her left cerebral hemisphere.
Which of the following forms of tissue C. Progestogenic steroid
necrosis is associated with her loss of D. Gonadotropin analogue
E. Selective estrogen receptor
function?
modulator
A. Caseous necrosis
B. Coagulative necrosis 8. The complement system consists of a
C. Colliquative necrosis number of small proteins found in the
D. Fat necrosis blood. Which of the following is the
E. Gangrenous necrosis major site of synthesis of complement
proteins?
3. Which of the following
A. Liver
immunoglobulin class has a dimeric
B. Spleen
structure?
C. Thymus
A. IgG D. Bone marrow
B. IgA E. Intestinal epithelium
C. IgM
D. IgD 9. Botulinum toxin is now used to treat
E. IgE neurogenic bladder because of its ability
to
4. Which of the following is the main
cellular origin of clotting factor VIII A. Block storage of acetylcholine
ACH in neuronal vesicles
(antihemophilic factor)?
B. Inhibit the release of acetylcholine
A. Hepatocytes ACH from the neurons
B. Histiocytes C. Inhibit acetylcholine ACH
C. Megakaryocytes synthesis
D. Plasma cells D. Block bladder nicotinic receptors
E. Vascular Endothelium E. Block bladder muscarinic
receptors
5. Which class I major
histocompatibility complex human 10. Which of the following structures
leucocyte antigen (HLA) is expressed passes through the lesser sciatic
only in extravillous trophoblast? foramen?

A. HLA-A A. Iliococcygeus
B. HLA-B B. Piriformis
C. HLA-C C. Sciatic nerve
D. HLA-F D. Posterior femoral cutaneous nerve
E. Tendon of obturator internus

3 ‫صفحة‬ Dr. MOHAMED YAHIA 0900988745


MRCOG RECALLS

11. A 50-year-old woman, recently B. Superior mesenteric artery


diagnosed with ovarian cancer, started C. Inferior mesenteric artery
her first cycle of chemotherapy. Several D. Right umbilical artery
days after a treatment, she notes that E. Intercostal artery
she has blood in her urine. Cystoscopy
reveals haemorrhagic cystitis. Which of 16. What of the following is the major
the following antineoplastic drugs is buffer in urine?
most likely responsible for this side
A. Bicarbonate
effect?
B. Phosphate
A. Etoposide C. Ammonia
B. Methotrexate D. Calcium carbonate
C. Paclitaxel E. Potassium chloride
D. Cyclophosphamide
E. Vincristine 17. Glucocorticoids are secreted from
which region of the adrenal gland?
12. Luteinizing hormone (LH) is
structurally related to what other A. Adrenal capsule
B. Zona reticularis
glycoprotein hormones?
C. Zona glomerulosa
A. ADH, oxytocin and FSH D. Zona fasciculata
B. FSH, TSH and growth hormone E. Adrenal medulla
C. HCG, FSH and TSH
D. HCG, ADH and FSH 18. How many carbon atoms do
E. TSH, growth hormone and HCG androgens contain?

13. Approximately what percentage of A. 18


B. 19
the total human DNA is mitochondrial
C. 20
DNA?
D. 21
A. 1% E. 22
B. 20%
C. 50% 19. From which germ cell layer is the
D. 80% allantois derived?
E. 99% A. Endoderm
14. Your consultant asks you to B. Ectoderm
prescribe a 3 month course of ulipristal C. Paraxial mesoderm
acetate to a patient with fibroids prior D. Intermediate mesoderm
to having a hysterectomy. To which E. Lateral plate mesoderm
class of drugs does ulipristal acetate 20. Which process establishes the three
belong? definitive germ layers?
A. Aromatase inhibitor A. Neurulation
B. Gonadotrophin releasing hormone B. Gastrulation
(GnRH) antagonist C. Craniocaudal folding
C. Progestogen antagonist D. Lateral folding
D. Prostaglandin analogue E. Angiogenesis
E. Selective estrogen receptor
modulator (SERM) 21. Which one of the following is the
termination of round ligament?
15. Which of the following arteries
supplies foregut derivatives of the A. Labia minora
digestive system? B. Labia majora
C. Deep Inguinal ring
A. Celiac trunk D. Superficial Inguinal ring

4 ‫صفحة‬ Dr. MOHAMED YAHIA 0900988745


MRCOG RECALLS

E. Lateral vaginal wall 27. Ondansetron is an effective


antiemetic. Which of the following best
22. Ductus venosus is completely describes the mechanism of action of
obliterated after birth to form what Ondansetron?
adult structure?
A. Histamine H1-receptor agonist
A. Ligamentum venosum B. Histamine H1-receptor antagonist
B. Ligamentum arteriosum C. Serotonin 5-HT3 receptor
C. Ligamentum teres antagonist
D. Medial umbilical ligament D. Dopamine D2 receptor agonist
E. Median umbilical ligament E. Dopamine D2 receptor antagonist
23. An inherited metabolic disorder 28. What type of joint is formed at the
called phenylketonuria (PKU) can sacroiliac joint?
result in serious problems in infancy.
Via which pattern of inheritance is the A. Primary cartilaginous
PKU? B. Secondary cartilaginous
C. Gomphosis
A. Mitochondrial D. Condyloid
B. Autosomal dominant E. Synovial
C. Autosomal Recessive
D. X-Linked Dominant 29. Which one of the following stages of
E. X-Linked Recessive the cell cycle is cell prepared for DNA
synthesis?
24. The ureteric bud is derived from
which of the following embryonic A. Synthetic phase (S)
structures? B. Gap phase 1 (G1)
C. Gap phase 0 (G0)
A. Allantois D. Gap phase 2 (G2)
B. Pronephros E. Mitotic phase (M)
C. Mesonephros
D. Mesonephric duct 30. The formation of uric acid from
E. Metanephric blastema purines is catalyzed by the action of

25. Which part of the endometrium is A. Nucleoside phosphorylase


deep to the fetus and forms the B. Xanthine oxidase
maternal component of the placenta? C. Glutaminase
D. Urease
A. Decidua capsularis E. Uricase
B. Decidua rubra
C. Decidua vera 31. A 19-year-old woman gives birth to
D. Decidua basalis her first child. She begins breast feeding
E. Decidua menstruales the infant. She continues breast feeding
for almost a year with no difficulties
26. Choriocarcinoma is a malignant and no complications. Which of the
form of gestational trophoblastic following cellular processes that
disease. When choriocarcinoma occurred in the breast during
metastasises, it has a propensity for pregnancy allowed her to nurse the
which type of spread? infant for this period of time?
A. Haematogenous A. Stromal hypertrophy
B. Implantation B. Epithelial dysplasia
C. Lymphatic C. Steatocyte atrophy
D. Transcoelomic D. Ductal dilation
E. Transplantation E. Lobular hyperplasia

5 ‫صفحة‬ Dr. MOHAMED YAHIA 0900988745


MRCOG RECALLS

32. What is the embryonic origin of the E. Direct-acting smooth-muscle


trigone of urinary bladder? relaxant

A. Mesonephric duct 37. The deep circumflex iliac artery is a


B. Paramesonephric duct branch of which artery?
C. Ureteric bud
D. Urogenital sinus A. Internal iliac artery
E. Caudal of cloaca B. Internal pudendal artery
C. External iliac artery
33. Misoprostol is a commonly used D. Common iliac artery
drug in the medical treatment of E. Femoral artery
miscarriage. What type of drug is
misoprostol? 38. What is the SI unit for the absorbed
dose of ionizing radiation?
A. Cyclooxygenase inhibitor
B. Progesterone antagonist A. Gray
C. Synthetic prostaglandin E1 B. Sievert
analogue C. Rad
D. Synthetic prostaglandin E2 D. Roentgen
analogue E. becquerels
E. Synthetic prostaglandin f2α
analogue 39. Which organ is the principal site of
urea synthesis via the urea cycle?
34. What is the most common contrast
A. Liver
agent used for MRI examinations?
B. Kidneys
A. Iodine C. GIT
B. Barium D. Lungs
C. Thallium E. Bone marrow
D. Gadolinium
E. Gallium 40. What is the most common cause of
caushing's syndrome?
35. Defective feedback control of
calcium hemostasis may lead to A. Pituitary adenoma
secondary hyperparathyroidism. It is B. Adrenal adenoma
C. Adrenal carcinoma
usually associated with
D. Adrenal hyperplasia
A. Parathyroid gland tumour E. Iatrogenic steroid administration
B. Chronic liver disease
C. Thyroid carcinoma 41. Which of the following is a remnant
D. Chronic kidney disease of the mesonephric duct in the female?
E. Syndrome of inappropriate A. Round ligament
antidiuretic hormone (SIADH) B. Ovarian ligament
C. Broad ligament
36. What is the mechanism of action of
D. Gartner duct
labetalol?
E. Mullerian tubercle
A. Antimuscranic
B. Selective alpha 1 receptors
antagonist 42. What is the most common cause of
C. Non selective beta receptors hyperprolactinemia?
antagonist
D. Combines selective alpha 1 and A. Macroprolactinoma
non selective beta receptors B. Microprolactinoma
antagonist C. Primary hypothyroidism
D. Chronic renal failure

6 ‫صفحة‬ Dr. MOHAMED YAHIA 0900988745


MRCOG RECALLS

E. Pituitary stalk interruption B. Statified Squamous, non


syndrome (PSIS) keratinised
C. Statified Squamous, keratinised
43. Where in the kidney is the majority D. Cuboidal
of bicarbonate reabsorbed? E. Transitional
A. Bowman's capsule 49. A patient asks you how long it will
B. Collecting duct be before her C-section wound is "at
C. Distal convoluted tubule maximum strength." What is the typical
D. loop of Henle time taken for wounds closed by
E. proximal convoluted tubule primary intention to reach maximum
tensile strength?
44. What type of inheritance pattern
does duchenne muscular dystrophy A. 14 days
follow? B. 21 days
C. 42 days
A. Mitochondrial D. 90 days
B. Autosomal dominant E. 120 days
C. Autosomal Recessive
D. X-Linked Dominant 50. In the developing foetal testis, the
E. X-Linked Recessive cells that produce mullerian-inhibiting
substance are the
45. What is the Lymphatic drainage of
cervix and upper vagina? A. Interstitial cells of leydig
B. Interstitial macrophages
A. Internal iliac LN C. Sertoli cells
B. External iliac LN D. Myoid cells
C. Aortic LN E. Germ cells
D. External iliac + internal iliac +
common iliac + aortic LNs 51. A 27-year-old woman who is 8
E. External iliac + internal iliac + weeks' gestation presents to your clinic
obturator + sacral LNs with increased sweating on her face. On
examination there are no remarkable
46. Vertical transmission of HIV findings. Which of the following
infection mostly occurs endocrine glands is related to increase
A. In the first trimester activity of sweat glands during
B. In the second trimester pregnancy?
C. In the third trimester A. Adrenal cortex
D. During labour and delivery B. Placenta
E. In the neonatal period through C. Pancreas
breastfeeding D. Parathyroid
47. From what parent compound are E. Thyroid
most eicosanoids derived? 52. A 60-year-old woman presents with
A. Arachidonic acid an erythematous erosive lichen planus
B. Leukotriene on the vulva. What type of cancer is
C. Prostacyclin linked to lichen planus?
D. Prostaglandins A. Squamous cell carcinoma
E. Cholesterols B. Basal cell carcinoma
48. What kind of epithelium lines the C. Merkel cell carcinoma
ectocervix? D. Malignant melanoma
E. Kaposi sarcoma
A. Columnar

7 ‫صفحة‬ Dr. MOHAMED YAHIA 0900988745


MRCOG RECALLS

53. Which of the following hormones 58. What is the most common adverse
enhances urinary excretion of calcium? effect experienced by women using
depot medroxyprogesterone acetate
A. Insulin (DMPA)?
B. Cortisol
C. Calcitonin A. Menstrual abnormalities
D. parathyroid hormone B. Loss of bone mineral density
E. Antidiuretic hormone C. Weight gain
D. Injection site reactions
54. Which immunoglobulin is a key to E. Headache
passive neonatal immunity?
59. A 24-year-old female presents with a
A. IgD 2-year history of infertility. An
B. IgE endometrial biopsy is obtained
C. IgM approximately 5 to 6 days after the
D. IgA predicted time of ovulation. This biopsy
E. IgG specimen reveals ''marked glandular
growth and mitosis, stromal edema with
55. Which of the following tocolytic subnuclear vacuoles and prominent
agents exert their effects by blocking the coiled spiral arterioles'' This finding are
function of the oxytocin receptor? consistent with what histological type of
A. Atosiban endometrium?
B. Magnesium sulphate A. Atrophic
C. Nifedipine B. Proliferative
D. Progesterone C. Secretary
E. Terbutaline D. Simple Hyperplasia
56. Which of the following hormones E. Arias-Stella phenomenon
increase after ovulation and decrease 60. A 28-day-old baby is brought to the
the proliferation of endometrium? clinic because of projectile vomiting
A. Progesterone after feeding. Until this time, the baby
B. Follicle-stimulating hormone has had no problems in feeding. On
(FSH) examination, a small knot is palpated at
C. Estrogen the right costal margin. A diagnosis of
D. Luteinizing hormone (LH) hypertrophic pyloric stenosis is
E. Gonadotropin-releasing hormone suspected. Which metabolic
(GnRH) abnormalities are seen in congenital
hypertrophic pyloric stenosis?
57. You are assessing a patient with an
ovarian mass and are using the Risk of A. Hypochloremic hypokalemic
Malignancy Index (RMI) score. Which metabolic alkalosis
of the following is used to calculate the B. Hyperchloremic hypokalemic
RMI? metabolic alkalosis
C. Hypochloremic hypokalemic
A. U (ultrasound score) x M metabolic acidosis
(menopause score) x Ca125 level D. Hyperchloremic hyperkalemic
B. U (ultrasound score) + M metabolic acidosis
(menopause score) + Ca125 level E. Hyperchloremic hypokalemic
C. U (ultrasound score) / M metabolic acidosis
(menopause score) x Ca125 level
D. U (ultrasound score) x M 61. Following is the narrowest part of
(menopause score) / Ca125 level fallopian tube?
E. U (ultrasound score) þ M A. Intramural part
(menopause score) / CA19-9 level B. Isthmus

8 ‫صفحة‬ Dr. MOHAMED YAHIA 0900988745


MRCOG RECALLS

C. Ampulla 3. Internal sphincter


D. Infundibulum
E. Fimbriae 4. Superficial part of external sphincter

62. Lancefield grouping of streptococci A. 1,4,2,3


is based on the presence of B. 2,1,4,3
C. 3,1,4,2
A. Carbohydrate antigen on cell wall D. 3,2,1,4
B. Hemolytic reactions on blood agar E. 2,4,1,3
C. M protein
D. Pigment production 67. A 33-year-old woman with an
E. Sugar fermentation intrauterine contraceptive device
develops symptoms of acute salpingitis.
63. Which enzyme found in red blood On laparoscopy, sulfur granules appear
cells which buffers blood CO 2 ? at the fimbria of the tubes. Which of the
following is the most likely organism?
A. Cytochrome-b5 reductase
B. 5-nucleotidase A. C. trachomatis
C. Glucose-6-phosphate B. Nocardia species
dehydrogenase C. N. gonorrhea
D. Carbonic anhydrase D. T. pallidium
E. Pyruvate kinase E. Actinomyces species

64. A 35-year-old pregnant woman with 68. A 30-years-old primigravida


multiple drug resistant (MDR) presents in a spontinous labour at 41
tuberculosis is receiving prolonged weeks of gestation. On vaginal
treatment with streptomycin. Prolonged examination the cervix is 8 cm dilated
administration of streptomycin during and the position of the vertex is left
pregnancy may result damage to the occipto-posterior. What is the
presenting diameter?
A. Optic nerve
B. Trigeminal nerve A. Bitemporal
C. Facial nerve B. Occipito Frontal
D. Vestibulocochlear nerve C. Submento Bregmatic
E. Glossopharyngeal nerve D. Sub-occipito Bregmatic
E. Vertico Mental
65. Increased glomerular filtration rate
during normal pregnancy is a 69. A neoplasm of the female genital
physiological finding results mainly tract occurring in an 18-year-old girl
from whose mother was treated with
diethylstilbestrol during the pregnancy
A. Increased cardiac output is likely to be a
B. Increased Plasma volume
C. Decreased peripheral vascular A. Brenner cell tumor of ovary
resistance B. Teratoma of ovary
D. Constriction of the afferent C. Sarcoma botryoides of vagina
arteriole D. Clear cell carcinoma of vagina
E. Compression of the renal capsule E. Squamous cell carcinoma of vulva

66. Which of the following describes the 70. Six hours after undergoing a
correct order of musculature of the anal laparoscopic tubal ligation, a 32-year-
canal from deep to superficial? old woman complains of suprapubic
pain and oozing. On examination there
1. Deep part of external sphincter is a tender palpable suprapubic mass
rising up to the umbilicus that is dull to
2. Subcutaneous part of external
sphincter

9 ‫صفحة‬ Dr. MOHAMED YAHIA 0900988745


MRCOG RECALLS

percussion. Her hemoglobin level is 11 C. Isoechoic


g/dl. What is the most likely diagnosis? D. Hyperechoic
E. Normoechoic
A. Bladder injury
B. Intra-abdominal bleeding 75. Which of the following arteries may
C. Ureteric trauma occasionally arise as a branch of the
D. Urinary retention external iliac artery or inferior
E. Fluid overload epigastric artery instead of as a branch
of the internal iliac artery?
71. A grand multiparous woman has a
postpartum haemorrhage soon after A. Internal pudendal artery
delivery. One liter of a colloid solution B. Obturator artery
has already been given intravenously. C. Superior gluteal artery
She has a pulse rate of 140 beats per D. Umbilical artery
min, is anxious, confused and E. Uterine artery
hypotensive with a capillary refill time
of 3 seconds. What do you think is the 76. Blood supply to the medial part of
estimated blood loss as a percentage of the anterior abdominal wall below the
blood volume thus far based on clinical umbilicus
findings?
A. Superficial epigastric artery
A. 5-10% B. Superior epigastric artery
B. 15-20% C. Superior mesenteric artery
C. 25-30% D. Inferior epigastric artery
D. 30-40% E. Inferior mesenteric artery
E. 45-55%
77. Some-town in the UK, has a
72. A 31-year-old nulliparous woman population of 350,000. During 2010,
has heavy bleeding at 8 weeks’ data pertaining to deliveries and there
gestation. An early pregnancy scan is outcome is as follows:
suggestive of a molar pregnancy, and no
fetus is observed. What is the most  Total number of live births: 20000
likely genotype of a complete molar  Total number of in utero deaths
pregnancy? delivered bet. 22 +0 and 23 +6 weeks
gestation: 200
A. 46 XX  Total number of stillbirths: 140
B. 46 XY  Total number of neonatal deaths
C. 69 XXX during first week after birth: 40
D. 69 XXY  Total number of neonatal deaths
E. 92 XXXY during second to fourth weeks after
birth: 80
73. Mechanism of action of Neostigmine
is Which of the following represents the
perinatal mortality rate in this town
A. Choline ester during 2009?
B. Cholinomimetic
C. Muscarinic antagonist A. 140/20,120
D. Anticholinesterase B. 140/20,120
E. Nicotinic antagonist C. 180/20,140
D. 180/20,260
74. Which term best describes a E. 380/20,140
homogeneously black area on an
ultrasound image? 78. A 30-year-old woman with bipolar
disorder is 12-week pregnant. She has
A. Anechoic been taken her lithium every night.
B. Hypoechoic Which of the following abnormality

10 ‫صفحة‬ Dr. MOHAMED YAHIA 0900988745


MRCOG RECALLS

may occur if she continues to take C. Anterior femoral cutaneous


lithium? D. Obturator
E. Femoral
A. Tetralogy of Fallot's
B. Dandy walker syndrome 83. The following cell contain haploid
C. Ebstein's anomaly number of chromosomes and results
D. Limb reduction deformity after completion of the 1 st meiotic
E. Renal agenesis division

79. Which part of the cell cycle is noted A. Oogonium


for sister chromatids separating and B. 2 nd polar body
moving to opposite sides of the cell? C. 1 ry oocyte
D. 2 ry oocyte
A. Anaphase E. Zygot
B. Metaphase
C. Prophase 84. Which of the following techniques is
D. Synthesis phase used to increase the number of copies of
E. Telophase a specific DNA sequences?

80. Remifentanil is an ideal controlled A. Western blotting


analgesia for management of labour B. Southern blotting
pain because C. DNA chip
D. Polymerase chain reaction (PCR)
A. It is a strong μ receptor antagonist E. Fluorescent in situ hybridization
B. It has a long duration of action (FISH)
C. It is rapidly metabolized by tissue
estrases
D. Multiple smaller doses over time
decrease the incidence of side 85. A 40-year-old female suffers from a
effects 2 x 2-cm vulval lesion. Biopsy
E. It cannot cross the placenta demonstrates a malignant melanoma.
The prognosis of this tumour best
81. In the development of the oocyte, depends upon
when does the second meiotic division
occur? A. Grade of tumour
B. Darkness of lesion
A. At birth C. Degree of color variation
B. At fertilisation D. Depth of invasion
C. At menarche E. Duration of the growth
D. At ovulation
E. Before fertilisation 86. The estrogen receptor is an example
of a
82. A 48-year-old woman visits her GP
complaining of pain on the inner A. Guanylate cyclase receptor
surface of her right thigh. A detailed B. Nuclear transcription factor
history reveals that she has recently C. Tyrosine kinase nuclear receptor
experienced some dull, diffuse pain in D. Ligand-gated ion channel in the
the lower right abdominal wall. She is cell membrane
sent for a CT scan which reveals an E. G protein coupled receptor on the
inflamed right ovary. Which nerve is Golgi complex
referring pain from the parietal
peritoneum near the ovary to the inner 87. A 28-year-old woman has undergone
thigh? emergency hysterectomy due to
obstetric haemorrhage during her 3rd
A. Ilioinguinal pregnancy. The previous pregnancies
B. Saphenous were both term C-section deliveries.
Examination of the uterus shows the

11 ‫صفحة‬ Dr. MOHAMED YAHIA 0900988745


MRCOG RECALLS

placenta had invaded the myometrium 91. Which of the following structure
up to the serosa. What is the most likely consists of two bands of connective
diagnosis? tissue and also support the bladder?

A. Placenta praevia A. Urachus


B. Placenta accreta B. Pubocervical ligament
C. Placenta increta C. Uterosacral ligament
D. Invasive mole D. Coccygeus muscle
E. Placenta percreta E. Levator ani muscle

88. Which of the following metabolic 92. A 40-year-old woman complains of


processes produces the most energy heavy menstrual bleeding for five years.
needed for uterine contraction during Ultrasound scan has revealed no
parturition? abnormality. Her family is complete.
She desires an endometrial ablation.
A. Aerobic glycolysis Which part of the uterus must be
B. Anaerobic glycolysis destructed to stop bleeding?
C. krebs cycle
D. Oxidative phosphorylation A. The basal layer of endometrium
E. Beta oxidation B. The functional layer of
endometrium
89. A couple with primary subfertility, C. Both layer of endometrium
who have been trying to conceive for D. The entire myometrium
over 12 months, attends a reproductive E. The spiral artery
medicine clinic. The male partner has
already given a semen sample for 93. The rate of growth hormone
analysis with the following results: secretion follows a circardian rhythm.
When is the highest peak of growth
 Progressive motility: 35% hormone release?
 Total sperm number: 32 million
spermatozoa A. Early morning
 Normal morphology: 5% B. After exercise
 Vitality: 63% live spermatozoa C. Afternoon
 Liquefaction time: 30 min D. Early sleep
 pH: 7.3 E. Deep sleep

What is the correct interpretation of 94. The optimal ratio between suture
this semen analysis? and wound length that is associated
with a lower rate of incisional hernia is
A. Normal semen analysis
B. Low sperm count (oligospermia) A. 5:1
C. Decreased motility B. 4:1
(Asthenospermia) C. 1:2
D. High ph indicate an infection D. 2:1
E. Teratospermia E. 1:1

90. What is the most common inherited 95. Which nerve is identified by its
bleeding disorder? position on the anterior surface of the
psoas major muscle?
A. Von Willebrand’s disease
B. Hemophilia A A. Ilioinguinal
C. Hemophilia B B. Femoral
D. Protein C deficiency C. Obturator
E. Thrombotic thrombocytopenic D. Genitofemoral
purpura E. lateral femoral cutaneous

12 ‫صفحة‬ Dr. MOHAMED YAHIA 0900988745


MRCOG RECALLS

96. How many days does it take from 101. A 25-year-old woman has a routine
the resting follicle stage to ovulation? smear test for the first time. She
complains of discomfort during the
A. 14 days procedure. Which nerve or nerve plexus
B. 25 days provided sensory innervations to the
C. 80 days cervix?
D. 120 days
E. 300 days A. Pudendal nerve
B. Sacral splanchnic nerve
97. The capsule of the developing C. Pelvic splanchnic nerve
Graafian follicle is D. Superior hypogastric plexus
E. Inferior hypogastric plexus
A. Theca interna
B. Theca externa 102. In a female who has BRCA 1 gene,
C. Zona pellucida besides hereditary breast and ovarian
D. Zona granulosa cancer, what is the next most common
E. Lamina propria cancer she may develop?
98. The probability of sarcomatous A. Cancer colon
change occurring in uterine fibroids is B. Cancer stomach
C. Pancreatic cancer
A. 1/5 D. Cervical cancer
B. 1/10 E. Endometrial cancer
C. 1/100
D. 1/1000 103. What is the most common clinically
E. 1/3000 significant congenital heart disease?
99. After an elective caesarean section A. Atrial septal defect
the anesthetic team decided to perform B. Ventricular septal defect
an ultrasound-guided transversus C. Patent ductus arteriosus
abdominis plane block for analgesia. If D. Tetralogy of Fallot
the needle is correctly positioned where E. Coarctation of aorta
will be deposit the local anesthetic
104. Warfarin inhibits which clotting
A. Beneath the peritoneum factors?
B. Into the transversus abdominis
muscle A. Factor II
C. Between the transversus B. Factor V
abdominis muscle and the internal C. Factor VI
oblique muscle D. Factor VIII
D. Between the transversus E. Factor XI
abdominis muscle and the external
oblique muscle 105. The enzyme which builds an
E. Between the internal oblique and mRNA strand complimentary to the
the external oblique muscle DNA transcription unit is called?

100. The most common in uetro A. DNA polymerase


infection causing fetal anemia in the UK B. RNA polymerase
is C. Reverse transcriptase
D. Helicase
A. Syphilis E. DNA ligase
B. Toxoplasmosis
C. Rubella 106. Haemophilus ducreyi is the
D. Parvovirus B19 causative agent of
E. Cytomegalovirus
A. Hard chancre
B. Chancroid

13 ‫صفحة‬ Dr. MOHAMED YAHIA 0900988745


MRCOG RECALLS

C. Granuloma inguinale 111. You see a patient on the ward who


D. Lymphogranuloma venereum is acutely short of breath and perform
E. Sarcoid an arterial blood gases (ABG). The
results are as follows:
107. Which of the following divisions of
cell growth precedes Mitosis in the cell  pH 7.25
cycle?  pO2 8.6 kPa
 pCO2 8.4 kPa
A. Synthetic phase (S)  HCO 3 - 28 mmol/l
B. Gap phase 1 (G1)
C. Gap phase 0 (G0) What is the acid-base disturbance?
D. Gap phase 2 (G2)
E. Gap phase 3 (G3) A. Mixed Alkalosis
B. Respiratory Acidosis
108. A 26-year-old pregnant woman is C. Respiratory Alkalosis
found to have an Hb of 11 g/dL on a D. Metabolic Acidosis
routine blood test, with an MCV of 70. E. Metabolic Alkolosis
Serum electrophoresis reveals an Hb F
of 0.5 per cent, Hb A1 of 60 per cent, Hb 112. Which hemoglobin (Hgb) result
A2 of 2 per cent and Hb S of 40 per represents the lowest acceptable value
cent. Her ferritin levels are normal. The for a woman in the second trimester of
most likely diagnosis is pregnancy?

A. Iron deficiency anemia A. 9.5 g/dl


B. α-thalassemia B. 10 g/dl
C. β-thalassemia C. 10.5 g/dl
D. Sickle cell trait D. 11 g/dl
E. Sickle cell disease E. 11.5 g/dl

109. A 22-year-old primiparous woman 113. Terbutaline is a β-adrenergic


books her pregnancy at 11 weeks’ receptor stimulator that is used to
gestation. Her booking blood tests arrest preterm labor. Which of the
reveal a haemoglobin level of 10.1 g/dL. following is a major maternal risk of its
Haemoglobin electrophoresis reveals use?
homozygous for HbS, she has no HbA
A. Hypertension
A. Sickle cyprositis B. Decreased plasma glucose
B. Sickle cell disease C. Decreased serum potassium
C. Sickle cell trait D. Tachyarrhythmia
D. Sickle cell anemia E. Asthma
E. Sickle beta thalassemia
114. A 32-year old woman complains of
110. A 28-year-old Asian woman G2P2 amenorrhea since delivery of a baby 15
is 26 weeks pregnant. Her BMI is 30 months ago. The delivery was
kg/m 2 . Her oral glucose tolerance test complicated by excessive hemorrhage
reports (75 g load) are as follows: that required transfusion of 2.5 liters of
blood. Few days later she complained of
 Fasting serum glucose: 4.2 mmol/l failure to produce any breast milk. The
 2-hour serum glucose: 7.4 mmol/l most likely cause for this is damage to
which of the following endocrine
What is the diagnosis? glands?
A. Normal A. Hypothalamus
B. Impaired glucose tolerance B. Anterior pituitary
C. Impaired fasting glucose C. Posterior pituitary
D. Postprandial hyperglycemia D. Thyroid
E. Gestational D.M

14 ‫صفحة‬ Dr. MOHAMED YAHIA 0900988745


MRCOG RECALLS

E. Adrenal cortex and PH is 6. What is the most likely


causative organism?
115. During which week of pregnancy
are normal cardiovascular and A. Candida albicans
hematological changes early begin? B. Chlamydia trachomatis
C. Gardnerella vaginalis
A. 4 weeks D. Neisseria gonorrhoeae
B. 8 weeks E. Trichomonas vaginalis
C. 10 weeks
D. 12 weeks 120. Caput medusae are due to
E. 14 weeks engorgement of which of the following
vessels?
116. Which of the following eicosanoids
cause arteriolar vasospasm before A. Hepatic vein
menstruation? B. Splenic vein
C. Umbilical vein
A. Leukotriene LTB4 D. Umbilical artery
B. Thromboxane TXA2 E. Superficial epigastric artery
C. Prostacyclin PGI2
D. Prostaglandin E2 121. Regarding cardiac changes during
E. Prostaglandin F2α pregnancy. Which of the following ECG
findings during pregnancy should be
117. One month after cesarean section, considered normal?
a 25 year old woman palpates a small
nodule beneath the skin at the site of the A. Prolonged QT interval
healed incision. The skin was closed B. Absent QRS complex
using absorbable sutures. The nodule is C. T-wave flattening in lead II
excised, and polarizable refractile D. Q-wave in lead III and aVF
material is seen in the nodule. Which of E. Right axis deviation due to
the following cell types is most likely to elevated diaphragm
be most characteristic in this
inflammatory response? 122. A woman is 8 weeks pregnant
attends for genetic counseling. She is
A. Neutrophil known to be a carrier of cystic fibrosis
B. Macrophage and her sister has a child with cystic
C. Plasma cell fibrosis. Her husband is Caucasian and
D. Lymphocyte the carrier rate for cystic fibrosis in the
E. Langhans giant cell Caucasian descent is 1 in 25. What is the
risk of the child having cystic fibrosis?
118. Which two HPV subtypes are
responsible for around 90% of genital A. 1%
warts and are targeted in HPV B. 12.5%
vaccines? C. 25%
D. 75%
A. 6 and 11 E. 100%
B. 16 and 18
C. 31 and 35 123. A woman and her husband are
D. 31 and 45 known carriers of cystic fibrosis. What
E. 73 and 82 is the probability of them having an
affected child?
119. A 16-year-old girl has recently
become sexually active. She complains A. 1%
of intensely irritating greenish frothy B. 12.5%
vaginal discharge. The organism is seen C. 25%
under microscope in a drop of saline D. 50%
E. 75%

15 ‫صفحة‬ Dr. MOHAMED YAHIA 0900988745


MRCOG RECALLS

124. A 30-year-old woman primiparous following diseases is aggravated by the


at 34 weeks' gestation comes to the use of this drug?
prenatal facility concerned about
reddish purple striae she has A. Parkinsonism
increasingly developed on her breast B. Bronchial asthma
and abdomen. Which of the following C. Closed angle glaucoma
hormones is the most likely responsible D. Diabetes mellitus
for her symptoms? E. Diarrhea

A. Melanocyte-stimulating hormone 129. Which of the following drugs is


B. Estrogen most appropriate for treating
C. Progesterone uncomplicated Chlamydia?
D. Aldosterone
E. Cortisol A. Azithromycin
B. Clindamycin
125. The process of programmed gene C. Neomycin
directed cell death characterized by D. Penicillin
blebbing, cell shrinkage, nuclear E. Metronidazole
condensation and fragmentation is
known as 130. A 26-year-old woman has just
given birth and her newborn baby
A. Apoptosis undergoes assessment. His body is pink
B. Autophagy with blue extremities and has a pulse of
C. Pyknosis 105 bpm. He appears to be limp and
D. Chromatolysis taking slow, gasping breaths. He also
E. Necrosis grimaces in response to stimuli. What is
his APGAR score?
126. You see a 21-year-old woman in
clinic complaining of vaginal discharge A. 4
for the last 3 days. She has recently B. 5
become sexually active with a new C. 6
boyfriend. Analysis of the discharge D. 7
reveals gram-negative intracellular E. 8
diplococci with scant neutrophils.
Which organism is most likely to be 131. A 23-year-old primigravida is
responsible? booked with midwife at 21 weeks. She
reveals that she had female circumcision
A. Chlamydia trachomatis done when she was a child. On
B. Neisseria gonorrhoeae examination the clitoris is completely
C. Gardnerella vaginalis absent. You are to document the type of
D. Treponema pallidum female genital mutilation. What type of
E. Mycoplasma hominis female genital mutilation is this best
described as?
127. The most biologically active form
of vitamin D is A. Type 1
B. Type 2
A. Calcidiol C. Type 3
B. Ergosterol D. Type 4
C. 7-dehydrocholesterol E. Atypical presentation
D. 1, 25-Dihydroxycholecalciferol
E. 24, 25-Dihydroxycholecalciferol 132. The most common cause of delay in
puberty in males is
128. You review a 65-year-old woman
with known urinary urgency and A. Constitutional delay in growth
prescribe oxybutynin. Which of the B. Klinefelter's syndrome
C. Low BMI

16 ‫صفحة‬ Dr. MOHAMED YAHIA 0900988745


MRCOG RECALLS

D. Pituitary failure 138. Which is the commonest type of


E. Primary hypothyroidism malignant ovarian neoplasm?

133. What is the mechanism of A. Endometrioma


antibacterial action of penicillin? B. Serous cystadenoma
C. Clear cell carcinoma
A. Alteration of cell membrane D. Germinal cell carcinoma
permeability E. Mucinous cystadenoma
B. Inhibition of cell wall synthesis
C. Inhibition of arabinogalactan 139. At the end of surgery you notice an
synthesis area of burn near the attached
D. Inhibition of nucleic acid diathermy plate. The diathermy plate
synthesis seems to be partially detached. Which of
E. Inhibition of folate metabolism the following would be the most likely
aetiology?
134. The ratio between indirect to direct
maternal cause of death in UK is A. Due to the plate not being secured
properly, the current density was
A. 1:2 concentrated in the smaller area
B. 2:1 where the plate was attached
C. 1:3 B. High voltage was used
D. 3:1 C. The injury may have resulted from
E. 1:1 insulation failure
D. The power output was accidentally
135. What type of joint is formed at the turned up
symphysis pubis? E. The surgeon may have accidently
A. Fibrous touched the instrument to the
B. Primary cartilaginous patient’s thigh
C. Secondary cartilaginous
140. Which type of pituitary gland
D. Synarthrodial tumors increase growth hormone
E. Synovial secretion lead to acromegaly?
136. Which muscles form the pelvic A. Adenoma
diaphragm? B. Glioma
A. Levator ani and coccygeus C. Fibroma
B. Levator ani and piriformis D. Sarcoma
C. Obturator internus and coccygeus E. Schwannoma
D. Obturator internus and levator ani
141. The ovarian artery is a branch of
E. Obturator internus and piriformis
A. Abdominal aorta
137. Biological actions of progesterone B. Common iliac artery
on estrogen include which of the C. External iliac artery
following? D. Internal iliac artery
A. Estrogen precursor E. Uterine artery
B. Methylation of estrogen receptors
142. A 62-year-old postmenopausal
gene woman presents for her annual physical
C. Decreased estrogen receptors exam. On exam of you find a small pink
D. Increased sex hormone binding exophytic lesion outgrowth out of the
globulins (SHBG) urethra. Otherwise there is no
E. Stimulation of the estrogen remarkable finding. What is the most
positive feedback in the luteal
likely diagnosis?
phase
A. Urethrocele

17 ‫صفحة‬ Dr. MOHAMED YAHIA 0900988745


MRCOG RECALLS

B. Urethral caruncle C. Derivative of 17α-


C. Urethral diverticulum ethinyltestosterone
D. Condyloma acuminatum D. Synthetic prostaglandin analogue
E. Squamous cell carcinoma E. Synthetic steroid hormone

143. Which type of cell junctions helps 147. Which neurotransmitter is secreted
in transmission of small molecules or onto the adrenal medulla?
ions from one cell to another?
A. Acetylcholine
A. Adherens junctions B. Norepinephrine
B. Desmosomes C. Epinephrine
C. Diffusion spheres D. Dopamine
D. Tight junctions E. Serotonin
E. Gap junctions
148. Which of the following defines
perinatal mortality using the
International classification of diseases-
144. A 15-year-old girl attends the 10 (ICD-10)?
genitourinary medicine clinic
complaining of frothy yellow vaginal A. Number of stillbirths from 20
discharge. She is sexually active with weeks and 7 days neonatal deaths
her 17-year-old boyfriend and uses the B. Number of stillbirths from 22
oral contraceptive pill. Speculum weeks and 7 days neonatal deaths
examination reveals haemorrhages on C. Number of stillbirths from 24
her cervix. A urine pregnancy test is weeks and 7 days neonatal deaths
negative. Considering the most likely D. Number of stillbirths from 22
diagnosis, what is the most appropriate weeks and 28 days neonatal deaths
first line antibiotic? E. Number of stillbirths from 24
weeks and 28 days neonatal deaths
A. Amoxicillin
B. Azithromycin
C. Metronidazole
D. Entamizole 149. A 27-year-old woman has a smear
E. Spiramycin test as part of the UK screening
programme. Following an abnormal
145. Which of the following result she attends a colposcopy clinic.
investigations is used to determine On colposcopy, the whitened
hormone receptor expression 'receptor appearance of her cervix on application
status' of the breast cancer? of acetic acid is suggestive of a human
papilloma virus (HPV) infection. Which
A. Mammogram of the following HPV subtypes is high-
B. Magnetic resonance imaging risk for the development of cervical
C. Fine needle aspiration cytology intraepithelial neoplasia?
D. Immunohistochemical staining
E. Pestron emission computed A. HPV 1, 2
tomography B. HPV 6, 11
C. HPV 16, 18
146. A 24-year-old female presents with D. HPV 32, 34
severe pain during menses. To treat her E. HPV 73, 82
symptoms, you advise her to take
indomethacin in the hopes that it will 150. The non pregnant uterus of a 20-
reduce her pain. What type of drug is year-old female measured 7 x 4 x 3 cm.
indomethacin? She became pregnant, and just before
delivery of a term infant, the uterus
A. Mixed opioid agonist antagonist measured 34 x 18 x 12 cm. Which of the
B. Cyclo-oxygenase inhibitor following cellular processes was the

18 ‫صفحة‬ Dr. MOHAMED YAHIA 0900988745


MRCOG RECALLS

major reason for the increase in the size 154. Healthcare workers must have
of the uterus? appropriate safe disposal of sharps
waste. Sharps containers must be
A. Endometrial glandular hyperplasia located in a safe position to avoid
B. Myometrial fibroblast spillage. According to NICE guidance
proliferation where we have to put the sharps
C. Endometrial stromal hypertrophy containers in the hospital?
D. Myometrial smooth muscle
hypertrophy A. Out of reach of kids and visitors
E. Vascular endothelial hyperplasia B. Near to medication cupboard
C. Near to garbage
151. A 27-year-old woman is seen D. Under patient's beds
because it is exactly 8 weeks since her E. Outside the hospital wards
last menstrual period, which was on
March 1, 2016. Her menstrual cycles 155. A woman presents at 12 weeks of
had previously been regular with a 35- pregnancy. She has had two previous
day interval. What is this patient’s spontaneous abortions at the first
estimated date of delivery? trimester. She has had one live birth at
term with the birth of a live baby at 26
A. February 8 of the next year weeks who died within a few minutes.
B. February 11 of the next year Which one of the following correctly
C. December 8 of the same year expresses her gravidity and parity?
D. December 11 of the same year
E. December 15 of the same year A. Gravida 4 Para 1 + 2
B. Gravida 4 Para 2 + 2
152. A 47-year- old woman with C. Gravida 5 Para 1 + 2
choriocarcinoma is treated with very D. Gravida 5 Para 2 + 2
high doses of methotrexate. You E. Gravida 5 Para 1 + 3
anticipate significant host cell toxicity so
immediately after giving methotrexate 156. Glucose-6-phosphatase deficiency
you administer which of the following is a common metabolic disease in
agents? Africa. What is the function of the
enzyme glucose-6-phosphatase in
A. Erythropoietin carbohydrate metabolism?
B. MESNA (2-mercaptoethane
sulfonate) A. Adds glucose to glycogen ends
C. Acetylcysteine B. Adds phosphate to glucose
D. Foloinic acid C. Production of ATP in aerobic and
E. Folic acid anaerobic oxidation
D. Production of NADPH in red
153. A 27-year-old woman is 2 weeks' blood cells
postpartum with her first child. E. Converts glucose-6-phosphate to
Suddenly she becomes very tearful, glucose
irritable and sleepless. She has strange
thoughts of hurting herself and feels it 157. A newborn girl is found to have
difficult to breast-feed her baby. Her multiple midline malformations. Her
physical examination is within normal physician suspects a chromosomal
limits. Which of the following is the best disorder and orders a karyotype
term to describe her symptoms? (picture below). Which of the following
is the most likely diagnosis?
A. Maternity blues
B. Bipolar disorder
C. Schizophrenia
D. Postpartum depression
E. Postpartum psychosis

19 ‫صفحة‬ Dr. MOHAMED YAHIA 0900988745


MRCOG RECALLS

C. 2nd degree heart block


D. 3rd degree heart block
E. Wolff-parkinson-white syndrome

161. Forms the lateral wall of the ischio-


rectal fossa

A. Skin
B. Anal canal
A. Cri-du-chat syndrome C. Obturator internus fascia
B. Down syndrome D. Vagina
C. Turner syndrome E. Rectum
D. Edward syndrome 162. You are fast-bleeped to attend a
E. Patau syndrome major post-partum hemorrhage on
158. The null hypothesis states that a labour ward. The midwife estimates
given variable is without effect. What that the patient has lost approximately
type of error is present when the null 1000 ml of blood. The patient requires
many units of blood transfusion, after
hypothesis is incorrectly not rejected?
how many units of packed red cells
A. Type I error transfusion fresh frozen Plasma should
B. Type II error be given?
C. Type III error
A. Two units
D. Type IV error
E. Type V error B. Three units
C. Four units
159. In the development of the oocyte, D. Five units
when does the second meiotic is E. Six units
completed?
163. The following diagram is
A. At birth concerning about a 12-day-old
B. At fertilization blastocyst. Which of the following
C. At menarche regions is represented by the arrow?
D. At ovulation
E. Before fertilization

160. A 45-year-old female patient with a


history of ischaemic heart disease is
being reviewed in the pre-operative
clinic prior to undergo endometrial
ablation. You review her A. Allantois
Electrocardiogram (ECG) below: B. Yolk sac
Which of the following is the most likely C. Amniotic cavity
diagnosis? D. Chorionic cavity
E. Extraembryonic coelom

164. A 25-year-old woman who is 20


weeks pregnant was admitted via A&E
with severe loin pain, shivering and
vomiting. Her temperature is 38.8°C,
pulse is 110 bpm and blood pressure is
88/56 mmHg. Urine dip shows
A. Atrial flutter leucocytes ++ and protein ++. Her
B. 1st degree heart block midstream urine sample result shows:

20 ‫صفحة‬ Dr. MOHAMED YAHIA 0900988745


MRCOG RECALLS

 Culture: E.coli +++ 167. You are carrying out analysis of a


 Sensitivities: new test for DVTs and decide to plot a
 Trimethoprim: Resistance receiver operating characteristic curve.
 Amoxicillin: Resistance What are the axes of the curve?
 Nitrofurantoin: Sensitive
 Cephalexin: Sensitive A. Sensitivity versus Specificity
 Gentamicin: Sensitive B. Specificity versus Sensitivity
C. Positive Predictive Value versus
What is the drug of choice for the Negative Predictive Value
treatment? D. Sensitivity versus (1-Specificty)
E. (1-Sensitivity) versus Specificity
A. Trimethoprim oral
B. Nitrofurantoin oral 168. Which antibiotic is effective against
C. Cephalexin oral staph. aurus exotoxin?
D. Cephalexin injection
E. Gentamicin injection A. Vancomycin
B. Co-amoxiclave
165. Which system organs have the C. Metronidazole
highest risk of teratogenicity in the D. Methicillin
embryonic period? E. Clindamycin

A. Cardiovascular system 169. Consider the figure. Muscle lactate


B. Genitourinary system is transported in liver and converted to
C. Respiratory system glucose by the process of
D. Digestive system
E. Central nervous system

166. A 25-year-old woman presents with


right-sided pelvic pain for more than
one year. An ultrasound scan is
arranged which shows a 5 cm well-
defined cystic lesion in the right adnexa A. Rapport luebering cycle
with mixed echoes. There is no fluid in B. Glucose alanine cycle
the cul-de-sac with the following C. Cori's cycle
sonographic image: What is the most D. Citric acid cycle
likely diagnosis for the image of the E. Glycolysis
adnexa?
170. A 42-year-old woman presents in
her first pregnancy with monochorionic
twins at ten weeks gestation. Antenatal
screening for anemia in multiple
pregnancies should be undertaken

A. As per routine antenatal care


B. Once in every month
C. At booking and at 16 weeks
D. At booking, 28 and 36 weeks
E. At booking, 24 and 28 weeks

171. Which cell type, in the islets of


A. Benign cystic teratoma Langerhans, secretes glucagon?
B. Endodermal sinus tumour
C. Endometrioma A. Alpha cells
D. Mucinous cystadenoma B. Beta cells
E. Serous cystadenoma C. Delta cells
D. Epsilon cells

21 ‫صفحة‬ Dr. MOHAMED YAHIA 0900988745


MRCOG RECALLS

E. PP cells E. No relationship between soy milk


and improve menopause
172. Which of the following growth symptoms
factors is the most important for
formation of granulation tissues during 175. A 21-year-old woman presents with
wound healing? heavy menstrual bleeding for 8 months.
She has a regular 28 day cycle and her
A. Epidermal growth factor (EGF) LMP was 3 weeks ago. For the past few
B. Transforming growth factor α months she has been feeling generally
(TGF-α) tired and noticed some weight gain.
C. Platelet-derived growth factor Pelvic examination demonstrates an
(PDGF) enlarged uterus. What are the
D. Vascular endothelial growth factor recommended investigations you would
(VEGF) offer?
E. Transforming growth factor β
(TGF-β) A. FBC, pelvic ultrasound scan
B. FBC, thyroid profile, pelvic
173. The following diagram describes a ultrasound scan
structural chromosomal abnormality. C. FBC, pelvic ultrasound scan,
What does this structural chromosomal Endometrial sampling
abnormality represent? D. FBC, coagulation profile, thyroid
profile, pelvic ultrasound scan
E. FBC, coagulation profile, thyroid
profile, pelvic ultrasound scan,
Endometrial sampling

176. What pattern of inheritance does


this picture most likely illustrate?

A. Insertion
B. Inversion
C. Deletion
D. Duplication
E. Translocation

174. A study is conducted to investigate


whether there is a relationship between
soy milk and improve menopause A. X-Linked Recessive
symptoms. The Null hypothesis B. X-Linked Dominant
represents which of the following C. Autosomal dominant
statements? D. Mitochondrial
E. Autosomal Recessive
A. A relationship between soy milk
and improve menopause 177. During childbirth a bilateral
symptoms pudendal nerve block may be
B. A positive relationship between performed to provide anesthesia to the
soy milk and improve menopause majority of the perineum and the lower
symptoms one fourth of the vagina. To do this an
C. A negative relationship between anesthetic agent is injected near the
soy milk and improve menopause pudendal nerve. During the procedure
symptoms blood comes in on withdrawl the
D. A difference between soy milk plunger of the anesthetic syringe. Which
and improve menopause of the following vessels has been
symptoms entered?

A. Obturator

22 ‫صفحة‬ Dr. MOHAMED YAHIA 0900988745


MRCOG RECALLS

B. Femoral D. 9 weeks
C. Vaginal E. 11 weeks
D. External pudendal
E. Internal pudendal 182. What is the most common
congenital solid tumour of the
178. Whooping cough vaccine leads to newborn?
provide long immunity from
A. Astrocytoma
A. Memory B cells B. Willim's tumour
B. Cytotoxic T cells C. Neuroblastoma
C. Naïve B cells and TH cells D. Sacrococcygeal teratoma
D. Innate immune cells E. Hepatic hemangioma
E. Inoculation of microorganism
183. A 25-year-old woman patient
179. Carbon dioxide gas laser is better presents with number of small genital
preferred for the removal of blisters. A swab results reveals a
endometriotic implants as diagnosis of genital herpes. Which of the
following is the best drug to be used to
A. It does not capable to vaporize treat this patient?
tissue
B. It penetrates deeply into tissue A. Topical acyclovir
C. It has a minimal lateral thermal B. Topical gabapentin
tissue damage C. Topical tacrolimus
D. It is less transmitted in soft tissue D. Topical ultrapotent steroid
that contains water E. Steroid with antifungal
E. It produces an invisible beam and
does not affect the one who use it 184. A 24-year-old G2P0 woman is seen
in clinic at 12 weeks gestation. She is
180. The diagram below illustrates some known to have epilepsy with grand mal
of the steps (reactions) of seizures, for which she is currently on
steroidogenesis in their proper lamotrigine. Her last fit was one year
sequence. Which substance is indicated ago. Her full blood count result shows:
by letter 'X' in the above diagram?
 Haemoglobin 9.9 g/dl
 Hematocrit 35%
 Mean corpuscular volume (MCV)
110 fl
 Mean corpuscular hemoglobin
(MCH) 34 pg
 Platelets 198 x 10 9 /l
 Total white cell count 8.0 x 10 9 /l

What is the most likely interpretation of


A. Estrone this blood count?
B. Testosterone
C. Dihydrotestosterone A. Physiological hyderemia of
D. Androstenedione pregnancy
E. Dehydroepiandrosterone sulphate B. Iron deficiency anaemia
C. Folate deficiency
181. When using gestational age, when D. β-thalassemia trait
does crown rump length (CRL) get to 40 E. Hemolytic anemia
mm?
185. A 23-year-old woman who is six
A. 6 weeks weeks by dates is referred to EPU with
B. 7 weeks vaginal spotting, which continuous
C. 8 weeks today. Ultrasound shows a fetal pole

23 ‫صفحة‬ Dr. MOHAMED YAHIA 0900988745


MRCOG RECALLS

with a CRL of 4 mm. There is no fetal A. Make safe environment


heart action noted. The ovaries and B. Shake and shout
adnexa appear normal. What is the C. Call for help
most likely diagnosis? D. Left lateral tilt
E. Commence cardiac compressions
A. Heterotopic pregnancy
B. Viable intrauterine pregnancy 189. A woman presents in spontaneous
C. Could be a viable pregnancy labour at term with her first baby; then
D. Non viable pregnancy she delivered at term and the baby was
E. Threatened miscarriage fine. Six hours post delivery she finds to
have group B streptococcus on a high
186. The blood test of a 36-year-old vaginal swap done three days previously
fertility patient shows elevated when she was seen in another maternity
luteinizing hormone (LH) levels and unit. What is the most appropriate
high estradiol levels. What is the most management plane to prevent early-
likely cause of this? onset neonatal group B streptococcal
disease?
A. Hypogonadotrophic
hypogonadism A. Reassure that no action is
B. Ovarian failure necessary
C. Midcycle LH surge B. Immediately refer to the
D. Polycystic ovary syndrome neonatologist
E. Weight-related amenorrhoea C. Start antibiotics to the baby
D. Start antibiotics to the mother
187. A 30-year-old woman presents with E. Inform the mother to be given
a three month history of amenorrhea. In antibiotics in the next pregnancy
addition, her periods are irregular of
two years' duration. She is overweight 190. A 55 year-old woman has recurrent
and complains of increasing lethargy. urinary retention after a radical
Nothing abnormal is found on hysterectomy done for stage I
examination. Her hormone profile was a carcinoma of the cervix. Which of the
follows: following nerves should be preserved in
radical hysterectomy?
 βhCG < 5 IU/L
 LH = 4 IU/L A. Obturator
 FSH = 3.5 IU/L B. Pudendal
 Prolactin = 400 mIU/L C. Genitofemoral
 TSH = 7 mIU/L D. Superior hypogastric plexus
 Free T4 = 9 IU/L E. Inferior hypogastric plexus
What is the most likely diagnosis? 191. The following table shows the
following results:
A. Hyperthyroidism
B. Hypothyroidism Lab test +ve Lab
C. Normal pregnancy test ‐ve
D. kallmann syndrome
E. Polycystic ovary syndrome New test +ve 20 5
188. While you are attending to the New test‐ve 15 60
hospital you found a pregnant woman
unresponsive and collapsed in the car What is the negative predictive value of
parking lot. During your initial the new test?
evaluation you open her airway but she
is not breathing. Which of the following A. 25%
is the appropriate initial management? B. 57%
C. 75%
D. 80%

24 ‫صفحة‬ Dr. MOHAMED YAHIA 0900988745


MRCOG RECALLS

E. 92% A. Merkel cell carcinoma


B. Papillary hidradenoma
192. In a given sample for a group of C. Squamous cell carcinoma
size 16, the standard deviation of the D. Basal cell carcinoma
data is 12 and the mean is 2. What is the E. Adenocarcinoma
standard error of mean (SEM)?

A. 2
B. 3
C. 4
D. 5 196. A 33-year-old woman is admitted
E. 6 with surgical wound infection after
emergency caesarean section for
193. A publication reports the outcome suspected fetal compromise. Culture of
of a new therapy in a placebo controlled the wound demonstrates methicillin-
primary prevention of venous resistant Staphylococcus aureus
thromboembolism (VTE). 200 patients (MRSA). She has no known allergies.
were randomised to receive the new Which is the most appropriate
therapy and 200 allocated to placebo. treatment?
The study was completed over a 2 year
period. In the placebo group there were A. Cephalosporins
50 have venous thromboembolism and B. Clindamycine
in the group treated with the new C. Vancomycin
therapy there were 100 have venous D. Clindamycine and vancomycin
thromboembolism. What is the number E. Piperacillin/tazobactam
needed to treat to prevent one VTE over
the course of this study? 197. A primiparous woman at 39 weeks
of gestation is in labour at 8 cm dilation.
A. 2 The cardiotocography (CTG) has been
B. 4 classified as pathological due to a base
C. 10 rate of 105 bpm and persistent, long
D. 20 standing typical decelerations. Fetal
E. 40 blood sampling (FBS) has been
performed, showing a pH of 7.29. What
194. In an investigation to study the is the most appropriate management
effect of smoking in cancer cervix, it is option?
observed that 30 of the 60 patients were
smokers as compared to 20 out of 60 A. Needs caesarean section
control subjects. The odds ratio of immediately with general
cancer cervix associated with smoking anaesthetic due to fetal hypoxia
will be? B. Needs delivery within 30 minutes
due to fetal hypoxia
A. 0.33 C. Repeat FBS in no more than 1
B. 0.5 hour if CTG remains the same
C. 1.5 D. Repeat FBS in no more than 20
D. 2 minutes independent of CTG
E. 3 findings
E. Repeat FBS in no more than 30
195. A 25-year-old female presents with minutes is CTG remains the same
a sharply circumscribed nodule on the
interlabial fold. The nodule has a 198. A 37-year-old lady had a
tendency to ulcerate and bleed. This hysterectomy for multiple fibroids at
nodule can be completely cured by 15:00 am. She felt unwell and developed
surgical excision. Which of the following a spiking fever of over 38.2ºC on the
is most likely? first postoperative day at 22:00 am.

25 ‫صفحة‬ Dr. MOHAMED YAHIA 0900988745


MRCOG RECALLS

Which of the following is the most


likely?
200. 19. A 37-year-old multiparous
A. Deep venous thrombosis woman with gestational diabetes is in
B. Pulmonary embolism labour. On examination her cervix is 3
C. Post operative inflammation cm dilated with intact membranes. The
D. Surgical site infection labour is being continuously monitored
E. Reactive temperature spike using a cardiotocograph (CTG). She is
response to surgery caused by reviewed by the obstetric team and the
stress hormones and cytokines parameters of the CTG are noted.

199. A 25-year-old woman intending to  Baseline < 100 beats per minute
commence the combined contraceptive  Accelerations: Absent
pills wishes to discuss the risk of venous  Variability: < 5 beats per minute for
thromboembolism (VTE). She has no 50 minutes
other risk factors for VTE. Which type  Decelerations: late decelerations for
of COCP is associated with the highest 20 minutes
risk of VTE?  Contractions: three to four
contractions in 10 minutes
A. Loestrin 30 (ethinylestradiol /
norethisterone) What is the most appropriate course of
B. Microgynon 30 (ethinylestradiol / action?
levonorgestrel)
C. Microgynon 50 (ethinylestradiol / A. Continuous fetal monitoring and
levonorgestrel) reassess in 1 hour
D. Yasmin (ethinylestradiol / B. Turn to left lateral position with
drosperinone) oxygen therapy
E. Cilest (ethinylestradiol / C. Perform a Fetal blood sampling
norgestimate) D. Proceed to Category 1 C.S
E. Proceed to Category 2 C.S

26 ‫صفحة‬ Dr. MOHAMED YAHIA 0900988745


MRCOG RECALLS

EXAM 2

27 ‫صفحة‬ Dr. MOHAMED YAHIA 0900988745


MRCOG RECALLS

1.During Laparoscopy an injury to B. T12 and L1


vessel during right lateral port insertion C. L1
this vessel is a branch of which artery? D. L2
E. L1 and L2
A. External iliac artery
B. Internal iliac artery 7. Following radical hysterectomy and
C. Uterine artery pelvic lymphadenectomy for Stage I
D. Aorta carcinoma of the cervix, 63-year-old
E. Inferior epigastric artery woman reports weakness of hip
adduction. Which nerve most likely to
2.which structure Forms the lateral wall have been injured during the operation?
of the ischio-rectal fossa
A. Obturator nerve
A. Skin B. Femoral nerve
B. Anal canal C. Inferior gluteal nerve
C. Obturator internus fascia D. Superior gluteal nerve
D. Vagina E. Sciatic nerve
E. Rectum
8. Surgeon wants to take blood from
3. To perform an elective lower segment femoral artery. What is the relation of
caesarean section, the obstetrician nerve with femoral artery and vein
makes a transverse suprapubic incision. from lateral to medial?
Which of the following abdominal wall
layers will not be encountered A. VAN
transected during this incision? B. NAV
C. AVN
A. Anterior rectus sheath
B. Posterior rectus sheath 9. Dorsal artery of clitoris and deep
C. Rectus abdominis muscle artery of clitoris are a branch of?
D. Skin and subcutaneous tissue
E. Transversalis fascia and parietal A. External iliac artery
peritoneum B. Internal pudendal artery

4. At what level ovarian artery arise 10. What is the dermatome level at
suprapubic incision
A. L2
B. L1 A. T10
C. L4 B. L1
C. T12
5. Which of the following muscles leaves D. L2
the lesser pelvis through the greater
sciatic foramen? 11. What is the level of dermatome for
epidural at umblical region ?
A. pubococcygeus
B. Iliococcygeus A. L1
C. Puborectalis B. T12
D. Piriformis C. T10
E. Coccygeus D. L3
E. L4
6. To reduce postoperative pain after
caesarean section, an obstetrician 12. What is the nerve supply of external
performs ilioinginal-iliohypogastric anal sphincter ?
nerve block (IINB) on his 25-year-old
patient. What is the nerve root of A. Pelvic splanchnic nerve
B. Pudendal Nerve
ilioinguinal and iliohypogastric nerve?
C. Inferior hypogastric plexus
A. T12

28 ‫صفحة‬ Dr. MOHAMED YAHIA 0900988745


MRCOG RECALLS

13.Which structure in female is A. Anterior division of Internal iliac


homologous to male prostate? B. External iliac
C. Umbilical artery
A. Skene gland
B. Gartner duct 21 In Renal transplant kidney is
C. Bartholin’s gland anastomosed to

14. Pudendal cleft is defined as A. Vesical artery


B. Aorta
A. Area btw labia majora and thigh C. External iliac artery
B. Area btw labia majora and hymen D. Internal iliac artery
C. Area between labia majora E. Uterine artery
15. What is the anatomical location of 22. Symphysis diathesis in a patient
Bartholin’s gland? ..what is the type of joint which is
involves
A. Deep perineal pouch
B. Ischiorectal fossa A. Primary cartilaginous
C. Pudendal canal B. Secondary cartilaginous
D. Superficial perineal pouch C. Synovial
E. Urogenital diaphragm
23. Correct relation of round ligament
16. Pyramidalis is supplied by which with its embryological origin and
nerve? anatomy is:
A. Obturator n. A. Gubernaculum ovarii. In front of
B. Genitofemoral nerve. the uterine cornu to the labia
C. Subcostal nerve majora
D. Ilioinguinal B. Gubernaculum ovaria. In front of
E. Iliohypogastric the uterine cornu to the ovarian
ligament
17. Pain due to ovarion irritation can be C. Paramesonephric duct. In front of
felt through which nerve segment the uterine cornu to the labia
A. T10 majora
B. T12 D. Paramesonephric duct. Behind the
C. L1 uterine cornu to the labia majora
D. L2
24. Partial mole genotype
18. During Laparoscopy structure is A. 46XX
seen which is a remnant of Mesonephric B. 69 XXY
duct C. 47XX
A. Epoophoron D. 46XY
B. Gartner duct cyst 25. Layer surrounding the zona
C. Ovarian ligament pellucida?
19.During Sacrospinous ligament A. Mural granulosa cell
fixation which artery can be damaged B. Cumulus oophorus
A. Internal pudendal artery C. Basal lamina
B. Internal iliac D. Theca externa
C. External iliac E. Theca interna
D. Femoral artery 26.. What happened to decidua basalis
E. Inferior vesical artery
is in third trimester?
20. Superior vesicles artery is direct A. Remain decidua basalis
branch from

29 ‫صفحة‬ Dr. MOHAMED YAHIA 0900988745


MRCOG RECALLS

B. Become nutabach 34. Indifferent gonads arise from


C. Disappear
D. Merge with placenta A. Intermediate mesoderm
B. Paraxial mesoderm
27. The ureteric bud is derived from C. Endoderm
which of the following embryonic D. Ectoderm
structures?
35. What is the name of structure that
A. Urogenital sinus shunts oxygenated blood from the right
B. Pronephros to left atrium in the fetus?
C. Mesonephros
D. Mesonephric duct A. Ductus arteriosus
B. Ductus venosus
28. Lower part of vagina is derived C. Foramen ovale
from D. Ligamentum venosum
E. Pulmonary trunk
A. Ectoderm
B. Endoderm 36. Which of the following agents can
C. Urogenital sinus cause premature closure of the patent
D. Mesonephric duct ductus arteriosus?

29. Which hernia will be the result of A. Prostaglandin E1


small defect in lateral umblical region B. Nitric oxide
due to regression of umblical vein C. Cortisone
D. Acetaminophen
A. Congenital umbilical hernia E. Indomethacin
B. Omphalocele
C. Gastroschisis 37. What do the bottom and top
represent on the box-and-whisker plot?
30. The medial umbilical ligament is a
remnant of what structure? A. Mean and mode
B. Mean and median
A. Fetal paraumbilical veins C. Standard deviation and mean
B. Fetal umbilical arteries D. Least and greatest value
C. Lateralumbilical fold E. Lower and upper quartile
D. Urachus
38. You wish to investigate the time it
31. In the new non invasive prenatal takes to perform a caesarean section on
diagnostic test of cell-free fetal DNA women with and without pre-eclampsia.
what is the source of the cells in At the end of the study you have two
maternal blood ? groups of women with the duration of
caesarean sections recorded for each. It
A. Fetal WBCs
can't be assumed that these are
B. Fetal fibroblasts normally distributed. Which non-
C. Amniotic cells
parametric statistic test would you use?
D. Placental trophoblast
A. Chi-squared test
32. Embryological origin of GnRH B. Linear regression analysis
neurons C. Mann-Whitney U test
A. neural crest D. Pearson’s R test
B. Neural tube E. Student’s t-test
C. Olfactory bulb
39. .Data was collected from maternal
D. Hypothalamus records of the women having shoulder
33. Gastrulation is the process of dystocia at delivery and reviewed and
compared with normal deliveries along
a. Triple germ cell layer formation

30 ‫صفحة‬ Dr. MOHAMED YAHIA 0900988745


MRCOG RECALLS

with birth weight of babies.what is the D. Categorical


type of study E. Discrete

A. Retrospective cohort 43. How to calculate standard error of


B. Case Control the mean ?
C. Cross sectional
D. Randomised control Answer : SEM = SD/√n

40.. The number of hours worked per 44. A range of one SD above and one SD
week for a sample of students is shown below the mean includes CI of
below
A. 68.2%
Hours. students B. 95.4%
C. 99.7%
1 12
45. Which one of the following
2 9 statements best describes a type I
statistical (α error)?
3 29
A. Rejecting a null hypothesis when
4 28 there is a difference
B. Rejecting a null hypothesis when
5 15 there is no difference
6 10 C. Rejecting the alternative
hypothesis when there is a
7 18 difference
D. Accepting a null hypothesis when
8 15 there is a difference
E. Accepting the alternative
What is the mode of the above data? hypothesis when there is a
difference
A. 10
B. 4 46. 700 women were enrolled for the
C. 3 assesment of PPV for test preterm
D. 12 labour?320 women were test positive
E. 2 out of them 80 had preterm labor .380
were test negative and out of them 20
41. Calculate the odds ratio of
had preterm labor. what is the positive
asymmetrical IUGR and symmetrical
predictive value of the test?
IUGR ....etc etc
A. 25%
Asymmetrical.
B. 50%
Symmetrical
C. 70%
Apgar <7 2 10 D. 10%
E. 100%
2 40
47.. In which phase of the cell cycle do
A. 4 chromosome are align to the equatorial
B. 2.5 plate
C. 16
D. 1.5 A. Metaphase
B. Anaphase
42. .What type of variable is BMI ? C. Telophase
D. Interphase
A. Ordinal
B. Nominal 48. Which chromosomes are capable of
C. Continuous robertsonian translocation

31 ‫صفحة‬ Dr. MOHAMED YAHIA 0900988745


MRCOG RECALLS

A. 16, 17,18, 20 a. Thymine


B. 9, 10, 11, 12
C. 13, 14, 15, 21, 22 57. Which protein coiled in DNA ?

49. .In which stage DNA replicates. a. Histones

A. G1 58. What is the diagnosis


B. G0
C. G2
D. S phase

50. What is the primary function of


DNA polymerase?

A. Addition of nucleotide
B. DNA repair
60. Which hormone prevent regression
51. Which one of the following stages of of corpus luteum?
the cell cycle is cell become quiescent?
A. Hcg
A. Synthetic phase (S) B. Progesterone
B. Gap phase 1 (G1) C. Estrogen
C. Gap phase 0 (G0) D. HPL
D. Gap phase 2 (G2) E. FSH
E. Mitotic phase (M)
61. Estradiol 20% bound to?
52.. Mother has sickle cell disease
married to sickle cell trait husband A. shbg
chance of baby to having disease B. albumin

A. 25% 62..Patient with primary


B. 50% hyperparathyroidism what will be the
C. 75% correct value of calcium and phosphate
D. 100%
A. Inc Ca. Dec phosphate
53. Father with cystic fibrosis mother B. Dec Ca. Inc phosphate
normal Chance of baby to have cystic C. Inc Ca. Inc phosphate
fibrosis ? D. Dec Ca. Dec phosphate

A. 0% 63. ADH synthesized in


B. 25%
A. Para ventricular nucleus of
C. 50%
hypothalamus
54.. What role does messenger RNA B. Acidophillic cells of anterior
play in the synthesis of proteins? pituitary
C. Neuroendocrine cells of posterior
a. Translation pituitary

55. Detection of protein? 64. HPL similar to?

A. Southern plot A. FSH


B. Northen plot B. GH
C. Western plot
D. PCR 65. What is the second common cause of
congenital adrenal hyperplasia (CAH)?
56. Which nucleotide only present in
DNA A. 17-hydroxylase deficiency
B. 21-hydroxylase deficiency

32 ‫صفحة‬ Dr. MOHAMED YAHIA 0900988745


MRCOG RECALLS

C. 11-Beta hydroxylase deficiency C. 36 hr


D. 3 Beta- hydroxysteroid D. 48 hr
dehydrogenase deficiency E. 72 hr
E. 17 Beta-hydroxysteroid
dehydrogenase deficiency 74. Which hormone causes rupture of
graffian follicle and release of esrtogen
66. What kind of cells produces and progesterone?
parathyroid hormone (PTH)?
A) LH
A. Chief cells B) FSH
B. Parathynotic cells C) TSH
C. Oxyphilic cells D) GnRH
D. C cells E) Hcg
E. Follicular cells
75. which of the following order of
67. most common cause of hormones according to that increase the
hyperthyroidism in young woman potency of male from maximum to
minimum
A. Graves disease
B. Hashimoto’s thyroiditis A) DHT ...Testosterone.
C. Iodine deficiency ...Androstanedione A2 ...DHEA
B) DHT...A2...DHEA ...testosterone
68. Commonest cause of Cushing C) Testosterone. .DHT...A2...DHEA
disease
76. Androgen secreted through which
A) pituitary adenoma layer of adrenal ?
B) adrenal adenoma
A) Zona fasciculata
69. What percentage (T4) is free? B) Zona reticularis
C) Zona glomerulosa
A. 0.1%
B. 1% 77. The graph showing normal
C. 5% spirometry of a preoperative woman.
D. 15% Which lung volume is indicated by the
E. 70% number 6 on the spirometry tracing?
70. Which test you is used to check A. Residual volume
ovarian reserves in pre menopausal B. Expiratory reserve volume
women C. Inspiratory reserve volume
D. Tidal volume
A. Antimullerian hormone E. Vital capacity
B. FSH
C. LH 78. In ECG lead V1 and aVR represent
which portion of heart
71. which inhibit prolactin
A. Inferior wall
Dopamine B. Right atrium and right ventricle
72. Question about obesity amenorhea C. Right atrium and left ventricle
which hormone decrease apetite? D. Left atrium right ventricle.

Leptin 79. women 7 week not feeling well came


in ER with shortness of breath
73. After how many hours of LH surge
ovulation occur  PH 7.36
 PCO2 normal
A. 16 hr  PO2 12
B. 24 hr  Base excess 9.8

33 ‫صفحة‬ Dr. MOHAMED YAHIA 0900988745


MRCOG RECALLS

 HCO3 18 86. Iodine accumulation in thyroid cells


 Normal pregnancy values involves symport with

What is the diagnosis? A. Chloride


B. Potassium
A. Respiratory acidosis C. Hydrogen
B. Metabolic acidosis D. Sodium
C. Compensatory metabolic acidosis E. Calcium
80. Renin is secreted from 87. Mg excreted :
Juxtaglomerular cells A. Excreted Unchanged in urine
B. Metabolized in liver and excreted
81 . What is the most abundant thru kidney
carbohydrate in the breast milk? C. Conjugated in liver and excreted
A. Caesin through kidney.
B. Galactose
88. Vitamin C action
C. Glucose
D. Lactose Synthesize collagen
E. Sucrose
89. Which enzyme is present in
82. Acid base disturbance in Conns granulosa cells but not in theca cells?
syndrome
A. Aromatase
metabolic alkalosis B. 17 B hydroxylase
83.. GFR is maximum at which 90. Cloasma in pregnancy is due to
gestation
A. Cortisol
A. 32 wk B. MSH
B. 36 wk
C. 18 wk 91. A women had two children who had
D. 16wk received phototherapy after delivery she
has O - blood group got married 2nd
84. A 20 year old girl goes for walking time and currently pregnant. Husband
in a hill with friends and when she is on has A + CDe CDe what can be the blood
the extreme of the hill she becomes little group of the fetus
breathless and has paraesthesia in her
hands . What is the appropriate acid A- O+
base imbalance? A. A+
B. O-
A. Metabolic acidosis C. A+ O-
B. Metabolic alkalosis D. A+ O +
C. Mixed metabolic and
respiratory acidosis 92. Physiological variation of the
D. Respiratory alkalosis menstrual cycle between 21-35 days due
E. Respiratory acidosis to

85. Vaginal arousal secretion are A. Aberrant LH surge


released from B. Estrogen fails to produce
negative feedback on pituitary
A) Ectocervix. C. Estrogen fails to produce
B) Endocervix positive feedback on pituitary
C) Skene gland D. Luteal phase varies in duration
D) Bartholin’s gland by time taken for corpus luteum
E) Transudate from vagina to degenerate

34 ‫صفحة‬ Dr. MOHAMED YAHIA 0900988745


MRCOG RECALLS

E. Loss of progesterone secretion C. Smooth ER

93. What is the change in plasma 100. In which part of the cell does
volume during pregnancy? glycolysis take place?

B- Increase 20 to 30% A. Cytosol


C- Increase 30 to 50% B. Mitochondrial matrix
D- Increase 60 to 70% C. Mitochondrial cristae
E- Increase 50 to 70% D. Rough endoplasmic reticulum
E. Smooth endoplasmic reticulum
94. Transfer of free fatty acid thru
placenta

A- Active 1. Arrow was on columnar


B- Passive epithelium
C- Facilitated
D- Endocytosis
E- Pinocytosis

95. Glucose transported across the


placenta thru

A- Facilitated diffusion
B- Active transport 2.
C- Passive diffusion 1) columner
D- Endocytosis A. 2)squamous
2) transformation zone
96.. Which types of cells are phagocytic
for residual bodies left over from the 2. a graph showing x y z levels (almost
process of spermiogenesis? same in past papers) ..what is y that was
asked
A. Testicular mesenchymal cells
B. Testicular macrophages LH
C. Fibroblast cells 3.Picture showing cut section of
D. Interstitial cells of leydig umblical cord and asked about labelled
E. Sertoli cells thing
97. NO from which amino acid

A- L arginine
B- L aspartate
C- L lysine
D- L guanine

98. Gonadotropin Releasing Hormone


(GnRH) is what type of Biochemical
structure
umbilical vein
A. Tripeptide
B. Octapeptide 4. Indian Origin Lives In London Have
C. Nanopeptide History Of Low Grade Fever Weight
D. Decapeptide Loss Cough For 5 Months. She is
E. Polypeptide pregnant. CXR image given. What is
your diagnosis?
99. Cristae are present in
A. Tuberculosis
A. Mitochondria B. Bronchitis
B. Rough endoplasmic reticulum

35 ‫صفحة‬ Dr. MOHAMED YAHIA 0900988745


MRCOG RECALLS

C. Pneumonia (LMWH) be stopped before regional


analgesia?
5. Fetal skull picture and question was
asking about bone marked A. 6 hr
B. 12 hr
Parietal bone C. 24 hr
D. 48 hr
6. Brow presentation in labor what is E. 72 hr
the head diameter
14. Mirena secret in 24 hours
mentovertical
A. 20 micro gram
7. Growth chart showing growth above B. 35 micro gram
10 th centile C. 75 micro gram
Normal for gestational age 15. Women on anti emetics came in ER
8. Ct scan of a post hysterectomy with complain of involuntary muscle
patient who came in emergency as not movements Which drug she is using
feeling well shows Answer : Metoclopramide
a. Ureteric obstruction
16. Main action of cocps
b. Pelvic hematoma
A. Inhibit ovulation
9. Labetalol B. Inhibit fertilization
A. Non selective alpha and beta C. Thick cervical mucus
blocker 17. Drugs readily cross the placenta
B. Selective alpha and beta blocker
because of which quality
C. Non selective beta blocker
D. Selective beta blocker A. High lipid solubility
B. High protein binding
10. What is the objective of phase 3 C. ionization of molecules
clinical trial?
18. What is the term used for the
Check effectiveness of the drug
anticancer agent methotrexate?
11. Which of the following drugs is used A. plant alkaloid
to suppress lactation postpartum in B. Antimetabolite
mother who suffers from stillbirth? C. Alkylating agent
A. Quetiapine D. Chain cutting agent
B. Metoclopramide E. Intercalating agent
C. Loperamide 19.. What is the mechanism of action of
D. Leuprolide
hydralazine?
E. Cabergoline
A. α2 agonist
12. Drosperinone is B. Angiotensin-converting enzyme
A. Antiandrogenic inhibitor
B. Androgenic C. α blocker
C. Estrogenic D. β2 agonist
D. Progestogenic E. Direct-action on smooth-muscle
E. Antiminrelocorticoid 20. What is the mechanism of action of
13. When should the prophylactic dose trimethoprim?
of low molecular weight heparin A. Inhibit Nucleic acid synthesis
B. Antimetabolite

36 ‫صفحة‬ Dr. MOHAMED YAHIA 0900988745


MRCOG RECALLS

21. Patient on lithium me which organ A. Clindamycin


anomaly occur B. Ceftriaxone
C. Erythromycin
A. Heart D. Ciprofloxacin
B. Kidney
C. Brain 28. Most common side effect of
misoprostol
22. Which organ anomaly occur with
the use of ACE inhibitor A. A-vomiting
B. B-diarrhea
A. Heart C. C-nausea
B. Cranio facial D. D-constipation
C. Kidney
29. Mifipristone action??
23. 28 year old Women on
carbamazepine for epilepsy what is the Anti progesterone
suitable contraception
30. A 70-year-old lady had an
A. COCP ultrasound because she was feeling
B. High dose COCP bloated which showed bilateral ovarian
C. Mirena masses with no solid areas associated
D. POP with abdominal ascites. She is post
E. Implant menopausal and otherwise well. She was
sent for a CA 125 which comes back as
24. Patient on ulipristal acetate for the 50 U/mL. What is her risk of
treatment of fibroid which malignancy index score (RMI)?
contraceptive method you advice to her
A. 5
A. Barrier method B. 10
B. POP C. 150
C. COC D. 300
D. Mirena E. 450
25.A woman experiences a postpartum 31. Commonest cause of
hemorrhage after delivery and is postmenopausal bleeding?
administrated carboprost to promote
uterine contraction. What type of drug A- Endometrial hyperplasia
is carboprost? B- Endometrial cancer
C- Atrophic vaginitis
A. Oxytocin agonist D- Endometrial polyp
B. Progestogen
C. prostaglandin E1 32. On performing hysteroscopy on a
D. prostaglandin E2 postmenopausal women surgeon find
E. prostaglandin F2 resistence at internal Os what is the
ratio of uterine corpus till this point
26. Aspirin and indomethacin what is
the difference in action? A- 1:3
B- 3:1
A. Aspirin Irreversibly inhibit COX 1 C- 1:1
than ibuprofen D- 1:2
B. Aspirin reversibly inhibits COX 1 E- 2:1
than ibuprofen
33. Women presented in emergency
27. Patient allergic to penicillin which with acute abdomen which waked her
drug is adviced in labor as GBS up at night..she has a history of surgical
Prophylaxis? management of missed miscarriage one
week ago.hitopathology report showed

37 ‫صفحة‬ Dr. MOHAMED YAHIA 0900988745


MRCOG RECALLS

normal decidua no chorionic villi.what D) treat hyerpemesis repeat tests in a


is your further action week.

A. Admit for observation and 38. Which vitamin is deficient in


ultrasound in morning wernick s encephalopathy
B. Urgent CT scan
C. Call surgical team to check for Vitamin B1
bowel perforation
D. Call a consultant as you think she 39. Most common site of vulval cancer?
may have ruptured ectopic and A) Labia majora,
might need intervention B) Labia minora
C) Vestibule
34. CTG image of 45 minutes
D) vagina
A. Baseline normal
B. Acceleration present 40. Women died after cesarian she had
C. Variability normal shortness of breath postpartum
D. Some variable decelerations which haemorrhage. .was in DIC Postmortem
are v shaped and going down upto showed fetal hair and epithelial cells
20 seconds. Amniotic fliud embolism
E. Good uterine contractions
41. 42 year old with frequency urgency
G2P1+0 with previous normal delivery urge incontinence. No stress
with labor pains and membrane incontinence bladder diary showed low
ruptured showed thick meconium she is compliance bladder us normal ..no
5cm dilated. what will be your plan improvement in symptoms with bladder
A. Continue CTG and reassess in drill. What is the next plan
30min A) Antimuscarinic
B. Do fetal blood sampling B) Pelvic floor exercise
C. Cesarian section category 1 C) Post tibial nerve stimulation
D. Cesarean section category 2 D) Urodynamic studies
35. Which radiotherapy need vaginal E) Beta blocker
devise insertion 42. Women with PPROM..Which test is
Brachytherapy using in practice to diagnose preterm
labor
36. Postpartum haemorrage after
A) Fern test
vaginal delivery. pharmaceutically
failed to stop the bleeding. what is the B) Fibronectin test
first surgical step to stop the bleeding? C) Nitrazine test

A. Intrauterine balloon tamponade 43. A male came for infertility check up


B. Bilateral uterine artery ligation who was taking Finasteride .Finasteride
C. Bilateral int. iliac ligation exert its action by inhibition of which
D. Uterine b-lynch brace suture enzyme?
A. E Uterine artery embolization A. 21-hydroxlase
37. mother with twins diagnosed B. 5-alpha reductase
C. Aromatase
hyperthyroidism comes with vomiting
D. Phosphodiesterase
TSH 0.1, T4 29 .
E. Desmolase
A) start propiothiouracil
B) start radioactive iodine 44. 15 yrs old British Teenager Ask
C) bring preganncy to singleton. Like about having contraception . What will
seriously. the Doctor do in line with best practice?

38 ‫صفحة‬ Dr. MOHAMED YAHIA 0900988745


MRCOG RECALLS

A. Follow Fraser's Law A. Amplitude


B. Follow British guidelines B. Acoustic impedence
C. Follow non British guidelines C. Frequency
D. Resolution
45. A 34 week prenatal lady came in E. Wave length
emergency with the complain of heavy
painless vaginal bleeding. She is vitally 52. A 14-year-old girl is treated with
stable.what you will advice radiotherapy. Which of following tissues
is most radioresistant?
A. CBC
B. Perform vaginal examination A. Intestinal epithelium
C. Cesarian section B. Cerebral nuerons
C. Respiratory epithelium
46. Treatment of hypovolemia after D. Bone marrow
surgery? Patient vitally stable E. Skin
A. 500 ml crystaloid over 15 min 53. An Asian woman books in for her
B. 500ml haemaccel over 15 min third pregnancy at 12 weeks of
C. Group specific blood gestation. She has recently moved to the
D. Gelofusin infusion in 30 min UK from Thailand to be with her new
husband. After pre-test counselling,
47. Which one of the following imaging with the aid of an interpreter, she
techniques gives maximum radiation agrees to hepatitis B virus (HBV)
exposure to the patient during checking screening. The results return as follows.
tubal patency? What is the significance of these results?
A. Robin tube cannulation
HBsAg POSITIVE
B. Hysterosalpingography
C. hystero contrast salpingography Anti-HBc POSITIVE
D. Hysteroscopic tube visualization
E. Laparoscopy and dye intubation Anti-HBc IgM NEGATIVE

48. Which modality you will use to AntiHBs NEGATIVE


check growing thyroid nodule in
pregnancy A. Post vaccination
B. Resolved infection
A. Technetium scan C. Natural HBV immunity
B. Radio isotope D. Chronic infection
C. Ultrasound E. Acute HBV infection
D. CT
E. MRI 54. Women has PPROM at 29 weeks
clear liquor managed conservatively as
49. What is the SI Unit of magnetic field an outpatient coticosteroids given. she
came in labor at 32 weeks and delivered
A. Tesla normally...she also had pph after
B. Weber delivery...she was asses (on next day or
C. Gray after 2 days ) in ward round
50. What is the threshold for ultrasound  Pulse 120
frequency  RR 22
 Temp 38.2
20 KHz  BP 110/50
 Urine output 100ml in last 4 hours
 Labs
51. The difference between the peak and  normal Hb
the base line of ultrasound wave is  Tlc 18000
called?  Plt normal

39 ‫صفحة‬ Dr. MOHAMED YAHIA 0900988745


MRCOG RECALLS

 Lactate 3 A. A kallman
 CRP >250 B. B steroid use
 Rest I forgot C. C retrograde ejaculatio
D. D pituitary adenoma
What is your diagnosis E. E adrogen tumor
A. Hypovolemic shock
B. Sepsis
C. Internal bleeding 59. Peripartum cardiomyopathy
resembles to which type
55. A pregnant woman got screening
test for down's syndrome and the A. Dilated cardiomyopathy
following report is obtained B. Hypertrophic cardiomyopathy

 Age related risk; 1:1000MoM 60. SIADH is associated with which


 AFP; 1.05MoM cancer
 β-hCG; 1.7 mom
 NT : 0.85MoM Small cell lung cancer
 Test risk; 1:100000
61. 23-year-old woman presents with
What will you counsel this woman? fishy foul smelling vaginal odour. 'Clue
cells' are found in the smear. Which of
A. The fetus has down's syndrome the following cell types is belong to
B. The fetus does not have down's 'Clue cells'?
syndrome
C. It is high risk for down's A. Neutrophils
D. It is low risk for down's B. Natural killer cells
E. Ask the patient to go home C. Lymphocytes
D. Macrophages
56. Young girl secondary amenorrhea E. Vaginal Epithelial cells
for 5 months
62. Most common site of endometriosis?
 Fsh 4
 .Lh 3 A. Fallopian tube
 Prolactin normal B. Ovaries
 Tsh normal C. Uterosacral
D. Pouch of douglas
What is your Diagnosis? E. Peritoneum

A. Hypothalamic amenorrhea 63. An 18 year-old woman is found to


B. Premature ovarian failure have a largely solid ovarian neoplasm.
C. Hyperthyroidism Tumor markers were obtained and
demonstrated a markedly elevated LDH
57. According to who definition of value. Which of the following tumors is
perinatal mortality the most likely histology for this
neoplasm?
No. Of stillbirth after 22 week and early
neonatal deaths A. Theca cell tumour
B. Dysgerminoma
58. A 28 year old tall man normal C. Immature teratoma
normally virilaized and do regular daily D. Brenner cell tumour
exercise came for infertility check up E. Endodermal sinus tumour
 Fsh <0.1iu/L 64. Psammoma bodies are a histologic
 Testosterone 42 feature of ovarian papillary serous
 Prolactin 300 cystadenoma. ' Psammoma bodies' are
What is the diagnosis composed of

40 ‫صفحة‬ Dr. MOHAMED YAHIA 0900988745


MRCOG RECALLS

A. calcium examination normal.Her husband had


B. iron vasectomy since 2 years. Her family is
C. Fibrin complete. Which investigation will you
D. Mucin do before starting treatment
E. Lipid
a. Complete blood count
65. Two months postoperatively, a 55- b. Arrange for ultrasound
year-old woman who is being treated c. Perform an endometrial biopsy
with a diagnosis of small lung cancer
presents with excessive polyuria and 72. What is the single best description
depression and urine showed red cells for the histological appearance of the
+++ what is the electrolyte imbalance? secretory endometrium?

A. Hypernetremia A. Simple columnar


B. Hyponatremia B. Simple columnar with sub nuclear
C. Hyperkalemia vacuolation
D. Hypokalemia C. Stratified columnar
E. Hypercalcemia D. Stratified cuboidal
E. Simple cuboidal epithelium
66. Which cancer is most commonly
diagnosed in pregnancy and postpartum 73. Which pattern of endometrim do
simple endometrial hyperplasia most
A. Breast cancer likely resemble?
B. Cervical cancer
C. Melanoma A. Proliferative endometrim
B. Secretory endometrim
67. Kleihauer test is a test to quantify C. Endometritis
the amount of fetal blood in maternal D. Endometrial polyp
circulation, following fetomaternal E. Endometrial carcinoma
hemorrhage. What is the basis behind
Kleihauer test? 74. From which organ / system does the
Krukenberg tumor of the ovaries
HbF is resistant to denaturation by acids originate from?

Commonest type of vulval Cancer A. Lung


B. Brain
A. Melanoma C. Kidney
B. Squamous cell carcinoma D. Uterus
C. Adenocarcinoma E. GIT
69. In pre eclampsia what changes occur 75. Twin pregnancy itching on hands
in kidney and on abdomen especially at linea
nigra ...rash was not on any other place
A. Vascular enditheliosis
B. Acute renal failure  ALT normal
C. Glomerulonephritis  Bile salt 5 mmol
 Alk phosphatase mildly raised
70. Most common type of degeneration
in fibroid What is your diagnosis?
a. red degeneration A. A Obs cholestasis
b. Hyaline degeneration B. B Polymorphic eruption of
c. sarcomatous changes pregnancy
71. A 42-year-old perimenopausal 76. Most common cancer type in
woman complains of 12 months history fallopian tube
of Heavy vaginal bleeding. Pelvic

41 ‫صفحة‬ Dr. MOHAMED YAHIA 0900988745


MRCOG RECALLS

A. Adenocarcinoma 82. Question about cystoscopy which


B. Sq cell carcinoma showed papillary bladder cancer which
C. Sarcoma is the commonest type of cancer in
bladder
77. Patient presented in emergency with
complain of shortness of breath and Transitional
hemoptysis she has history of
miscarriage and histopathology showed 83. A women got infection in first
molar pregnancy .Chest x ray showed trimester rash and joint pain later on
above picture. which tumor marker is ultrasound showed only fetal ascites
raised in this condition with no other structural defect

Parvovirus b19

84 .A patient is diagnosed as having


vaginal infection Trichomonas
vaginalis. .what will be the treatment

A. 1gm metronidazole stat


B. Metronidazole 400 mg BD for 5
Bhcg days
C. 1 gm Azithromycin
78. Patient with complete molar
pregnancy with ultrasound showing 85. Which group of viruses causes
bilateral cyst with thin septa molluscum contagiosum?

A. Theca leuten cysts A. Adenovirus


B. PCOS B. Herpes virus
C. Corpus luteal cysts C. Papovavirus
D. Follicular cysts D. Parovirus
E. Pox virus
79. Ovarian solid mass on ultrasound
Benin histopathology with spindle 86. Bacteria causing sepsis in pregnancy
shaped cells chest X ray showing pleural
A. Group A streptococcus
effusion diagnosis?
B. Group Bstreptococcus
A. Fibroma
87. Bacteria causing toxic shock
B. Leiomyoma
syndrome?
C. Endomerioma
D. Dermoid A. Staph aureus
B. Group B streptococcus
(This is meig syndrome)
88. Condylomata lata seen in which
80. Most commonly seen female
stage of syphilis
urethral cancer?
A. Second stage syphilis
A. Transitional cell
B. Third stage syphilis
B. Squamous cell
89. Which HPV virus is high risk for
81. What is the most common congenital
cervical cancer
solid tumor of the newborn?
A. 33
A. astrocyte
B. 11
B. wilms tumor
C. 6
C. Sacrococcygeal teratoma
D. hepatic Hemangioma 90. Patient with IUCD came for a
routine check up and high vaginal sab

42 ‫صفحة‬ Dr. MOHAMED YAHIA 0900988745


MRCOG RECALLS

showed gonorrhoea .What will be your B) activates complement pathway


management? C) makes up 7p percent of pool
D) binds to fc from antibody
A. Only remove iucd complexes
B. Treat the patient with antibiotics
and remove iucd 95. Most important complement in
C. Treat with antibiotics and keep opsonisation
iucd in situ
A) C3a
91 . Patient presented with thin yellow B) C3b
fishy smelling discharge. Vaginal ph 5
What is the most likely causative 96.. Which of the following is the main
organism? immunoglobulin in mucosa

A. Trichomonas A. IgA
B. Gardenela vaginalis B. IgD
C. Chlamydia C. IgE
D. Gonorrhoea D. IgG
E. IgM
92. Patient came out to be chlamydia
positive .She is 12 weeks pregnant.what
is the management plan?
97. Which antigen-presenting cells
A. Treat the partner (APCs) are mainly found in the cervix?
B. Do not treat the patient till
delivery A. Plasma cells
C. Treat the patient and screen the B. Hofbauer cells
partner C. Langerhans cells
D. treat the patient and screen the D. B cells
partner E. T cells
E. Ask to use barrier method in 98 Rubella vaccine is which type of
pregnancy vaccine
93. Origin of macrophage Live attenuated vaccine
A- lymphocyte
B- neutrophil
A. C Monocytes 99. Thrombosis in protein C and S
deficiency is due to
94. Characteristics of igG
A) Decrease antithrombin III
A) crosses placenta B) Factor 5 and 8a

43 ‫صفحة‬ Dr. MOHAMED YAHIA 0900988745


MRCOG RECALLS

EXAM 3

44 ‫صفحة‬ Dr. MOHAMED YAHIA 0900988745


MRCOG RECALLS

1. A 16-year-old has recently become B. Muscarinic receptors agonist


sexually active. She complains of C. Selective Beta-1 agonist
intensely irritating greenish frothy D. Selective Beta-3 agonist
vaginal discharge. The organism is seen E. Selective Beta-3 antagonist
under microscope in a drop of saline
and PH is 6. What is the most likely 6. Which drug antagonises the action of
causative organism? oxytocin?

A. Candida albicans A. A: Atosiban


B. Chlamydia trachomatis B. Nifedipine
C. Gardnerella vaginalis C. Methyl-Dopa
D. Trichomonas vaginalis D. Terbutuline
E. Treponema palladium E. Sulbutamol

2. Which artery supplies the structures 7.Misoprostol is analogue of which of


derived from the midgut of the embryo? the following :

A. Coeliac trunk A. PGE1


B. Inferior mesenteric artery B. PGE2
C. Middle rectal artery C. PGF2
D. Renal arteries D. PGI2
E. Superior mesenteric artery E. Thromboxane A2

3. Regarding Influenza vaccine , what 8. Which of the following is a Side effect


type of vaccine? of Terbutaline :

A. Killed vaccine A. A . Maternal tachycardia


B. Toxoid vaccine B. B . Maternal bradycardia
C. Conjugate vaccine C. C . Fetal tachycardia
D. Live attenuated vaccine D. D . Fetal bradycardia
E. Organism Subunit vaccine E. E . IUGR

Answer : Organism Subunit vaccine 9. A 50 years old women with absent


periods for 2years, has bilateral ovarian
4. A 65-year-old nulliparous women mass with solid areas on scan and CA
presents with vaginal spotting. A 125 of 100. What is her RMI score?
hysterectomy is performed, and
pathologic examination of the removed A. A.900
uterus reveals heterologous components B. B.600
of a malignant tumor of the endometrial C. c.700
glands and metaplastic cartilage. Which D. d.450
of the following is the most likely E. e.250
diagnosis?
10. A 30-year-old woman with bipolar
A. Stromal sarcoma disorder is 12-week pregnant. She has
B. Carcinosarcoma been taken her lithium every night.
C. Adenosarcoma Which of the following abnormality
D. Endometrioma may occur if she continues to take
E. Adenocarcinoma lithium?

5. For overactive bladder a new drug A. Tetralogy of Fallot's


called Mirabegron is used. Which is the B. Dandy walker syndrome
single mechanism of action for this C. Ebstein's anomaly
drug? D. Limb reduction deformity
E. Transposition of great arteries
A. Selective serotonin re-uptake
inhibitor

45 ‫صفحة‬ Dr. MOHAMED YAHIA 0900988745


MRCOG RECALLS

11. In ECG which lead reflects the 17. Which class of immunoglobulin is
inferior wall of heart? primarily secreted in breast milk and
protects the infant’s intestinal mucosa
A. V1, V2 from infection?
B. V3, V4
C. aVR A. IgA
D. I, aVL B. IgD
E. II, III, aVF C. IgE
D. IgG
12. What is the embryological origin of E. IgM
Gartners duct cyst?
18. which Immunoglobin is a pentamer?
A. Cloaca
B. Genital tubercle A. IgG
C. Mesonephric duct B. IgM
D. Paramesonephic duct C. IgE
E. Urogenital septum D. IgA
E. IgD
13. Which of these chromosomes in
humans have centromere located at one 19.What day of a regular 25-day
end? menstrual cycle is a woman likely to
ovulate?
A. Telocentric chromosome
B. Metacentric chromosome A. 4
C. Dicenteric chromosome B. 11
D. Submetacentric chromosome C. 14
E. Acrocentric chromosome D. 22
E. 25
14. Which one of the following stages of
the cell cycle immediately preceeds 20.what is the SI unit for the activity of
Mitosis ? radioactive decay?

A. Synthetic phase (S) A. Gray


B. Gap phase 1 (G1) B. Sievert
C. Gap phase 0 (G0) C. Rad
D. Gap phase 2 (G2) D. Roentgen
E. Gap phase 3 (G3) E. Becquerel

15. Urea transfer from fetal circulation 21.What is the % of fibroid undergo
to maternal circulation? sarcomatous change?

A. Active transport A. 0.1%


B. Endocytosis B. 1%
C. Exocytosis C. 5%
D. Facilitated diffusion D. 10%
E. Passive diffusion E. 15%

16. What is the most likely histological 22. What is the contrast dye used in
subtype of ovarian cancer associated MRI scans?
with endometriosis?
A. Iodine
A. Clear cell carcinoma B. Barium
B. Serous adenocarcinoma C. Thallium
C. Borderline serous tumour D. Gadolinium
D. Muscinous cystadenocarcinoma E. Tecnisia
E. Granulosa cell tumour

46 ‫صفحة‬ Dr. MOHAMED YAHIA 0900988745


MRCOG RECALLS

23. A 7 week pregnant women with d. Ringer lactate


history of previous two miscarriage. e. Hartmans solution
Ultrasound show :
28. Mechanism of action of
ondansetrone?

a. H2 Antagonist
b. Seretonin HT3
c. H1 receptor
d. D2 Agonist
e. D2 Antagonist

29. A Doctor is making a discharge


summary for a women who underwent
what is the most likely karyotype?
a laparoscopic saplingo-oophorectomy
A. 46, XX for Ectopic pregnancy after 4days. You
B. 46, XY realise that her blood group is O-. What
C. 69, XXX is the best course of action?
D. 69, XXY
a. Anti D
E. 69, XYY
b. check partner blood group
24. which of the following is narrowest & Rh.
segment of fallopian tube? c. there is no need for anti D
as it passed 72 hour.
A. Ampulla d. High dose Anti-D
B. Isthmus e. Anti-D in next pregnancy
C. Infundibulum
D. Interstitial portion 30. A 19-year-old woman gives birth to
E. Fimbrial end her first child. She begins breast feeding
the infant. She continues breast feeding
25. A 32-year-old woman 7 weeks for almost a year with no difficulties
pregnant. Her Chlamydia swab is and no complications. Which of the
positive. Which is the single most following cellular processes that
appropriate treatment? occurred in the breast during
pregnancy allowed her to nurse the
A. Amoxicillin infant for this period of time?
B. Clindamycin
C. Doxycycline 100 BD A. Stromal hypertrophy
D. Erythromycin 500 QDS B. Epithelial dysplasia
E. Azithromycin C. Steatocyte atrophy
D. Ductal dilation
26. Which anti-emetic drug has E. Lobular hyperplasia
extrapyramidal side effect?
31.Which of the following hormones
A. Meclizine increases the excretion of calcium in
B. Scopolamine Kidney?
C. Ondansetron
D. Granisteron A. Insulin
E. Metoclopramide B. Cortisol
C. Calcitonin
27. Pregnant women with Nausea and D. parathyroid hormone
vomiting. Which fluid should not be E. Antidiuretic hormone
given?
32. When Choriocarcinoma
a. 0.9%NS metastasizes, it has a predominantly
b. 0.9% NS with KCL spread by which route?
c. 5% Dextrose

47 ‫صفحة‬ Dr. MOHAMED YAHIA 0900988745


MRCOG RECALLS

A. Direct invasion B. Narrow angle glaucoma


B. Hematogenous C. Diabetes mellitus
C. Lymphatic D. Parkinsonism
D. Surface implantation E. Peptic ulcer
E. Transcoelomic
38.A neoplasm of the female genital
33. What sexually transmitted disease tract occurring in an 18-year-old girl
(STD) is caused by Haemophilus whose mother was treated with
ducreyi? diethylstilbestrol during the pregnancy
is likely to be a:
A. Chancre
B. Chancroid A. Brenner cell tumor of ovary
C. Granuloma inguinale B. Teratoma of ovary
D. Condyloma acuminatum C. Sarcoma botryoides of vagina
E. Molluscum contagiosum D. Clear cell carcinoma of vagina
E. Squamous cell carcinoma of vulva
34.What drug can be given in
methotrexate toxicity? 39.When should the prophylactic dose
of low molecular weight heparin
A. Folic acid (LMWH) be given post-c-section?
B. Vitamin B12
C. Pyridoxine A. 6 hr
D. Thymidine B. 12 hr
E. Folonic acid C. 24 hr
D. 48 hr
35.A 40-year-old woman had bilateral E. there is no limit
silicone breast implants placed two
years ago. Since that time, she has noted 40. Active form of Vit-D is produced in
increased firmness with slight deformity which organ?
of the breast on the left. The implants
are removed, and there is evidence for A. Skin
leakage of the implant contents on the B. Liver
left. Which of the following cell types is C. Lungs
most likely to be most characteristic of D. Kidney
the inflammatory response in this E. Adrenal glands
situation?
41.What is the mode of inheritance of
A. Neutrophils Duchenne muscular dystrophy (DMD)?
B. Mast cells
C. Plasma cells A. X-linked dominant
D. Giant cells B. Autosomal recessive
E. T lymphocytes C. Autosomal dominant
D. Mitochondrial inheritance
36. Which is common malignant tumour E. X-linked recessive
in newborn?
42.What is the most common cause of
A. Retinoblastoma secondary hyperparathyroidism?
B. Teratoma
C. Fibroma A. Parathyroid adenoma
D. Neuroblastoma B. Multiple myeloma
E. Granulosa cell tumour C. Sarcoidosis
D. Chronic renal failure
37. Which of the following is E. Multiple endocrine neoplasia type
contraindication of the use of 1
oxybutynin?
43. Regarding the following
A. Bronchial asthma Karyotyping

48 ‫صفحة‬ Dr. MOHAMED YAHIA 0900988745


MRCOG RECALLS

A. Thymine
B. Guanine
C. Inosine
D. Uracil
E. Uranine

54. Lancefield grouping of streptococci


is based on the presence of ?
Which of the following is the most likely A. Carbohydrate antigen on cell wall
diagnosis? B. Peptidoglycan antigen on cell wall
C. M protein
A. Cri-du-chat syndrome D. Color of blood agar
B. Down syndrome E. Sugar fermentation
C. Turner syndrome
D. Edward syndrome 55. Antihypertensive which causes
E. Patau syndrome postnatal depression, when used in
permpartum period?
50.Considering the following pedigree ,
A. Methyl dopa
A. b. Labetalol
B. c. Nifedipine
C. d. Lisinopril
D. Hydralazine

56. Mechanism of action of Neostigmine


is
What is the mode of inheritance ?
A. Choline ester
A. Autosomal dominant B. Cholinomimetic
B. X-linked dominant C. Muscarinic antagonist
C. Y-linked dominant D. Anticholinesterase
D. Autosomal recessive E. Nicotinic antagonist
E. X-linked recessive
57. Coumarin drug antagonise which
51.Remifentanil is an ideal controlled vitamin?
analgesia during labour because
A. A.Vitamin A
A. It is a strong μ receptor antagonist B. Vitamin D
B. It has a long duration of action C. Vitamin K
C. It is rapidly metabolized by tissue D. Vitamin C
estrases E. Vitamin E
D. It is more effective than epidural
analgesia 58. Clomifene citrate mechanism of
E. It cannot cross the placenta action ?
52.The most common cause of delay in A. Androgenic steroid
puberty in males is: B. Estrogenic steroid
C. Progestogenic steroid
A. Constitutional delay in growth D. Gonadotropin analogue
B. Klinefelter's syndrome E. Selective estrogen receptor
C. Noonan's syndrome modulator
D. Primary hypothyroidism
E. Gonadal dysgenesis 59. Which of the following the only
structures passes through the lesser
53.Which base pairs with Adenine in the sciatic foramen?
standard DNA helix?

49 ‫صفحة‬ Dr. MOHAMED YAHIA 0900988745


MRCOG RECALLS

A. Iliococcygeus 64. Which process establishes the three


B. Piriformis definitive germ layers?
C. Sciatic nerve
D. Posterior femoral cutaneous nerve A. Neurulation
E. Tendon of obturator internus B. Gastrulation
muscle C. Craniocaudal folding
D. Lateral folding
60. Which of the following E. Angiogenesis
Antineoplastic drugs causing
hemorrhagic cystitis? 65. Which one of the following is the
termination of round ligament?
A. Cisplatin
B. Methotrexate A. Labia minora
C. Paclitaxel B. Labia majora
D. Cyclophosphamide C. Deep Inguinal ring
E. Vincristine D. Superficial Inguinal ring
E. Lateral vaginal wall
61. Luteinizing hormone (LH) is
structurally related to what other 66. Ductus venosus is completely
glycoprotein hormones? obliterated after birth to form what
adult structure?
A. ADH, oxytocin and FSH
B. FSH, TSH and growth hormone A. Ligamentum venosum
C. HCG, FSH and TSH B. Ligamentum arteriosum
D. HCG, ADH and FSH C. Ligamentum teres
E. TSH, growth hormone and HCG D. Medial umbilical ligament
E. Median umbilical ligament
62. Mechanism of action of ulipristal
acetate ? 67. What is the frequency used in trans-
abdominal Ultrasonography ?
A. Partial progesterone receptor
blockage A. 7-10 MHZ
B. Selective Estrogen receptor B. 10-15 MHZ
modulator C. 15-20 MHZ
C. Prostaglandin analogue. D. 2-6 MHZ
D. Selective progesterone receptor E. >20 MHZ
modulator
E. Oxytocin Antagonist 68. A 60-year-old with erythematous
erosive lichen planus on the vulva. What
63. A girl presented to your clinic type of cancer is linked to lichen
requesting emergency contraception. planus?
LMP 20 days ago. h/o unprotected
intercourse 7days ago for which she A. Squamous cell carcinoma
B. Basal cell carcinoma
took levonorgestrel. The couple had
C. Merkel cell carcinoma
regular intercourse since then and last
night the condom had breached. What D. Malignant melanoma
is the best course for action? E. Adenocarcinoma

A. Urlipristal acetate. 69. which of the following organisms


B. Copper T IUD Colonize copper IUD?
C. No need for additional A. Lactobacillus.
contraception as levonorgestrel B. Actinomyces species.
already inhibited ovulation this C. Mobilincus
cycle D. Ureaplasma
D. Levonorgestrel 1500 mu E. Staphyllococcus aurues
E. Levonorgestrel 3000 mu

50 ‫صفحة‬ Dr. MOHAMED YAHIA 0900988745


MRCOG RECALLS

70. Bacterial vaginosis is diagnosed with What is your Diagnosis ?


depletion in numbers of which
organism? A. Urinary retention
B. b. Intra-abdominal bleeding
A. Lactobacillus C. c. Urinary bladder injury
B. Mobiluncus D. d. Injury to ureter
C. Gardnella vaginalis E. Port Site infection
D. Trichomonas vaginalis
E. Trepenoma pallidum. 75.What is the most common inherited
bleeding disorder?
71. The following image from ICSI
procedure . A. Von Willebrand’s disease
B. Hemophilia A
C. Hemophilia B
D. Protein C deficiency
E. Thrombotic thrombocytopenic
purpura

76. The most common in Utero infection


causing fetal anemia in the UK is

A. Syphilis
What is the structure represented by
B. Toxoplasmosis
the arrows ?
C. Rubella
A. Oocyte. D. Parvovirus B19
B. First polar body. E. Cytomegalovirus
C. Second polar body.
D. Cumulus oophorus 77. Women with abnormal smear
referred to colposcopy. HSIL. Which
E. Zona pellucida
HPV is associated with this?
72. Side effect of Aminoglycosides is
A. HPV 18
mainly by affection of which of the
B. HPV 11
following nerves?
C. HPV 6
A. Optic nerve D. HPV 12
B. Trigeminal nerve E. HPV 20
C. Facial nerve
D. Vestibulocochlear nerve 78. Which HPV causes Genital Warts?
E. Glossopharyngeal nerve A. HPV 6
B. HPv 16
73. Regarding Breech presentation,
C. HPV 18
What is the Bi-trochanteric diameter?
D. HPV 31
A. 9 cm E. HPV 45
B. 10 cm
C. 11 cm 79. A 20 week GA women with h/o
D. 8 cm recent travel to India, presents with
fever, chills. Blood test falciparum
E. 13 cm
Vivax +VE. Best treatment?
74. 6 hours post laparoscopic tubal
A. Primaquine
ligation, patient c/o pain abdomen, not
B. Mefloquine
reliefed with analgesics. On examination
C. Choloroquine
a tender cystic mass felt under
D. Procholoquine
umbilicus with oozing pus from one of
E. Artisunate
the port sites. (LABS)

Hb: 11, WCC :17000.

51 ‫صفحة‬ Dr. MOHAMED YAHIA 0900988745


MRCOG RECALLS

80. A Women with 150cm height and 90 B. Linea alba


kgs weight, What is her category of C. Linea Nigra
BMI? D. Stria gravidarum
E. Polymorphic eruption of
A. Normal pregnancy
B. Obesity Class I
C. Obesity Class II 86. Mechanism of Action of
D. Obesity Class III Indomethacin?
E. Morbid obesity
A. cyclo-oxygenase inhibitor
81. The Lowest acceptable Hb in second B. Lipo-oxygenase inhibitor
trimester? C. Prostaglandin E1 agonist
D. Cyclo-oxygenase agonist
A. 9.5 g/dl E. Prostaglandin E2 agonist
B. 10 g/dl
C. 10.5 g/dl 87. The following diagram is concerning
D. 11 g/dl about a 12-day-old blastocyst.
E. 11.5 g/dl

82. Sheehan syndrome Mainly affects?

A. Hypothalamus
B. Anterior pituitary
C. Posterior pituitary
D. Thyroid
E. Adrenal cortex

83. A 16-year-old girl has recently


become sexually active. She complains
of intensely irritating greenish frothy Which of the following regions is
vaginal discharge. PH is 6. What is the represented by the arrows ?
most likely causative organism?
A. Allantois
A. Candida albicans B. Yolk sac
B. Chlamydia trachomatis C. Amniotic cavity
C. Gardnerella vaginalis D. Chorionic cavity
D. Neisseria gonorrhoeae E. Extraembryonic coelom
E. Trichomonas vaginalis
88. A 25-year-old woman presents with
84. A woman is 8 weeks pregnant right-sided pelvic pain for more than
attends for genetic counseling as she one year. An ultrasound scan is
worried about CF, her sister has a child arranged which shows a 5 cm well-
with cystic fibrosis. She is a carrier and defined cystic lesion in the right adnexa
her partner is Caucasian with a rate of with mixed echoes. There is no fluid in
1/25 What is the risk of the child having the cul-de-sac with the following
cystic fibrosis? sonographic image:
A. 1%
B. 12.5%
C. 25%
D. 75%
E. 100%

85. Pregnant women when exposed to


sun, get pigmentation on cheeks & face?

A. Melasma

52 ‫صفحة‬ Dr. MOHAMED YAHIA 0900988745


MRCOG RECALLS

91. Regarding Genital herpes infection ,


what is the best management?

A. Acyclovir
B. Gabapentin
C. Tacrolimus
D. Steroid
E. Antifungal

92. A 24-year-old G2P0 woman is seen


in clinic at 12 weeks gestation. She is
What is the most likely diagnosis for the known to have epilepsy with grand mal
image of the adnexa? seizures, for which she is currently on
lamotrigine. Her last fit was one year
A. Benign cystic teratoma ago. Her full blood count result shows:
B. Endodermal sinus tumour
C. Endometrioma  Haemoglobin 9.9 g/dl
D. Mucinous cystadenoma  Hematocrit 35%
E. Serous cystadenoma  Mean corpuscular volume (MCV)
110 fl
89. Regarding the following image what  Mean corpuscular hemoglobin
it is called ? (MCH) 34 pg
 Platelets 198 x 109/l
 Total white cell count 8.0 x 109/l

What is the most likely interpretation of


this blood count?

A. Physiological hyderemia of
pregnancy
B. Iron deficiency anaemia
A. Rapport luebering cycle C. Folate deficiency
B. Cori's cycle D. β-thalassemia trait
C. Citric acid cycle E. Hemolytic anemia
D. Glycolysis
E. kreb's cycle 93. A 23-year-old woman who is six
weeks by dates is referred to EPU with
90. Which substance is indicated by vaginal spotting, which continuous
letter 'X' in the Below diagram? today. Ultrasound shows irregular sac,
with yolk sac and no Fetal pole. There is
no fetal heart action noted. The ovaries
and adnexa appear normal. What is the
most likely diagnosis?

A. Heterotopic pregnancy
B. Viable intrauterine pregnancy
C. inevitable miscarriage
D. Missed miscarriage
E. Threatened miscarriage

94. A 33-year-old woman is admitted


A. C holesterol with surgical wound infection after
B. Testosterone emergency caesarean section for
C. Dihydrotestosterone suspected fetal compromise. Culture of
D. Androstenedione the wound demonstrates methicillin-
E. Aldosterone resistant Staphylococcus aureus

53 ‫صفحة‬ Dr. MOHAMED YAHIA 0900988745


MRCOG RECALLS

(MRSA). She has no known allergies. B. E.


Which is the most appropriate
treatment? 99. A 22 years old patient presents to
your clinic with on and off pain RIF.
A. Cephalosporins During Workup of the pain an
B. Clindamycin ultrasound Scan shows 4cm simple
C. Vancomycin ovarian cyst. What is your
D. Clindamycin and vancomycin Management?
E. Piperacillin/tazobactam
A. Reassure & discharge
95. What is Evidence based medicine? B. US guided aspiration
C. lap cystectomy.
a. Using current guidelines for D. CA 125.
treating individual patients. E. Yearly MRI scan follow-up
b. Using current clinical evidence to
treat individual patients. 100. What is the source for
c. Using most experience consultant premenopausal Oestrogen secretion?
advise to treat patients
d. Using current experimental A. Theca cells
evidence for treating patients B. Oocyte
e. Studying the most recent clinical C. Granulosa cells
evidence in treating patients D. Polar body
E. Adipose tissue

101. 41 years old women presents to


96. During laparoscopy you visualize clinic Complaining of galactorrhea.
medial umbilical folds. what structure it Labs show: FSH, LH and TSH raised.
contains?
A. Hypothyroidism
A. Umbilical vein B. Hyperthyroidism
B. Medial umbilical ligament C. Premature ovarian
C. Median umbilical ligament insufficiency
D. Inferior epigastric artery D. PCOS
E. Deep Circumflex artery E. Hypothalamic Dysfunction

97. Blood supply for ascending part of 102. Diazepam can cross placenta, due
colon ? to?

A. Superior mesenteric a. Lipid solubility


B. Inferior mesenteric b. Protein binding
C. Right anterior mesenteric c. Large molecular weight
D. Celiac trunk d. Facilities diffusion
E. Right colic artery e. Active transport

98. A study compares 2 types of suture 103. 80 years old women presents to
materials for skin closure for caesarean clinic with dysuria, on Examination you
section, subcuticular monofilament and notice a growth at urethral orifice
subcuticular polyfilament, who should exophytic with well circumscribed
close the skin to obtain best external edges?
validity?
a. Urethral caruncle
A. One consltant in each unit. b. Squamous cell carcinoma
B. Anyone who normally closes skin. c. Basal cell carcinoma
C. Anyone with 3 OSAT for d. Uretheral diverticulum
competence in abdominal closure e. Uretheritis
any specialist.
A. D.

54 ‫صفحة‬ Dr. MOHAMED YAHIA 0900988745


MRCOG RECALLS

104. which clotting factor does not rise E. Papillary serous


in pregnancy? cystadenocarinoma.

A. VIII & VIII 109. A 25 years old patient presents to


B. VIII & IX your clinic with history of Heavy
C. XI & XIII menstrual bleeding for 1year. She has a
D. XII & IX history of asthma. She plans to start
E. II & VII family in a year's time. This bleeding is
limiting her routine activity. What is the
105. You see a patient in antenatal clinic First line in Management of this
who is 14 week pregnant and is worried. patient?
Her 3 year old son has chicken pox. she
does not remember if she had chicken A. COCP
pox in past? her lab reports are as B. Norethisterone
follows: C. Mirena
D. Tranexmic acid
Varicella IgG negative E. Mefanimic acid
Varicella IgM negative 110. What is affected in Horner
syndrome?
A. she had past
infection/immunity to chicken A. Loss of Motor supply
pox B. Excessive Sympathetic
B. she is not immune to chicken stimulation
pox C. Excessive Parasympathetic
C. she had general disease in past stimulation
D. No need for vaccination D. Loss of Parasympathetic supply
E. Immunity to Rubella . E. Loss of Sympathetic supply
106. Which cells secrete histamine and 111. Regarding the following brachial
serotonin? plexus Diagram
A. Basophils
B. Neutrophils
C. Dendritic cells
D. Platelets
E. Macrophage

107. Post-delivery the registrar notes


that baby's HR 150bpm, pink body and
blue extremities, Grimace, some flexion
& friable cry. What is the apgar score?
What is the “ D “ structure represents ?
A. 3
B. 4 A. Axillary nerve
C. 6 B. Median nerve
D. 8 C. Musculo-cutaneous nerve
E. 10 D. Ulnar nerve
E. Radial nerve
108. Which Type of Ovarian cancer
occurs which BRCA 1 mutation ? 112. Regarding Varicella Zoster , What
its infectivity period?
A. Clear cell carcinoma
B. Endrometroid carcinoma A. 2 days before rash and 2 days after
C. Stromal Ovarian tumours crust disappears
D. Krukenberg tumour B. 2 days before rash until the crust
appear.

55 ‫صفحة‬ Dr. MOHAMED YAHIA 0900988745


MRCOG RECALLS

C. 10-14 day after appearance of C. External iliac artery


rash. D. Common iliac artery
D. 24 hour after rash appear E. Femoral artery
E. When lesion is Crusted.
119. Which cells produce mullerian-
113. Gestation age is which type of inhibiting substance in fetal testis?
data?
A. Interstitial cells of leydig
a. Nominal B. Interstitial macrophages
b. Ordinal C. Sertoli cells
c. Interval D. Theca cells
d. Regression E. Granulosa cells
e. Ratio
120.Which enzyme found in red blood
114. Which type of electrosurgical cells which buffers blood CO2?
circuits is safest in theatres?
A. Cytochrome-b5 reductase
A. Grounded B. 5-nucleotidase
B. Patient is Attached to pad C. Glucose-6-phosphate
C. Isolation dehydrogenase
D. On parallel D. Carbonic anhydrase
E. On series E. Pyruvate kinase

115. A 30 years old women who 121 . Which of the following essential
regularly takes insulin for diabetes. amino acids not found in OMEGA 3
Presents with lethargy and loss of and should be taken from dietry intake
weight , TSH is low & Total T3 high ? ?

A. Graves disease A. α-Linoliec acid


B. Hashimotos thoiditis B. Ecosapentoenoic acid
C. Toxic nodule C. Arginine
D. Ketoacidosis D. Arachidonic acid.
E. Hypothyroidism E. Histidine

116. What is the most common cause of 122 . Correct interpretation of the
cushing's syndrome? following Urogram :

A. Pituitary adenoma
B. Adrenal adenoma
C. Adrenal carcinoma
D. Adrenal hyperplasia
E. Iatrogenic steroid administration

117. Glucocorticoids are secreted from


which region of the adrenal gland?

A. Adrenal capsule
B. Zona reticularis A. Stress incontinence.
C. Zona glomerulosa B. Urge incontinence
D. Zona fasciculata C. Detrusor muscle instability
E. Adrenal medulla D. Detrusor muscle overactivity
E. Urinary tract infection .
118. The deep circumflex iliac artery is
a branch of which artery? 123. A town with a total number of live
birth is 250,000, and maternal death is
A. Internal iliac artery 750, number of stillbirth is 250, what is
B. Internal pudendal artery the Maternal mortality ratio?

56 ‫صفحة‬ Dr. MOHAMED YAHIA 0900988745


MRCOG RECALLS

a. 300 35-42w 10
b. 600
c. 900 1 st week after birth 50
d. 200
e. 750 A. 1/9000.
B. 180/24000.
124 . A women 40 week induced by C. 160/20000.
dinoprostone pessary 20 minutes later D. 210/20000.
and the pessary is removed . Regarding E. 240/20000.
the following CTG , what is the most
appropriate management ? 127. What is the anatomical space of
External uretheral sphincter?

A. Superficial perineal pouch.


B. Deep perineal pouch.
C. Ischiorectal fossa.
D. Superficial inguinal space.
E. Deep inguinal space

128. Pain due to ovarian irritation is felt


through which spinal segment ?

A. Give terbutuline 20 mg tocolytic. A. T10


B. Category 1 CS. B. T11
C. Category 2 CS. C. T12
D. Reassure and observe. D. L1
E. Perform fetal blood sampling. E. L2

125. Cervical cancer incidence in UK ? 129. What is the mechanism of action of


Trimethoprim ?
A. 1/250
B. 1/300 A. Inhibit bacterial cell wall
C. 1/1000 synthesis.
D. 1/3000 B. Inhibit 30S Ribosomal protein
E. 1/5000 synthesis
C. Inhibit Dihydrofolate reductase
126. In a town with 20,000 live birth, enzyme
calculate perinatal mortality ? D. Inhibit Nucleic acid synthesis.
E. Act on bacterial Cell membrane
Weeks prenatally
Postnatally 130. Post-operatively , the nurse noticed
an area of burn near attached
20w 20 diathermy plate , what is the mechanism
of injury ?
21w 10
A. Inadequate current used .
22w 40 B. Low frequency current focused on
23w 10 small area
C. High frequency current focused on
24w 20 small area
A. D.
25w 20 B. E.

26-30w 10 131. What is the measurement of


Anteroposterior diameter of pelvic inlet
31-35w 20 in Engagement ?

57 ‫صفحة‬ Dr. MOHAMED YAHIA 0900988745


MRCOG RECALLS

A. 10cm D. Likely biological false positive


B. 11cm E. Suggests a past treated infection
C. 12cm.
D. 13cm 134. What is the site of production of
E. 9cm. Thromboxane A2 ?

132. An Asian woman books in for her A. Platelets


third pregnancy at 12 weeks of B. Liver.
gestation. She has recently moved to the C. Macrophages.
UK from Thailand to be with her new D. Kidney.
husband. After pre-test counselling, E. Mast cells
with the aid of an interpreter, she
agrees tohepatitis B virus (HBV) 135. What is the site where Urea cycle
screening. The results return as follows. occurs ?
What is the significance of these results? A. Liver
B. Kidney
HBsAg POSITIVE
C. Adrenal gland.
Anti-HBc POSITIVE D. Brain.
E. Red blood cells
Anti-HBc IgM NEGATIVE
136. What makes the lateral border of
AntiHBs NEGATIVE ischiorectal fossa?

A. A.Post vaccination A. Levator ani.


B. B Resolved infection B. Anal canal.
C. C.Natural HBV immunity C. Coccygeus muscle.
D. D.Chronic infection D. Obtrator internus fascia.
E. E.Acute HBV infection E. Obtrator Externus fascia.

133. A 34 yr old woman has just 137. What is the Nerve that most
returned to the UK following commonly injured during Gynecological
missionary work in the northern part of laparotomy ?
South America. She is delighted to be
pregnant and presents for booking at 12 A. Obtrator nerve.
wks of gestation. After counseling she B. ilioinguinal nerve.
agrees to have routine blood C. iliohypogastric nerve.
investigations including screening for D. Femoral nerve.
syphilis. The results are returned as E. Lateral cutaneous nerve.
following:
138. The Umbilical artery is branch of ?
Venereal Disease Research Lab.
A. Anterior division of Internal iliac
(VDRL) test : Reactive
artery.
particle agglutination (TP-PA) : non B. Posterior division of Internal iliac
reactive artery.
C. Abdominal aorta.
Fluorescent Traponemal Ab absorption D. Internal pudendal artery.
(FTA-Abs) : non reactive E. External iliac artery

What is the most likely interpretation 139. How Diagonal conjugate diameter
for theses result? is measured ?

A. Consistent with traponemal A. From upper border of symphysis


infection at sometime pubis to sacral promontory.
B. Diagnostic of latent syphilis B. From Lower border of Symphsis
C. Indicative of early syphilis pubis to sacral promontory.

58 ‫صفحة‬ Dr. MOHAMED YAHIA 0900988745


MRCOG RECALLS

C. From the back of symphsis pubis C. Tyrosine kinase nuclear receptor


to tip of coccyx. D. Ligand-gated ion channel in the
D. From Sacroiliac joint to opposite cell membrane
iliopectineal line. E. G protein coupled receptor on the
E. It is the distance between ischial Golgi complex
tuboristies.
145. Regarding OOCYTE , when the
140. Preterm labour Normal labour second meiotic division is completed ?

+ve Fibronectin 5 20 A. At fertilization.


B. At Ovulation.
-ve Fibronectin 15 60 C. At birth.
D. At puberty.
What is the Negative predictive value ? E. first 2 days of menstrual cycle.
A. 70%
146. Regarding Wound healing process ,
B. 80%
what is the first stage ?
C. 85%
D. 90% A. Hemostasis.
E. 92% B. Inflammation.
C. Proliferation.
141. A women with inflammation in D. Remodeling.
ovary develop pain along medial side of E. Wound contraction.
thigh , what nerve most likely to be the
cause ? 147. Regarding inferior vesical artery ,
where does it come from ?
A. Obturator nerve.
B. Femoral nerve. A. Anterior division of Internal iliac
C. Lateral cutaneous nerve of thigh. artery.
D. Sciatic nerve. B. Posterior division of Internal iliac
E. Common peroneal nerve. artery.
C. Abdominal aorta.
142. What is the best investigation to D. Internal pudendal artery.
detect receptor status in breast cancer? E. External iliac artery
A. MRI scan. 148. A 67-year-old woman attends for a
B. Ultrasound preoperative assessment. She has been
C. Fine needle aspiration cytology
taking bendroflumethiazide for several
D. Immuno-histochemistry years for hypertension. Her ECG
E. CA 15-3 shows: What is the most likely
143. A Women after delivery found to electrolyte imbalance?
be in lithotomy position for 2 hours ,
now she develop drop foot , what nerve
most likely to be the cause ?

A. Tibial nerve.
B. Femoral nerve.
C. Sciatic nerve.
D. Common peroneal nerve. A. Hypercalcaemia
E. Lateral cutaneous nerve B. Hyperkalaemia
C. Hypernatraemia
144. The Progesterone receptor is D. Hypokalaemia
example of which type of receptor ? E. Hyponatraemia
A. Guanylate cyclase receptor 149. A 22-year-old primiparous woman
B. Nuclear transcription factor books her pregnancy at 11 weeks’

59 ‫صفحة‬ Dr. MOHAMED YAHIA 0900988745


MRCOG RECALLS

gestation. Her booking blood tests D. The maximal volume that can be
reveal a haemoglobin level of 10.1 g/dL. inhaled from the end-inspiratory
Electrophoresis reveals haemoglobin level
karyotype HbAS. What is the E. The maximum volume of air
diagnosis? inhaled from the point of
maximum expiration
A. Beta-thalassaemia major
B. Beta-thalassaemia trait 155. Regarding cell cycle , when does
C. Hereditary spherocytosis chromatids seperates to opposite poles ?
D. Sickle cell anaemia
E. Sickle cell trait A. Prophase.
B. Metaphase.
C. Anaphase.
D. Telophase.
150. Which biochemical technique is E. Interphase.
used to detect the presence and the
amount of a protein? 156. Which of the following methods
used for DNA Amplification ?
A. Northern blotting
B. Polymerase chain reaction A. Western blotting.
C. Southern blotting B. Northern blotting.
D. Western blotting C. Polymerase chain reaction.
E. X-ray crystalography D. Eastern blotting.
E. G-banding
151. A study design is made with a
Power sensitivity 90% and P = 0.05 . 157. What is the mechanism of
What is the chance of type 2 error ? Letrozole drug ?

A. 5% A. Aromatase inhibitor.
B. 10% B. 5 alpha reductase inhibitor.
C. 15% C. 5HT3 Antagonist.
D. 50% D. Selective estrogen reuptake
E. 90% inhibitor.
E. Selective progesterone reuptake
152. Which of the following drugs cause inhibitor
Kernictrus ?
158. A Couple with primary subfertility
A. Penicillin , who have been trying to conceive for
B. Gentamycin. over 12 months , attends a reproductive
C. Sulphonamides medicine clinic , the male partner has
D. Methotrexate. already give a semen sample for
E. Erythromycin analysis which return back showing : 32
million spermatozoa , morphology >5 ,
154. Which of the following statements vitality >5% , live spermatozoa 63% ,
best describe Tidal volume ? PH 7.2 , What is the correct
A. The normal volume of air interpretation of this result ?
displaced between normal A. Normal semen analysis
inhalation and exhalation(in and B. Azoospermia
out ) when extra effort is not C. Oligospermia
applied D. Asthenospermia
B. The volume in the lungs at E. PH indicate infection.
maximal inflation
C. The volume of air remaining in 159. A 28 years old women undergoing
the lungs after a maximal a D&C for a missed abortion , develop
exhalation

60 ‫صفحة‬ Dr. MOHAMED YAHIA 0900988745


MRCOG RECALLS

pain on cervical dilatation , what is the asymptomatic for UTI , but her urine
nerve supply of cervix ? analysis show heavy growth
(100000X/cm3) E-Coli , WBC 17000 ,
A. Inferior rectal nerve. She is allergic to penicillin whats your
B. Perineal nerve. management ?
C. Pelvic splanchnic nerve.
D. Superior rectal nerve. A. Reassure .
E. Obtruator nerve. B. Give her Amoxicillin.
C. Give her Gentamycin.
160. Regarding Insulin, it gets activated D. Give her Nitrofurantoin.
by removing ? E. No need for treatment as she is
asymptomatic.
A. A-peptide
B. B-peptide 166. What is the incidence of
C. C-peptide pregnancies complicating by cancer ?
D. D-peptide
E. E-peptide A. 1/300
B. 1/500
161. What represent the 2 axis of ROC C. 1/1000
curve ? D. 1/3000
E. 1/5000
A. Senstivity Vs 1/specifity
B. Specifity Vs 1/ Senstivity 167. Which of the following ovarian
C. Senstivity Vs specifity Cysts most prone to undergo Torsion ?
D. 1-Senstivity Vs 1-Specifity
E. 1/Senstivity Vs 1/Specifity A. Mucinous cyst
B. Serous cyst.
162. A condition where the uterus is C. Dermoid cyst.
bulky and endometrial glands are in D. Endodermal sinus tumour.
myometrium is called ? E. Hemorrhagic cyst
A. Leiomyoma. 168. A 28 years old women present with
B. Endometriosis. 8 weeks pregnancy , she develop heavy
C. Adenomyosis. bleeding ( abortion ) and the condition
D. Choriocarcinoma is uncomplicated , what the next step ?
E. Endometriod carcinoma
A. Expectant management.
163. From what structure does B. Dilatation and curettage.
ligamentum Arteriosus come from? C. Vacuum extraction.
D. Misoprostol + mifepristone
A. Ductus arteriosus E. Surgical Evacuation
B. 4 th pharyngeal arch
C. 6 th Pharyngeal arch 169. Which of the following enzymes
D. Sinus venousus used in Transcription ?
E. Dorsal arch
A. DNA Polymerase.
164. Conversion of Fe3+ to Fe2+ in gut B. RNA Polymerase.
require which of the following vitamins? C. Reverse transcriptase
D. Restriction endonuclease.
A. Vit A E. DNA Ligase
B. Vit B6
C. Vit B12 170. What is the Direct arterial blood
D. Vit C supply of decidua ?
E. Vit K
A. Uterine artery.
165. 88 years old women with vaginal B. Basal arteries
vault prolapse come for repair , she is C. Spiral arteries

61 ‫صفحة‬ Dr. MOHAMED YAHIA 0900988745


MRCOG RECALLS

D. Straight arteries. 176. A Mother delivered her baby 2


E. Arcuate arteries weeks ago , now she comes with
delusions and hallucinations , she feed
171. What is the component of Placental her baby normally with no other
barrier at term pregnancy? complains. Whats your diagnosis ?
A. Endothelium and cytotrophoblast. A. Postpartum Blues.
B. Syncitiotrophoblast & B. Postpartum psychosis
Cytotrophoblast. C. Panic attacks.
C. Endothelium and connective D. Postpartum depression
tissue. E. Manic attacks
D. Syncitiotrophoblast and
Endothelium 177. During a procedure of Pudendal
E. Syncitiotrophoblast and nerve block , a blood come back in the
connective tissue syringe , which artery most likely to be
injured ?
172. Regarding sex differentiation , at
what gestational age does the External A. Internal iliac artery.
genitalia of male and female is Identical B. Internal pudendal artery.
? C. Inferior rectal artery.
D. Perineal artery
A. 6 weeks. E. Inferior gluteal artery.
B. 12 weeks.
C. 20 weeks. 178. What hormone secreted by
D. 25 weeks. placenta leading to fetal diuresis ?
E. 30 weeks.
A. Relaxin.
173. Which of the following testicular B. Estriol
cells is responsible for androgen C. Oxytocin
production? D. Vasopressinase.
E. Inhibin
A. Sertoli cells.
B. Lydig cells 179. Which of the following statements
C. Seminal vesicles. describe Type 2 error ?
D. Theca cells.
E. Granulosa cells A. The erroneous acceptance of the
null hypothesis
174. Which of the following is male B. The erroneous rejection of the null
homologous of bartholin gland ? hypothesis
C. The inclusion of extreme outliers
A. Bulbouretheral gland ( Cowper’s in a data set
gland ) D. The occurrence of a γ error
B. Seminal vesicles. E. None of the above
C. Prostatic utricle
D. Prostatic gland. 180. Regarding the following data : The
E. Skene gland Mean BMI is 25 , and the Standard
deviation is 5 , what is the percentage of
175. Which of the following obese people ?
neurotransmitters secreted onto adrenal
medulla? A. 5
B. 10
A. Adrenaline C. 16
B. Nor epinephrine D. 34
C. Serotonin E. 36
D. Acetylcholine
E. Dopamine 181. What is the blood supply of
Foregut ?

62 ‫صفحة‬ Dr. MOHAMED YAHIA 0900988745


MRCOG RECALLS

A. Celiac trunk. E. Trimethoprim


B. Superior mesenteric artery
C. Inferior mesenteric artery 187. Regarding the following table,
D. Abdominal aorta calculate value of specifity ?
E. Internal iliac artery
Diseased Not
182. A Multiparous women with diseased
prolapse , which muscle is most likely to
be injured ? +ve test 85 15

A. Pubococcygeus. -ve test 40 60


B. Puborectalis. A. 20%
C. Coccygeus muscle B. 40%
D. Ischocavernosus. C. 60%
E. Bulbospongiosus. D. 80%
183. What is the site of production of E. 90%
Ammonia ? 188. What is the most common solid
A. Liver benign tumour of ovary ?
B. Kidney A. Serous tumours.
C. Lungs. B. Mucinous tumours.
D. Adrenals. C. Endodermal sinus tumours
E. Red blood cell D. Benign teratoma.
184. Regarding Standard error of mean E. Yolk sac tumour
[ SEM ] how it is calculated ? 189. A Patient come complaining of a
A. Square root SD / n groin pain , a ureteric stone is suspected
B. SD / n and Xray is requested , at what level
C. SD / Square root n you would expect to see the stone ?
D. SD x square root n A. iliac crest
E. SD – Square root n B. Paravertebral
C. Pelvic prim
185. A 55 year old presents to clinic due
to vulval itch and discolouration. D. Mid-clavicular line.
examination reveals pale white E. Mid-axillary line
discoloured areas to the vulva. A biopsy 190. Which adverse reaction is common
shows epidermal atrophy with and usually dose related in patients
subepidermal hyalinization and deeper taking misoprostol?
inflammatory infiltrate. What is this
characteristic of? A. Diarrhea
B. Nausea
A. Lichen Simplex Chronicus C. Vomiting
B. Vulval intraepithelial neoplasia D. Bloating
C. Vitiligo E. Dyspepsia
D. Extramammary Pagets
E. Lichen Sclerosus 191. The ureteric buds is derived from
which of the following structures ?
186. A Pregnant lady 11 week ,
presented by UTI , what the best course A. Metanephros
of action ? B. Mesonephros.
C. Mesonephric duct
A. Penicillin. D. Paramesonephric duct
B. Gentamycin E. Allantois.
C. Doxycyline
D. Nitrofurnatoin.

63 ‫صفحة‬ Dr. MOHAMED YAHIA 0900988745


MRCOG RECALLS

C. Phosphate.
D. Chloride
192. A Patient come with an ischemic E. Sodium
stroke , regarding brain ischemia and
necrosis , what type of necrosis occur in 197. Which Immunoglobulin is a Dimer
brain ? ?

A. Colliquative necrosis. A. IgA


B. Coagulative necrosis. B. IgD
C. Fat necrosis. C. IgE
D. Fibrinoid necrosis. D. IgG
E. Dry gangrene E. IgM

193. What is the Action of mRNA ?

A. Translation 198. Which nerve emerges on anterior


B. Transcription border of psoas major muscle ?
C. Transfer
D. DNA synthesis A. Obturator nerve.
E. Reverse transcription B. Femoral nerve.
C. Lateral cutaneous nerve.
194. From Which structure inhibin is D. Genitofemoral nerve.
produced ? E. iliohypogastric nerve.

A. Theca cells. 199. LMP : 1 st of march in a regular


B. Granulosa cells. 35-day cycle, What is [ EDD] ?
C. Cumulus oophorus
D. Corpus leutem A. 8 DEC.
E. Lydig cells B. 15 DEC.
C. 8 NOV
195. Which of the following cells is D. 15 NOV.
considered the basis for the adaptive E. 22 DEC
immune system?
200. Which of the following is
A. Neutrophils responsible for the striae during
B. Monocytes pregnancy ?
C. Lymphocytes
D. Basophils A. Melatonin.
E. Eosinophils B. Cortisol.
C. Estrogen
196. Main hydrogen ion buffer found in D. Melanocyte stimulating hormone.
urine ? E. Aldosterone

A. Bicarbonate.
B. Magnisium

64 ‫صفحة‬ Dr. MOHAMED YAHIA 0900988745


MRCOG RECALLS

EXAM 4

65 ‫صفحة‬ Dr. MOHAMED YAHIA 0900988745


MRCOG RECALLS

1.Which of the following depression 7.Atosiban is ...


screening questionnaire are frequently
used during pregnancy? a) Progesterone agonist
b) Progesterone antagonist
A. Edinburgh Postnatal Depression c) Use as tocolytic
Scale d) Oxitocin agonist
B. General Health Questionnaire e) Oxytocin antagonist
C. Hamilton Depression Rating Scale
D. Beck Depression Inventory 8.Which adverse reaction is common
E. Hospital Anxiety and Depression and usually dose related in patients
Scale taking misoprostol?

2.Pain due to ovarian irritation is felt a. Diarrhea


through which spinal segment ? b. Nausea
c. Vomiting
A. T10 d. Bloating
B. T11 e. Dyspepsia
C. T12
D. L1 9.Terbutaline has the following
E. L2 mechanism of action

3.What makes the lateral border of A. Alpha agonist


ischiorectal fossa? B. Alpha receptor blocker
C. Beta 1 agonist
A. Levator ani. D. Beta 2 agonist
B. Anal canal. E. Dopaminergic
C. Coccygeus muscle.
D. Obtrator internus fascia. 10.71 y/o with unilocular cyst measuring
E. Obtrator Externus fascia. 30 mm and a Ca 125 of 25 iu/ml. What
is her RMI?
4.Which artery supplies the structures
derived from the midgut of the embryo? a. 0
b. 25
A. Coeliac trunk c. 75
B. Inferior mesenteric artery d. 225
C. Middle rectal artery e. 250
D. Renal arteries
E. Superior mesenteric artery 11.30-year-old woman with bipolar
disorder is 12-week pregnant. She has
5.In which phase of oocyte development been taken her lithium every night.
does zona pellucida apear around ovum Which of the following abnormality
? may occur if she continues to take
lithium?
A) primordial follicle
B) primary oocyte A. Tetralogy of Fallot's
C) secondary oocyte B. Dandy walker syndrome
D) Dominant follicle C. Ebstein Anomaly
E) Tertiary follicle/graffian follicle D. Limb reduction deformity
E. Transposition of great arteries
6.The HPV vaccine is what type of
vaccine? 12.The ureteric buds is derived from
which of the following structures ?
a. Live attenuated viruses
b. Inactivated viruses A. Metanephros
c. Toxoid based vaccine B. Mesonephros.
d. Polysaccaride based vaccine C. Mesonephric duct
e. Recombinant/ Subunit Vaccine D. Paramesonephric duct

66 ‫صفحة‬ Dr. MOHAMED YAHIA 0900988745


MRCOG RECALLS

E. Allantois. a. metabolized in the liver


b. Excreted through the kidneys
13.Allantois embryonic origin c. Excreted Unchanged in urine
d. b. Metabolized in liver and
a. Endoderm excreted through kidney
b. Paraxial mesoderm e. Conjugated in liver and excreted
c. Internediate mesoderm through kidney
d. Lateral plate mesoderm
e. Ectoderm 20.Which is common malignant tumour
in newborn?
14.Which one the following stage of the
cell cycle immediately preceeds mitosis A. Retinoblastoma
B. Teratoma
a. S phase C. Fibroma
b. G1 phase D. Nephroblastoma
c. G2 phase E. Sacrococcygeal Teratoma
d. G0phase
21.Vitamin A has 2 diet sources. Plant
15.Amino acids transfer through source is carotene and the animal
placenta is by which mechanism.?? source is...
a. A.simple diffusion a. cryptoxantene
b. B.facilitated diffusion. b. gamma carotene
c. C.active transport c. isotretinoin
d. D.passive transport d. retinal
e. E.endocytosis e. retinyl palmitate
16.Which of the following antibodies is 22.Young female 25 years old patient 7
found as a dimer? weeks pregnant with complaint of
A. IgA tiredness
B. IgM
TSH 8.5
C. IgG
D. IgD T4 14
E. IgE
T3 5.4
17.With regard to MRI scanning what is
the SI unit measure of strength of a. Euthyroid
magnetic field? b. Primary Hypothyroidism
c. Secondary Hypothyroidism
a. Weber d. Subclinical Hypothyroidism
b. Tesla e. Hyperthyroidism
c. Coulomb
d. Ampere 23.Identify the karyotype
e. Gauss

18.The most likely karyotype for a


patient with complete hydatidiform
mole is

A. 46, XX
B. 46, XY a. Down's syndrome
C. 69, XXX b. Edwards
D. 69, XXY c. Patau
E. 69, XYY d. Klinefelter's Syndrome
e. Turner's
19.Magnesium excretion

67 ‫صفحة‬ Dr. MOHAMED YAHIA 0900988745


MRCOG RECALLS

24.Case of a male with azoospermia and b. Cranio facial


loss of smell(anosmia) diagnosis? c. Kidney
d. Limbs
a. Kallman's Syndrome e. Teeth
b. Klinefelter Syndrome
a. c Kartagener Syndrome 30.Which organ anomaly occur with the
c. Prader Willi Syndrome use of Tetracycline
d. Turner Syndrome
a. Heart
25.Edwards syndrome manifestation in b. Cranio facial
a male patient c. Kidney
d. Limbs
a) 46 XX e. Teeth
b) 46 XXY
c) 47 XX 31.CNS is the most affected during
d) 47 XY teratogenesis. What is the first sensation
e) 45 XO developed in the fetus?

26.Nitrogenous bases make up cellular a. hearing


nucleic acids including DNA and b. touch
RNA. Which of these nitrogen bases is c. smell
found only in DNA but not in RNA? d. taste
e. eyesight
A. Uracil
B. Cytosine 32.What is the mechanism of action of
C. Thymine Clindamycin?
D. Guanine
E. Adenine a. It inhibits bacterial protein
synthesis by binding to the 50s
27.A 30 year old women comes to see subunit of the ribosome.
you and advises she has felt a little b. inhibits cell wall formation
unwell with diarrhoea and flu like c. inhibits DNA gyrase
symptoms. She is 28 weeks pregnant. A d. interfere with DNA synthesis
blood culture confirms listeria infection. e. dihydrofolate reductase
What is the appropriate treatment (she
has no known dug allergies)? 33.What is the mechanism of action of
Dibagatran?
a. Amoxicillin
b. Cefalexin a. Thrombin inhibitor
c. Gentamicin b. Thrombin activator
d. Erythromycin c. Antithrombin inhibitor
e. Clindamycin d. Antithrombin activator
e. Vitamin K inhibitor
28.The antibacterial action of Penicillin
is due to its effect on? 34.Warfarin acts where in the clotting
cascade
a. Cell membrane
permeabilty a. factor V precursor
b. Cell wall synthesis b. factor VII precursor
c. Protein synthesis c. fibrinogen precursor
d. DNA synthesis d. prothrombin precursor
e. RNA synthesis
35.Pyramidalis is supplied by which
29.Which organ anomaly occur with the nerve?
use of ACE inhibitor?
a. Obturator n.
a. Heart b. Genitofemoral nerve.

68 ‫صفحة‬ Dr. MOHAMED YAHIA 0900988745


MRCOG RECALLS

c. Subcostal nerve d) D.descending aorta


d. Ilioinguinal
e. Iliohypogastric 42.Which of the following is
predominant type of bacteria in the
36.The pathway of piriformis muscle?? vagina during pregnancy?

A. Lumbar surface of sacrum,greater A. A-lactobacillus.


sciatic notch,greater trochanter B. B-peptostreptococcus.
B. Pelvic surface of sacrum,greater C. C-listeria monocytogenous.
sciatic notch,greater trochanter D. D-streptococcus agalactia.
C. Lumbar surface of sacrum,lesser
sciatic notch,greater trochanter What is the structure pointed by the
D. Pelvic surface of sacrum,lesser arrow
sciatic notch,greater trochanter

37.What is the term used for the


anticancer agent methotrexate?

A. plant alkaloid
B. Antimetabolite
C. Alkylating agent
D. Chain cutting agent A. A.Trophoblast
E. Intercalating agent B. B.Amniocytes
C. C.Polar body from zygote
38.Which hormone has a structure D. D.Inner cell mass
similar to Prolactin? E. E.Blastocyst cells
a) Anti-müllerian hormone 43.The following diagram describes a
b) Cortisol structural chromosomal abnormality.
c) Follicle-stimulating hormone What is the name of this structural
d) Insulin abnormality?(picture similar to this)
e) Growth Hormone

39.Gastrulation is process of :

a) Development of three germ layer


b) Fusion of sperm and ovum
c) Type of mitotic division
d) Formation of GIT A. Deletion
e) Formation of neural tubes B. Insertion
C. Inversion
40.Which drug trial phase primarily D. Robertsonian translocation
assesses the safety of a drug? E. Reciprocal translocation
A. Phase IV 44.Identify the bone of the fetal skull
B. Phase III
C. Phase II
D. Phase I
E. Phase 0

41.In fetal circulation which structure


caryy oxygenated blood from right
atrium to left atrium?

a) A.Ductus venosus
b) B.foramen ovale b. frontal bone
c) C.ductus arteriosus c. parietal bone

69 ‫صفحة‬ Dr. MOHAMED YAHIA 0900988745


MRCOG RECALLS

d. occipital bone
e. temporal bone

45.Coronal suture is located in?

A. between frontal and parietal


B. between posterior part of
parietal and occipital
C. between parietal and temporal
D. between two parietal bones
E. between temporal and occipital

46.Identify the structure pointed by the a. Renal stones


arrow b. Ischemia secondary to Uterine artery
obstruction
c. UTI
d. Urethral obstruction
e. Ureteric Obstruction

49.Indian Origin Lives In London Have


a. allantoic duct History Of Low Grade Fever Weight
b. umbilical artery Loss and Cough For 5 Months.She is
c. umbilical vein pregnant. CXR image below. What is
d. warthon's jelly your diagnosis?
e. amniotic epithelium

47.What is the area pointed by the thin


arrows

a. Tuberculosis
b. Bronchitis
c. Pneumonia
a. squamous keratinizing epithelium d. Sarcoidosis
b. squamous non keratinizing e. Silicosis
c. columnar epithelium
d. transformation zone 50.Patient post abortion came in with
e. cuboidal epithelium difficulty of breathing, and hemoptysis.
Chest xray done (like the one below).
48.A 33 year old women with known With the following findings what is the
stage III cervical cancer post tumor marker that is possibly elevated?
hysterectomy patient presents to A&E
with lower abdominal and unilateral a. AFP
flank pain. CT scan contrast urogram b. CEA
was done. From the following list what c. HCG
is the likely diagnosis?(similar picture d. Ca 125
was given in exam where there is e. LDH
interrupted flow of the dye)

70 ‫صفحة‬ Dr. MOHAMED YAHIA 0900988745


MRCOG RECALLS

51..A pregnant woman desires screening 56.Which enzyme is present in


test for down's syndrome and the granulosa cells but not in theca cells?
following report is obtained :
A. 11 b hydroxylase
 Age related risk; 1:1000MoM B. Glucoronidase
 AFP; 1.05MoM C. 17 b hydroxylase
 β-hCG; 0.85 D. Acid Phosphatase
E. Aromatase
 Test risk; 1:10000
57.Where in body is androgen
What will you counsel this woman?
produced?
A. The fetus has down's syndrome
A. Distal tubules
B. The fetus does not have down's
B. Lung
syndrome
C. Zona fasciculata
C. It is high risk for down's down's
D. Zona reticularis
syndrome
E. Zona glomerulosa
D. It is low risk for down's syndrome
E. Ask the patient to go home 58.A woman is 8 weeks pregnant
attends for genetic counseling as she
52.LDH elevates in which type of
worried about CF, her sister has a child
ovarian ca ?
with cystic fibrosis. She is a carrier and
a. Dysgerminoma her partner is normal What is the risk
b. Endodermal sinus tumor of the child having cystic fibrosis?
c. Choriocarcinoma
A. 0%
d. Yolk Sac Tumor
B. 25%
e. Endometriod Tumor
C. 50%
53.Infection with which one of the D. 75%
following human papillomaviruses E. 100%
(HPVs) is associated with an increased
59..Mother has sickle cell disease
risk of cervical cancer?
married to sickle cell trait husband
A. HPV 6 chance of baby to having disease?
B. HPV 11
a. 0%
C. HPV 17
b. 25%
D. HPV 30
c. 50%
E. HPV 33
d. 75%
54.Which group of viruses cause e. 100%
molluscum contagiosum?
60..Aspirin and ibuprofen what is the
A. Adenovirus difference in action?
B. Herpes virus
a. Aspirin Irreversibly inhibit COX 1
C. Papovavirus
than ibuprofen
D. Parovirus
b. Aspirin Irreversibly inhibit COX 2
E. Pox virus
than ibuprofen
55.HPV high risk for cervical cancer c. Aspirin reversibly inhibits COX 1
include than ibuprofen
d. Aspirin reversibly inhibits COX 2
a) 6 than ibuprofen
b) 11
c) 16 61..Patient menopause came in with
d) 30 profuse vaginal bleeding 1 week ago.
e) 31 U/S done with the following findings

71 ‫صفحة‬ Dr. MOHAMED YAHIA 0900988745


MRCOG RECALLS

point A to point B = 1.2mm. What is Mesentery| Blood Supply


your management? | Lymphatic

a. ask her to come back if bleeding ----———————————————


recurs
b. give her COCP a. No Superior Mesenteric a. pre
c. give her POPs aortic LN
d. for diagnostic curettage b. No Inferior mesenteric a. pre
e. hysterectomy aortic LN
c. Yes Inf Mesenteric a. pre aortic
62..26yr woman on COCP with ethyl LN
estradiol 30mcg came in with frequent d. Yes Superior Mesenteric a. pre
light bleeding episodes after 3 weeks of aortic LN
use with negative chlamydia test and e. Yes Inferior Mesenteric a.
negative cx smear. What is your inguinal LN
management?
66..Appendiceal artery is a branch of
a. No need to do anything follow up
after 3 months if bleeding persist a. Ileocolic artery
b. Change to ocp with higher b. inferior mesenteric a.
eostrogen content 35ug EE c. left colic a.
c. Change to ocp with lower d. middle colic a.
eostrogen content 20 ug EE e. right colic a.
d. Repeat cervical smear
e. Endometrial biopsy 67.What is the major estrogen found in
menopause?
63.A 25-year-old woman presented to
a. a...Estradiol
an early pregnancy unit with mild
b. b...Estriol
vaginal bleeding after 5 week's
c. c...Estrone
amenorrhea. There is history of
irregular menses. Ultrasound shows no 68.Drugs readily cross the placenta
evidence of intrauterine pregnancy. because of which quality?
Beta hCG is 400 IU/l and beta hCG
after 48hrs later is 700 IU/l. What is the A. High lipid solubility
most likely diagnosis? B. High protein binding
C. ionization of molecules
A. Pregnancy of uncertain viability
B. Early normal pregnancy 69..HIV positive patient came with
C. Ectopic pregnancy ruptured membranes 38 weeks. Already
D. Incomplete miscarriage on HAART. Viral copies 1450.
E. Pregnancy of unknown location Management?
64.Grade A recommendation of RCOG a. Category 1 CS
means b. Category 2 CS with IV
Zidovudine
a. at least one randomized controlled c. Immediate Cesarean Delivery
trial
b. meta analysis from several 70..A 34-year-old woman has just
randomized controlled trials returned to the UK following
c. at least one well designed study missionary work in the northern parts
not randomized of South America. She is delighted to be
d. well controlled studies but non pregnant and presents for booking at 12
randomized weeks of gestation. After counselling she
e. expert opinion agrees to have routine blood
investigations including screening for
65..Sigmoid Colon

72 ‫صفحة‬ Dr. MOHAMED YAHIA 0900988745


MRCOG RECALLS

syphilis. The results are returned as a. Adenocarcinoma


follows: b. Transitional cell carcinoma
c. Paraganglioma
 Venereal Disease Research d. Squamous cell carcinoma
Laboratory (VDRL) e. Adenocarcinoma
test REACTIVE
 Treponema pallidum particle 76..Most common cancer type in
agglutination (TP-PA) test NON- fallopian tube?
REACTIVE
 Fluorescent treponemal antibody a. Adenocarcinoma
absorption (FTA-Abs) test NON- b. Squamous cell carcinoma
REACTIVE c. Sarcoma
d. Carcinosarcoma
What is the most likely appropriate e. Clear Cell Carcinoma
interpretation of these results?
77..What type of variable is BMI ?
A. Consistent with treponemal
infection at sometime a. Ordinal
B. Diagnostic of latent syphilis b. Nominal
C. Indicative of early primary c. Continuous
syphilis d. Categorical
D. Likely biological false positive e. Discrete
E. Suggests a past treated infection
78..Half life of Mifepristone?
72..What is the most common cause of
a. 5-10 hrs
the female urethral caruncle?
b. 25-30 hrs
A. Trauma c. 45-50 hrs
B. Tumour d. 75-80 hrs
C. Infection e. 85-90 hrs
D. Hypoestrogenism
E. Congenital 79.In pregnancy, what is the total
percentage increase in plasma volume
73..Macrophages are derived from what by term compared to a non-pregnant
type of cell? woman?

a. Natural Killer Cells a. 10 to 30%


b. Lymphocytes b. 30 to 50%
c. Basophils c. 50 to 70%
d. Monocytes d. 70 to 90%
e. Neutrophils e. No increase

74..From which organ / system does the 80.Motor fibers to detrusor muscle of
Krukenberg tumor of the ovaries the urinary bladder are derived from
originate from? which of the following nerves?

a. Lung A. Pelvic splanchnic


b. Brain B. Greater splanchnic
c. Kidney C. T11-L2
d. Uterus D. Superior hypogastric plexus
e. GIT E. Sacral plexus

75.A 57-year-old woman presents with 81.Deep perineal and dorsal clitoral
hematuria and dysuria based on the nerve is a branch of ?
clinical finding what is your possible
a. external pudendal
diagnosis?
b. internal pudendal

73 ‫صفحة‬ Dr. MOHAMED YAHIA 0900988745


MRCOG RECALLS

c. ilioinguinal a. cytoplasm
d. iliohypogastric b. mitochondria
c. plasma membrane
82.During laparoscopy, the remnant of d. rough ER
theumbilical artery could be identified e. smooth ER
sometimes the it is patent. In the adult
the patent segment supplies which 88..Which of the following is precursor
vessel? of Serotonin? .

a. internal iliac A) Histidine.


b. inferior vesical B) Glycine.
c. superior vesical C) Arginine.
d. external iliac D) Tyrosine.
e. internal pudendal E) Tryptophan.

83.Which test you is used to check 89..What is the percentage of uterine


ovarian reserves in pre menopausal rupture during IUCD insertion?
women
A. a.1/25
a. Estradiol B. b.1/50
b. Anti mullerian hormone C. c.1/100
c. FSH D. d.1/500
d. LH E. e.1/5000
e. Progesterone
90..Which cancer most commonly
84..Which is the most common cause of diagnosed in pregnancy and
the premature ovarian failure? postpartum?

A. A)idiopathic a. Breast CA
B. B)autoimmune b. Cervical CA
C. C)chromosomal c. Melanoma
D. D)infection d. Colon CA
E. E)iatrogenic
91..What is the most common cause of
85..A male came for infertility check up postmenopausal bleeding?
who was taking Finasteride for
treatment of his male pattern baldness. A. Atrophic vaginitis and
Finasteride exert its action by inhibition endometritis
of which enzyme? B. Endometrial cancer
C. Endometrial hyperplasia
A. 21-hydroxlase D. Endometrial polyp
B. 5-alpha reductase E. Hormone replacement therapy
C. Aromatase
D. Phosphodiesterase 92..An Asian woman books in for her
E. Desmolase third pregnancy at 12 weeks of
gestation. She has recently moved to the
86..The enzyme involve in the final UK from Thailand to be with her new
oxidation of uric acid? husband. After pre –test counselling,
with the aid of an interpreter, she
a. decarboxylase agrees to hepatitis B virus (HBV)
b. carbonic anhydrase screening. The results return as follows.
c. dihydropurine What is the significance of these results?
d. xanthine oxidase
 HBsAg POSITIVE
87..Most enzymes that take part in  Anti-HBc POSITIVE
Kreb’s cycle are located in?  Anti-HBcIgM NEGATIVE
 AntiHBs NEGATIVE

74 ‫صفحة‬ Dr. MOHAMED YAHIA 0900988745


MRCOG RECALLS

A) A.Post vaccination b. 40/50=80%


B) B Resolved infection
C) C.Natural HBV immunity 98.Which of the following ovarian Cysts
D) D.Chronic infection most prone to undergo Torsion during
E) E.Acute HBV infection pregnancy?

93.What is the function of DNA A. Mucinous cyst


polymerase? B. Serous cyst.
C. Dermoid cyst.
A. To unwind DNA helix D. Endodermal sinus tumour.
B. To add nucleotides to the end of a E. Hemorrhagic cyst
template DNA strand
C. To synthesis the nucleotides of 99..Patient came in with abdominal pain
DNA strands with history of heavy and painful
D. To seal together the broken ends periods. Exlap was done which revealed
of DNA strands the following: What is the likely
E. DNA repair diagnosis?

94..What is represented by the equation A. Functional cyst


Sensitivity/(1-specificity)? B. Physiologic cyst
C. Ovarian Endometrioma
A. Accuracy D. Ovarian Teratoma
B. Positive predictive value E. Germ cell tumor
C. Negative predictive value
D. Positive likelihood ratio 100.Which one of the following best
E. Negative likelihood ratio describes the histological type of
endometrium that is with enlarged and
95..What is the correct formula to tortous glands?
calculate specificity?
A. Decidual reaction
a. TP/(TP+FP) B. Atrophic endometrium
b. TN/(TN+FN) C. Proliferative endometrium
c. TN/(TN+FP) D. Secretory endometrium
d. TP/(TP+FN) E. Arias-Stella phenomenon

96..700 women were enrolled for the 101..Telogen Effluvium (hair loss) is
assesment of PPV for test preterm more pronounced in what stage of
labour?320 women were test positive pregnancy?
out of them 80 had preterm labor .380
were test negative and out of them 20 a. first trimester
had preterm labor.what is the positive b. second trimester
predictive value of the test? c. third trimester
d. during delivery
A. 25% e. after delivery
B. 50%
C. 70% 102..Natural Killer Cells(NK) is
D. 10% increased significantly at which stage of
E. 100% menstrual cycle?

97..Out of the women enrolled in a a. early proliferative


study 50 had PTL and 40 of those who b. late proliferative
had preterm labor were tested positive c. early secretory
for fetal fibronectin and 10 who had d. late secretory
preterm labor were tested negative what e. at ovulation
is the sensitivity of the test?
103..Sympathetic supply of bladder
a. 10/50=20% largely comes from which nerve?

75 ‫صفحة‬ Dr. MOHAMED YAHIA 0900988745


MRCOG RECALLS

a. Pudendal 109.What is the chemical structure of


b. Pelvic splanchnic Renin?
c. Hypogastric
d. Sacral a. Polypeptide
b. Oligopeptide
104.Which structure in female is c. Carbohydrate
homologus to male prostate? d. Steroid
e. Protein
a. Skenes gland
b. Gartner duct 110.The difference between the peak
c. Bartholin gland and the throughs of ultrasound wave is
d. Bulbourethral gland called?

105..Rate limiting step in the synthesis A. Amplitude


of catecholamines? B. Acoustic impedence
C. Frequency
a. acetylcholine esterase D. Resolution
b. DOPA decarboxylase E. Wave length
c. tyrosine hydroxylase
d. dopamine B hydroxylase 111..There are 3 components of
e. phenyl N methyl transferase interaction of ultrasound with the tissue
medium: absorption, scattering, and the
106..The anesthesiologist wanted to third one is...
anesthesize the anterior andomen using
TAP block. In this procedure where are a. cavitation
the nerves mostly located? b. reflection
c. refraction
a. between external and internal d. attenuation
oblique e. amplitude
b. between internal oblique and
transversus abdominis 112.A study between soya milk and
c. between transversus abdominis improvement of menopause symptom.
and transversalis fascia the null hypothesis H0 should state that
d. between transversalis fascia and
parietal peritoneum A. There is a insignificant relation/
difference between soya milk
107..Antidiuretic hormone (ADH) is intake and menopause symptoms
released from which part of B. There is a relation/ difference
hypothalamus: between soya milk intake and
menopause
A. Arcuate nucleus C. There is no relation/difference
B. Preoptic nucleus between soya milk intake and
C. Periventricular nucleus menopause symptoms
D. Suprachiasmatic nucleus
E. Supraoptic and Paraventricular 113.Blood supply to the medial part of
nuclei the anterior abdominal wall below the
umbilicus?
108.What is the chemical structure of
GnRH ? A- Superior epigastric artery
B- Superior mesenteric artery
a. glycoprotein C- Intercostal arteries
b. Tripeptide D- Inferior epigastric artery
c. Nonapeptide E- Inferior mesenteric artery
d. Octapepide
e. Decapeptide 114..A primigravida at 22 weeks is
scared to have vaginal delivery. On
examination she had absent clitoris and

76 ‫صفحة‬ Dr. MOHAMED YAHIA 0900988745


MRCOG RECALLS

normal labia majora. What type of d. d-luteal phase varies in duration


genital mutilation is it? by time taken for corpus luteum to
degenerate
a. Type 1 e. e-loss of progesterone secretion
b. Type 2
c. Type 3 120.A 60-year-old woman with stage IIB
d. Type 4 cervical cancer is currently undergoing
e. Type 5 radiotherapy by impact of gamma-
emitting isotopes containing Casieum
115..Regarding the following data: 137 on her vagina. This type of therapy
mean bmi is 25% and standard is best described by which of the
deviation is 5, what is the percentage of following terms?
obese class 2 and 3?
a. A Adjuvant
A. 2.5% B. Neoadjuvant
B. 5% C. Radical
C. 16% D. Brachytherapy
D. 32% E. Teletherapy
E. 34%
121.Choroid Plexus can be found
116..Ovarian CA associated with impinging on ?
mutation of mismatch repair genes
a. arachnoid and dura matter
a. clear cell Ca b. dura matter and pia matter
b. endometriod CA c. arachnoid and pia matter
c. papillary Ca d. dura matter and endocranium
d. Serous CA
e. Mucinous CA 122.Adult hemoglobin is synthesized
when?
117..What type of necrosis is seen in the
heart after an acute myocardial a. 28 wks
infarction? b. 34 wks
c. 36 wks
A. Liquefactive necrosis d. at birth
B. Coagulative necrosis e. 7 days post natal
C. Caseous necrosis
D. Gangrenous necrosis 123..Where is prolactin produced that is
E. Fibrinoid necrosis not under dopaminergic control?

118..Increase in cell size is called? a. maternal decidua


b. anterior pituitary
a. hyperplasia c. hypothalamus
b. hypertrophy d. posterior pituitary
c. atrophy e. warthon's jelly
d. metaplasia
e. dysplasia 124.Patient with ovarian mass plus
pleural effusion histopath revealed
119..Physiological variation in white, firm, spindle shape cells.
menstrual cycle between 21 to 35 days Diagnosis?
due to
a. glioma
a. a-aberrant LH surge b. fibroma
b. b-estrogen fails to produce c. sarcoma
negative feedback on pituitary d. leiomyoma
c. c-estrogen fails to produce e. adenoma
positive feedback on pituitary

77 ‫صفحة‬ Dr. MOHAMED YAHIA 0900988745


MRCOG RECALLS

125..You see a patient who is 35 weeks D) Staphylococcus aureus


pregnant in your day assessment unit. E) Enterococcus
She presents with itching... more on the
area of the abdominal striae. Lab 130..A lady was injured while walking
results given which showed normal in the garden barefoot. After that she
ALP, normal bilirubin levels. developed cellulitis.. What's the
Diagnosis? possible cause?

A. Intrahepatic Obstetric Cholestasis A. Stap aureus


B. Polymorphic Eruption of B. Strep Viridens
Pregnancy C. Staph Epidermidis
C. Fatty Liver D. Pseudomonas
D. Acute Hepatitis E. Clostridium
E. Cholecystitis
131.Nerve passing below inguinal
126..A 27-year-old woman after normal ligament medial to anterior superior
vaginal birth develops shortness of birth iliac spine?
and seizures. Despite resuscitation she
died. At autopsy squamous cells and A. Femoral n.
B. Anterior iliac n.
fetal hair are found in lungs. What is
C. Ilioinguinal n.
the most likely cause of death?
D. Genitofemoral n.
A. Amniotic fluid embolism E. Lateral cutaneous n. Of thigh
B. Placental abruption
C. Eclampsia 132..There is difficulty in extracting
D. Pulmonary embolism blood in the upper arm, the intern
E. Postpartum hemorrhage decided to extract from the femoral
vein. What is the relation of the femoral
127..Primary amenorrhea with absent vein to the artery and nerve from lateral
uterus, vagina ends in a blind pouch, to medial?
normal breasts and scant pubic hair is
a. VAN
seen in:
b. NAV
a. Klinefelter syndrome c. AVN
b. Mayer Rokitanski Kuster Hauser d. NVA
syndrome e. ANV
c. Turner’s syndrome
d. Androgen Insensitivity Syndrome 133.Which organelles do type II
e. Noonan syndrome pneumocytes utilize for surfactant
release?
128..Which one of the following bacteria
produces an exotoxin causing 'toxic A. Golgi complex
B. Lysosomes
shock syndrome'?
C. Endosomes
A. Escherichia coli D. Lamellar Bodies
B. Streptococcus agalactiae E. Rough endoplasmic reticulum
C. Clostridium welchii
D. Staphylococcus aureus 134.Most important complement in
E. Mycoplasma hominis opsonization

129..Which organism is the most a) C3a


b) C3b
common cause of sepsis in pregnancy?

A) Group A Streptococcus 135..Psammoma bodies are a histologic


B) Group B Streptococcus feature of ovarian papillary serous
C) Gardnerella vaginalis cystadenoma. ' Psammoma bodies' are
composed of ?

78 ‫صفحة‬ Dr. MOHAMED YAHIA 0900988745


MRCOG RECALLS

A. calcium 140..What is the most common cause of


B. iron acromegaly?
C. Fibrin
D. Mucin a. MEN I
E. Lipid b. MEN II
c. Pituitary infarct
136..23-year-old woman presents with d. Pituitary adenocarcinoma
fishy foul smelling vaginal odour. 'Clue e. Pituitary adenoma
cells' are found in the smear. Which of
the following cell types is belong to 141..What is the most common adverse
'Clue cells'? effect experienced by women taking the
progestogen- only pill?
A. Neutrophils
B. Natural killer cells A. Acne
C. Lymphocytes B. Weight gain
D. Macrophages C. Erratic bleeding
E. Epithelial cells D. Reduced libido
E. Mood swings
137..Vestibule is
142.Labetalol is:
a. area between two labia majora
b. area between two labia minora a) Selective alpha and beta blocker
c. area between hymen and labia b) Non Selective alpha and beta
minora blocker
d. area between hymen and labia c) Non selective beta blocker
majora d) Selective Alpha blocker
e) Direct vasodilation
138..WHO defines perinatal mortality
as 143..What is the mechanism of action of
hydralazine?
a. A.number of neonatal deaths per
1000 births a. Alpha 2 agonist
b. B.number of still births after 22 b. Angiotensin converting
wks & early neonatal deaths per enzyme inhibitor
1000 births B. C.Antimuscarinic
c. C.number of still births after 24 a. Beta 2 Agonist
weeks & early neonatal births per b. Direct acting smooth
1000 births muscle relaxant
d. D.number of still births per 1000
births 144..A woman experiences a
e. E.number of still births per year postpartum hemorrhage after delivery
and is administrated carboprost to
139..A 28-year-old woman with promote uterine contraction. What type
complete hydatidiform mole is found to of drug is carboprost?
have bilateral multiple tiny ovarian
cysts along with an enlarged uterus at A. Progestogen
12 weeks’ gestational age. The B. prostaglandin E1
C. prostaglandin E2
quantitative β-hCG level of 300,000
mIU/mL.What is the most likely type of D. prostaglandin F2
these ovarian cysts? 145..Patient has heavy menstrual
A. Polycystic ovaries bleeding. She desires an
B. Granulosa cell cyst endometrial ablation to stop bleeding
C. Follicular cyst after failure of medical treatment.
D. Theca lutein cyst✅ Which part of the uterus is destructed
E. Corpus luteum cyst to stop bleeding?

79 ‫صفحة‬ Dr. MOHAMED YAHIA 0900988745


MRCOG RECALLS

A. Basal layer of endometrium 149..The most common site of


B. Functional layer of endometrium endometriosis is?
C. Entire myometrium
D. Spiral artery A. ovary
B. Uterosacral ligament
146..A 28-year- C. Vagina
old woman has undergone emergency D. Rectum
hysterectomy due to obstetric E. Oviduct
haemorrhage during her 3rd pregnan
cy. The previous pregnancies were bo 150.In which subsites does vulvar
th term C- cancer most commonly arise?
section deliveries. Examination of the
A. Mons pubis
uterus shows the placenta had invad
B. Labia majora
ed through the full thickness of the
myometrium. C. Labia minora
D. Clitoris
What is the most likely diagnosis?
E. Posterior fourchette
A. Placenta praevia
B. Placenta accreta 151..What type of Laser is Neodymium:
C. Placenta increta YAG laser?
D. Invasive mole A. Semiconductor laser
B. Excimer Laser
146.What is the new non invasive
C. Dye Laser
prenatal diagnostic test of cell-free fetal
D. Gas Laser
DNA of maternal blood derived from?
E. Solid Laser
A. Fetal RBCs
B. Fetal WBCs 152..Which type of epithelium lines the
C. Fetal fibroblasts distal urethra near the external urethral
D. Amniotic cells orifice?
E. Placental Trophoblast A. Transitional
147..Which of the option correctly B. Squamous✅
identified the embryological origin C. Cuboidal
D. Stratified columnar
beginning and ending of the round
E. Simple columnar
ligament of the uterus?

A. Gubernaculum ovarii , ant surface 153..A 35 year old woman has a pelvic
of uterine cornua, labia majus ultrasound scan showing multiple
B. Gubernaculum ovarii, post inf to fibroids. What is the most common
uterine cornua, labium majus form of fibroid degeneration?
C. Gubernaculum ovarii post inf to a. Red degeneration
the uterine cornua,labium minus b. Hyaline degeneration
D. Paramesonephric duct, post sup to c. Carneous degeneration
uterine cornua, labium minus d. Myxoid degeneration
E. Paramesonephric duct, ant inf to e. Cystic degeneration
uterine cornua, labium majus
154..From where in the nephron is the
148.Which of the following is enzyme renin secreted ?
predominant type of bacteria in the
vagina during pregnancy: a) Collecting ducts
b) Distal convoluted tubules
A. A-lactobacillus c) Juxtaglomerular apparatus
B. B-peptostreptococcus. d) Loop of Henle
C. C-listeria monocytogenous. e) Proximal convoluted tubules
D. D-streptococcus agalactia.

80 ‫صفحة‬ Dr. MOHAMED YAHIA 0900988745


MRCOG RECALLS

155.Which neoplasm is E) Fibrous


characteristically associated with
paraneoplastic syndrome of 161.Which kind of cell produce
inappropriate antidiuretic hormone parathyroid hormone?
(SIADH)?
A. chief cells
A. Renal cell carcinoma B. oxyphilic cells
B. Breast carcinoma C. parathynotic cells
C. Small cell lung carcinoma D. follicular cells
D. Papillary thyroid carcinoma E. basophil cells
E. Cancer colon
162.Which chromosomes are capable of
156.Which of the following drugs is used robertsonian translocations?
to suppress lactation postpartum in
A. X, Y
mother who suffers from stillbirth?
B. 15, 17, 19
A. Quetiapine C. 16, 18, 20
B. Metoclopramide D. 9, 10, 11, 12
C. Loperamide E. 13, 14, 15, 21, 22
D. Leuprolide
E. Cabergoline✅ 163.Which signaling molecule that
stimulate the renin-angiotensin system
157.Wernicke-Korsakoff syndrome to regulate the sodium concentration?
Deficiency disease of vitamin?
A. Angiotensin
A. Vitamin B1 ✅ B. Aldosterone
B. Vitamin B2 C. Bradykinin
C. Vitamin B3 D. Oxytocin
D. Vitamin B6
E. Vitamin B12 164..Which class I major
histocompatibility complex human
158.Which types of cells are phagocytic leucocyte antigen (HLA) is not
for residual bodies left over from the expressed in extravillous trophoblast?
process of spermiogenesis?
A. HLA-A, HLA-B
A. Testicular mesenchymal cells B. HLA-B, HLA-C
B. Testicular macrophages C. HLA-C, HLA-F
C. Fibroblast cells D. HLA-F, HLA-DR
D. Interstitial cells of leydig E. HLA-DR, HLA-G
E. Sertoli cells
165.31 yr old c/o subfertility for 2yrs ,
159.Insensible loss of water from sweat no medical problem , BMI 38kg/m2 ,
and lungs in 70 yr old healthy woman is hormone result :

A. 100ml/kg  FSH 8
B. 450ml/kg  LH 18
C. 850ml/kg  progesterone 4 nm m/l
D. 1350ml/kg  E2 198
E. 2600 ml/kg  HSG patent bil tubes
 U/s poly cystic ovaries ,
160.The sacroiliac joint is what type of
joint? Male ( husband ) normal ,

A) Condyloid What is most appropriate management


B) Synovial :
C) Primary Cartilaginous
D) Secondary Cartilaginous a. Clomiphene

81 ‫صفحة‬ Dr. MOHAMED YAHIA 0900988745


MRCOG RECALLS

b. Metformin ages.What is the correct interpretation


c. Metformin + Clomiphene of this chart?
d. Letrozole
e. Weight loss✅ a. Normal Growth
b. Below the 10th percentile
166.Woman climbed up the hill with her c. Above the 10th percentile
friends suddenly felt breathless plus d. Small for gestational age (SGA)
parasthesia in e. Large for gestational age (LGA)

her hands. What is the acid-base 172..Trichomonas Vaginalis ????


imbalance ?
a. facultative anaerobe
A. Metabolic alkalosis b. flagelated protozoan✅
B. Metabolic acidosis c. obligate aerobe
C. Respiratory alkalosis d. obligate anaerobe
D. Respiratory acidosis e. fungi
E. Mixed acidosis
173..A midwife asks the obstetric
167.What type of acid–base disturbance registrar to review a primigravida in
result in a case of Conn's syndrome? labour who has progressed to 6 cm
dilatation with an abnormal CTG. The
A. Metabolic acidosis CTG has been normal up to 30 minutes
B. Metabolic alkalosis previously. The registrar reviews the
C. No effect patient and confirms that the CTG has
D. Respiratory acidosis a baseline heart rate of 150 bpm with a
E. Respiratory alkalosis baseline variability of 4 for the last 30
minutes. There were no accelerations
168..The Haldane effect refers to which but variable decelerations were present.
of the following options? What would be appropriate
A. The inhibitory effect of oxygen on management with these CTG findings?
fermentation
B. The chloride shift that maintains
electrical neutrality
C. The dissociation constant for the
bicarbonate buffer system
D. The increased capacity for
deoxygenated blood to carry CO2
E. The reduction in affinity of
hemoglobin for oxygen in active A. This is a normal CTG and no
tissues in response to intervention required
B. This is a suspicious CTG and
a drop in pH caused by increased CO2 requires continued obstetric
production review
C. This is a pathological CTG and
169..Areas appear dark or black in fetal blood sampling is required
ultrasound? ???? D. This is a pathological CTG and
urgent delivery is required
a. hyperechoic (Category 1 CS)
b. anechoic E. This is a suspicious CTG and fetal
c. isoechoic blood sampling is required
d. hypoechoic
174.25 y/o had lscs at term for breech
170.The following intrauterine growth presentation with spinal anesthesia
chart shows the 10th, 50th and 90th After what timen you start low dose
weight percentiles by gestational molecular heparin(LMWH)

82 ‫صفحة‬ Dr. MOHAMED YAHIA 0900988745


MRCOG RECALLS

a. 4 hrs a. internal pudendal


b. 6 hrs b. superior hemorrhoidal
c. 12 hrs c. inferior mesenteric
d. 24 hrs d. superior vesical
e. No need for treatment only e. ovarian
hydration and mobilization✅
181..Renal Calculus referred
175..VIN3 is caused by which organism pain ??????
?
a. groin
a. HPV 11 b. lumbar region
b. HPV 16 c. anterior abdominal wall
c. HIV d. umbilicus
d. HLT e. hypochondriac area

176..Zika virus is 182..CAH is caused by 21 hydroxylase


the remaining is due to?
a. Ss DNA
b. DsDNA a. 17a- hydroxylase
c. Ss RNA b. 11b- hydroxylase
d. Ds RNA c. 3b HSD
e. DNA replicating RNA d. 20,22 desmolase

177.Thrombosis in protein C deficiency 183..Incidence of "RARE" disease when


is due to translated into mathematics is?

a. A.decrease antithrombin 3 a. 1:100


b. B.Factor 5 and 8a b. 1:1000
c. 1:10,000
178..Transport of iodide in the cells is d. 1:10,000 -1:100,000
associated with e. 1:100,000
a. calcuim 184..You are assessing a patient with an
b. sodium ovarian mass and are using the Risk of
c. chlorine Malignancy Index 1 (RMI) score.
d. magnesium Which of the following is used to
e. potassium calculate the RMI?
179..Patient presented with heavy A. Menopause status, Serum Ca 125
vaginal discharge but no itching,PH 6. iu/ml, US score
What is the cause? B. Menopause status , Serum Ca 125
mg/dl, US score
a. Candida C. Age, Serum Ca 125 iu/ml, US
b. Bacterial vaginosis score
c. Trichomonas vaginalis D. Age, Serum Ca 125 mg/dl, US
d. Physiological discharge score
e. Chlamydia trachomatis
185.Embryonic origin of oocytes
180..During a hysterectomy, vaginal
bleeding may be a significant a. extra embryonic coelom
complication even after removal of the b. yolk sac
uterus and ligation of the uterine c. primordial germ cell
arteries. Such bleeding would most
likely originate from which of the 186..Definition of Tidal Volume
following arteries which anastomoses
with uterine artery? a. amount of air left inside the lungs
after forceful expiration.

83 ‫صفحة‬ Dr. MOHAMED YAHIA 0900988745


MRCOG RECALLS

b. amount of air that goes in and out c. weakening of conjoint tendon


of the lungs during normal
respiration 193..What is the most common cause of
c. additional amount of air that can endogenous Cushing's syndrome?
be expired from the lungs by
determined effort after normal a. Small cell carcinoma of the lung
expiration b. Adrenal adenoma
d. additional amount of air that can c. Phaeochromocytoma
be inspired from the lungs by d. Multiple Endocrine Neoplasia
determined effort after normal type 1
inspiration e. Pituitary adenoma
e. the volume of air contained in the 194..What is the most common cause of
lungs at the end of a maximal hyperthyroidism in pregnant woman?
inspiration
a. Adenoma
187..What type of compound is b. Graves disease
Histones? c. Iodine excess
A. Carbohydrate d. Subacute thyroiditis
B. Fatty acid e. Toxic multinodular goitre
C. Prostaglandin 195..Trophoblast cells that invade the
D. Protein lumen of spiral arteries
E. Steroid
a. extravillous trophoblast
188..Most important function of b. endovascular trophoblast
Vitamin C c. interstitial trophoblast
a. antioxidant d. villous trophoblast
b. boost immune system e. extravascular trophoblast
c. collagen formation 196..Patient pregnant with right
d. helps in wound healing adnexal mass on ultrasound not
189..Cells in the body produce separable from ovary with no
hormones among them how many are intrauterine gestational sac sac, patient
lactotrophs? is stable

a. 1-5% a. rescan after 7 days


b. 10-15% b. serum b hcg 48 hrs apart
c. 15-25% c. explore lap
d. 25-30% 197..Sudden disorientation of a women
e. 40-50% following delivery now on 35% Oxygen
190..Inflammatory mediators involved Abgs showed
in wound healing  Ph 7.2
a. tumor necrosis factor  Spo2 13.4
b. vascular endothetial growth factor  Spco2 8.4
c. transforming growth factor beta  Base excess -2.4
d. interferon  Hco3 25
e. interleukin Whats the metabolic disorder?
192..Indirect Inguinal Hernia a. Respiratory Acidosis
a. lateral to inferior epigastric b. Respiratory Alkalosis
arteries c. Metabolic Acidosis
b. medial to inferior epigastric d. Metabolic Alkalosis
arteries

84 ‫صفحة‬ Dr. MOHAMED YAHIA 0900988745


MRCOG RECALLS

e. Respiratory Acidosis with d. Coagulation Screen, FBC, US


Metabolic compensation e. US, TFT, Endometrial biopsy

198..Heavy menstrual bleeding, tired, 199..HPV proliferate in?


lethargy, examination reveals larger
uterus, what investigation will u do? a. Basal Cell of
endometrium
a. Coagulation Screen, FBC, US, b. Liver sinusoids
TFT c. Spleen
b. FBC, US, TFT d. Bone marrow
c. CBC, Endometrial biopsy e. Trigeminal neuron

85 ‫صفحة‬ Dr. MOHAMED YAHIA 0900988745


MRCOG RECALLS

EXAM 5

86 ‫صفحة‬ Dr. MOHAMED YAHIA 0900988745


MRCOG RECALLS

1. Atosiban is d. prothrombin
e. Vitamin K antagonist
A. Progesterone agonist
B. Progesterone antagonist 6. Grade B recommendation of RCOG
C. Use as tocolytic means
D. Oxitocin agonist
E. Oxytocin antagonist a. at least one randomized controlled
trial
2. 30-year-old woman with bipolar b. meta analysis from several
disorder is 12-week pregnant. She has randomized controlled trials
been taken her lithium every night. c. at least one well designed study
Which of the following abnormality not randomized
may occur if she continues to take a. d. well controlled studies but
lithium?
non randomized
A. Tetralogy of Fallot's b. e. expert opinion
B. Dandy walker syndrome
C. Ebstein Anomaly 7. Appendiceal artery is a branch of ?
D. Limb reduction deformity
a. Ileocolic artery
A. E. Transposition of great
b. inferior mesenteric a.
arteries c. left colic a.
d. middle colic a.
3. A 7 week pregnant women with
e. right colic a.
history of previous two miscarriage.
Ultrasound showed... 8. What is the most common cause of
the female urethral caruncle?

A. Trauma
B. Tumour
C. Infection
D. Hypoestrogenism
E. Congenital

What is the most likely karyotype ? 9. What type of variable is ABO Blood
Group ?
A. 46, XX
B. 46, XY A. Ordinal
C. 69, XXX B. Nominal
D. 69, XXY C. Continuous
E. 69, XYY D. Categorical
E. Discrete
4. The antibacterial action of
BenzylPenicillin is due to its effect on? 10. Which of the following is precursor
of Serotonin?
a. Cell membrane permeabilty
b. Cell wall synthesis A) Histidine.
c. Protein synthesis B) Glycine.
C) Arginine.
d. DNA synthesis e RNA synthesis D) Tyrosine.
E) Tryptophan
5. Warfarin acts where in the clotting
cascade?(SEPT 2017) 11.Identify the area represented by B in
the diagram
a. factor V
b. factor VII
c. fibrinogen

87 ‫صفحة‬ Dr. MOHAMED YAHIA 0900988745


MRCOG RECALLS

hysterectomy is performed, and


pathologic examination of the removed
uterus reveals heterologous components
of a malignant tumor of the endometrial
glands and metaplastic cartilage. Which
of the following is the most likely
diagnosis?

a. Pudendal cleft A. Stromal sarcoma


b. Vestibule B. Carcinosarcoma
c. c. Fourchette C. Adenosarcoma
d. Fossa Navicularis D. Endometrioma
E. Adenocarcinoma
12. Wernicke-Korsakoff syndrome
Deficiency disease of vitamin? 17. For overactive bladder a new drug
called Mirabegron is used. Which is the
A. Vitamin B1 single mechanism of action for this
B. Vitamin B2 drug?
C. Vitamin B3
D. Vitamin B6 A. Selective serotonin re-uptake
E. Vitamin B12 inhibitor
B. Muscarinic receptors agonist
13. Incidence of "RARE" disease when C. Selective Beta-1 agonist
translated into mathematics is? D. Selective Beta-3 agonist
E. Selective Beta-3 antagonist
a. 1:100
b. 1:1000 18. Which of the following is a Side
c. 1:1000- 1:10,000 effect of Terbutaline
d. 1:10,000 -1:100,000
e. >1:100,000 A. A . Maternal tachycardia
B. B . Maternal bradycardia
14. Which protein is coiled in DNA? C. C . Fetal tachycardia
D. D . Fetal bradycardia
a. Polymerase
E. E. IUGR
b. B. Cytosine
c. C. Helicase 19. A 50 years old women with absent
d. Histones periods for 2years, has bilateral ovarian
e. Steroid mass with solid areas on scan and CA
125 of 100. What is her RMI score?
15. A 16-year-old has recently become
sexually active. She complains of A. A.900
intensely irritating greenish frothy B. B.600
vaginal discharge. The organism is seen C. c.700
under microscope with peri nuclear
halo. What is the most likely causative D. d.450
organism? E. e.25
a. Candida albicans 20. In ECG which lead reflects the
b. B. Chlamydia trachomatis inferior wall of heart?
c. Gardnerella vaginalis
d. Trichomonas vaginalis A. V1, V2
e. Treponema palladium B. B. V3, V4

16. A 65-year-old nulliparous women C. C. aVR


presents with vaginal spotting. A D. I, aVL

88 ‫صفحة‬ Dr. MOHAMED YAHIA 0900988745


MRCOG RECALLS

E. E. II, III, aVF A. Chancre


B. Chancroid
21. What is the embryological origin of C. Granuloma inguinale
Gartners duct cyst? D. Condyloma acuminatum
E. Molluscum contagiosum
A. Cloaca
27. What drug can be given in
B. B. Genital tubercle methotrexate toxicity?
C. C. Mesonephric duct
A. Folic acid
D. D. Paramesonephic duct B. Vitamin B12
E. Urogenital septum A. C. Pyridoxine
22. What is the SI unit for the activity of C. Thymidine
radioactive decay? D. Folinic acid

A. Gray 28. 40-year-old woman had bilateral


B. Sievert silicone breast implants placed two
C. Rad years ago. Since that time, she has noted
D. Roentgen increased firmness with slight deformity
E. Becquerel of the breast on the left. The implants
are removed, and there is evidence for
23. What is the contrast dye used in leakage of the implant contents on the
MRI scans? left. Which of the following cell types is
most likely to be most characteristic of
A. Iodine the inflammatory response in this
a. B. Barium situation?
b. C. Thallium A. Neutrophils
E. Gadolinium B. Mast cells
F. Tecnisium C. Plasma cells
D. Giant cells
24. Which of the following hormones E. T lymphocytes
increases the excretion of calcium in
Kidney? 29. Which is common malignant tumour
in newborn?
A. A. Insulin
A. Retinoblastoma
B. Cortisol B. Teratoma
C. Calcitonin C. Neurofibroma
D. D. parathyroid hormone D. Sarcococcygeal teratoma
E. Antidiuretic hormone E. E. Granulosa cell tumour

25. When Choriocarcinoma 30. Which of the following is


metastasizes, it has a predominantly contraindication of the use of
spread by which route ? oxybutynin?

A. Direct invasion A. Bronchial asthma


B. Hematogenous B. Narrow angle glaucoma
A. C. Lymphatic C. Diabetes mellitus
D. Parkinsonism
C. Surface implantation E. Peptic ulcer
D. Transcoelomic
31. A neoplasm of the female genital
26. What sexually transmitted disease tract occurring in an 18-year-old girl
(STD) is caused by Haemophilus whose mother was treated with
ducreyi?

89 ‫صفحة‬ Dr. MOHAMED YAHIA 0900988745


MRCOG RECALLS

diethylstilbestrol during the pregnancy E. X-linked recessive


is likely to be a:
35. The most common cause of delay in
A. Brenner cell tumor of ovary puberty in males is:
B. Teratoma of ovary
C. Sarcoma botryoides of vagina A. Constitutional delay in growth
D. Clear cell carcinoma of vagina F. Klinefelter's syndrome
A. E. Squamous cell carcinoma of G. C. Noonan's syndrome
vulva C. Primary hypothyroidism
D. Gonadal dysgenesis
32. Active form of Vit-D is produced in
which organ? 36. Lancefield grouping of streptococci
is based on the presence of ?
a. Skin
b. Liver A. Carbohydrate antigen on cell wall
c. Lungs A. B. Peptidoglycan antigen on cell
d. Kidney wall
e. Adrenal glands
B. C. M protein
33. Regarding the following C. Color of blood agar
Karyotyping Which of the following is D. Sugar fermentation
the most likely diagnosis?
37. Antihypertensive which causes
postnatal depression, when used in
permpartum period?

A. Methyl dopa
b. Labetalol
c. Nifedipine
d. Lisinopril
E. Hydralazine
A. Cri-du-chat syndrome 38. Clomifene citrate mechanism of
a. B. Down syndrome action ?
B. Turner syndrome
A. Androgenic steroid
C. Edward syndrome
B. Estrogenic steroid
D. Patau syndrome
C. Progestogenic steroid
34. Considering the following pedigree. F. D. Gonadotropin analogue
G. E. Selective estrogen receptor
modulator

39. Which of the following


Antineoplastic drugs causing
hemorrhagic cystitis?

A. Cisplatin
A. B. Methotrexate
What is the mode of inheritance? B. C. Paclitaxel
A. Autosomal dominant C. Cyclophosphamide
B. X-linked dominant D. Vincristine
A. C. Y-linked dominant
B. D. Autosomal recessive

90 ‫صفحة‬ Dr. MOHAMED YAHIA 0900988745


MRCOG RECALLS

40. Luteinizing hormone (LH) is o Adenocarcinoma


structurally related to what other
glycoprotein hormones? 45. Which of the following organisms
Colonize copper IUD?
A. A. ADH, oxytocin and FSH
o Lactobacillus.
B. B. FSH, TSH and growth o Actinomyces species.
hormone o Mobilincus
C. HCG, FSH and TSH  D. Ureaplasma
D. D. HCG, ADH and FSH  E. Staphyllococcus aurues
E. TSH, growth hormone and HCG
46. Bacterial vaginosis is diagnosed with
41. Mechanism of action of ulipristal depletion in numbers of which
acetate ? organism?
A. Partial progesterone
receptor blockage  A. Lactobacillus
B. Selective Estrogen  Mobiluncus
receptor modulator  Gardnella vaginalis
C. Prostaglandin analogue.  D. Trichomonas vaginalis
D. Selective progesterone
 Trepenoma pallidum.
receptor modulator
E. Oxytocin Antagonist 47. The most common in Utero infection
causing fetal anemia in the UK is
42. Which one of the following is the
termination of round ligament?  Syphilis
A. Labia minora  B. Toxoplasmosis
B. Labia majora o Rubella
a. C. Deep Inguinal ring o Parvovirus B19
o Cytomegalovirus
C. Superficial Inguinal ring
D. Lateral vaginal wall 48. Which HPV causes Genital Warts?
43.Ductus venosus is completely
obliterated after birth to form what o HPV 6 and 11✔
adult  B. HPV 16 and 18
structure?  C. HPV 16 and 45
 HPV 18 and 45
 A. Ligamentum venosum  HPV 31 and 33
 B. Ligamentum arteriosum 49. A Women with 150cm height and 90
 Ligamentum teres kgs weight, What is her category of
 Medial umbilical ligament BMI?
 Median umbilical ligament
 A. Normal
44. A 60-year-old with erythematous
 Overweight
erosive lichen planus on the vulva. What
type of cancer is linked to lichen  Obesity Class I
planus?  Obesity Class II
 Obesity Class III
o Squamous cell carcinoma
50. Sheehan syndrome affects?
 B. Basal cell carcinoma
o Merkel cell carcinoma
o Malignant melanoma

91 ‫صفحة‬ Dr. MOHAMED YAHIA 0900988745


MRCOG RECALLS

o Hypothalamus B. o Rapport luebering cycle


o Cori's cycle
Anterior pituitary
o Posterior pituitary  C. Citric acid cycle
o Thyroid  D. Glycolysis
o Adrenal cortex
 kreb's cycle
51. A woman is 8 weeks pregnant
attends for genetic counseling as she 54. Which substance is indicated by
worried about CF, her sister has a child letter 'X' in the diagram below?
with cystic fibrosis. She is a carrier and
her partner is Caucasian with a rate of
1/25 What is the risk of the child having
cystic fibrosis? (MARCH 2017)

o 1%
 B. 12.5%
o 25% o Cholesterol
o 75%
o 100%  B. Testosterone
 C. Dihydrotestosterone
52. A 25-year-old woman presents with
right-sided pelvic pain for more than  Androstenedione
one year. An ultrasound scan is  E. Aldosterone
arranged which shows a 5 cm well-
defined cystic lesion in the right adnexa 55. A 33-year-old woman is admitted
with mixed echoes. There is no fluid in with surgical wound infection after
the cul-de-sac with the following emergency caesarean section for
sonographic image: What is the most suspected fetal compromise. Culture of
likely diagnosis for the image of the the wound demonstrates methicillin-
adnexa? resistant Staphylococcus aureus
(MRSA). She has no known allergies.
Which is the most appropriate
treatment?

o Cephalosporins
 B. Clindamycin + flucloxacillin
o Vancomycin
o Clindamycin and
o Benign cystic teratoma vancomycin
o Piperacillin/tazobactam
 B. Endodermal sinus tumour
o Endometrioma 56. Post-delivery the registrar notes that
 D. Mucinous cystadenoma baby's HR 150bpm, pink body and blue
extremities, Grimace, some flexion &
 Serous cystadenoma friable cry. What is the apgar score? 3
53. Regarding the following image what o 4
it is called ? o 6
o 8
 e. 10

57. What is affected in Horner


syndrome

92 ‫صفحة‬ Dr. MOHAMED YAHIA 0900988745


MRCOG RECALLS

o Loss of Motor supply  Carbonic anhydrase


o Excessive Sympathetic  Pyruvate kinase
stimulation
o Excessive 63. Correct interpretation of the
Parasympathetic following Urogram :
stimulation
o Loss of Parasympathetic
supply
o Loss of Sympathetic
supply

58 Hashimotos thoiditis

 C. Toxic nodule
o Ketoacidosis Stress incontinence.
 E. Hypothyroidism ✔
o Urge incontinence
59. What is the most common cause of  C. Detrusor muscle instability
cushing's syndrome?  D. Detrusor muscle overactivity

o Pituitary adenoma B.  E. Urinary tract infection .


Adrenal adenoma C. 64. A town with a total number of live
Adrenal carcinoma birth is 250,000, and maternal death is
 Adrenal hyperplasia E. 750, number of stillbirth is 250, what is
Iatrogenic Steroid Administration the Maternal mortality ratio?

o 300
60. Glucocorticoids are secreted from
o 600
which region of the adrenal gland?(
o 900
Adrenal capsule
o 200
o Zona reticularis  750
o Zona glomerulosa
o Zona fasciculata 65. A women 40 week induced by
dinoprostone pessary 20 minutes later
E. Adrenal medulla and the pessary is removed . Regarding
the following CTG , what is the most
61. Which cells produce mullerian- appropriate management ?
inhibiting substance in fetal testis?

Interstitial cells of leydig

 B. Interstitial macrophages
o Sertoli cells
o Theca cells o Give terbutuline 20 mg
 E. Granulosa cells tocolytic.
o Category 1 CS.
62. Which enzyme found in red blood  C. Category 2 CS.
cells which buffers blood CO2?
 D. Reassure and observe
o Cytochrome-b5 reductase  Perform fetal blood sampling.
 B. 5-nucleotidase
66. In a town with 20,000 live birth,
 C. Glucose-6-phosphate calculate perinatal mortality ?
dehydrogenase (MARCH 2017)

93 ‫صفحة‬ Dr. MOHAMED YAHIA 0900988745


MRCOG RECALLS

Weeks Prenatally Postnatally o Nuclear transcription


factor
20w 20 o Tyrosine kinase nuclear
receptor
21w 10 o Ligand-gated ion channel
22w 40 in the cell membrane
o G protein coupled
23w 10 receptor on the Golgi
complex
24w 20
70. A 67-year-old woman attends for a
25w 20 preoperative assessment. She has been
taking bendroflumethiazide for several
26-30w 10 years for hypertension. Her ECG
shows: What is the most likely
31-35w 20
electrolyte imbalance?
35-42w 10

1st week after birth 50

o 9/1000
 B. 180/24000
o 160/20000
o 210/20000
o 240/20000 Hypercalcaemia

67. What is the anatomical space of o Hyperkalaemia


External uretheral sphincter? o Hypernatraemia
o Hypokalaemia
o Superficial perineal o Hyponatraemia
pouch.
71. Which of the following drugs cause
 B. Deep perineal pouch.
Kernictrus ?
 C. Ischiorectal fossa.
o Penicillin
 Superficial inguinal space.
 Deep inguinal space  B. Gentamycin
o Sulphonamides
68. A Women after delivery found to be o Methotrexate.
in lithotomy position for 2 hours , now o Erythromycin
she develop drop foot , what nerve most
likely to be the cause ? 72. Regarding cell cycle , when does
chromatids divides and seperates to
 A. Tibial nerve. opposite poles ?
 B. Femoral nerve. o Prophase.
 Sciatic nerve.  B. Metaphase.
 D. Common peroneal nerve.  C. Anaphase. Telophase.
 Lateral cutaneous nerve  Interphase.
69. The Progesterone receptor is 73. A Couple with primary subfertility ,
example of which type of receptor ? who have been trying to conceive for
o Guanylate cyclase over 12 months , attends a reproductive
receptor medicine clinic , the male partner has
already give a semen sample for

94 ‫صفحة‬ Dr. MOHAMED YAHIA 0900988745


MRCOG RECALLS

analysis which return back showing : 32 o Square root SD / n


million spermatozoa , morphology >5 , o SD / n
vitality >5% , live spermatozoa 63% , o SD / Square root n
PH 7.2 , What is the correct o SD x square root n
interpretation of this result ? o SD – Square root n

o Normal semen analysis 79. A 55 year old presents to clinic due


o Azoospermia to vulval itch and discolouration.
 C. Oligospermia examination reveals pale white
discoloured areas to the vulva. A biopsy
 D. Asthenospermia shows epidermal atrophy with
 PH indicate infection. subepidermal hyalinization and deeper
inflammatory infiltrate. What is this
74. Regarding Insulin, it gets activated characteristic of?
by removing ?
o Lichen Simplex Chronicus
o A-peptide
 B. Vulval intraepithelial
 B. B-peptide neoplasia
 C. C-peptide o Vitiligo
 D-peptide  D. Extramammary Pagets
 E-peptide  E. Lichen Sclerosus
75. What represent the 2 axis of ROC
80. A Pregnant lady 11 week , presented
curve ?
by UTI , what the best course of action ?
o Senstivity Vs 1/specificity
o Penicillin.
o Sensitivity Vs 1-
Specificity  B. Gentamycin
o Senstivity Vs specifity o Doxycyline
o 1-Senstivity Vs 1-  D. Nitrofurnatoin.
Specificity
o 1/Senstivity Vs  Trimethoprim
1/Specificity 81. What is the Action of mRNA ?
76. What hormone secreted by placenta o Translation
leading to fetal diuresis ? o Transcription
o Relaxin.  C. Transfer
 B. Estriol o DNA synthesis
o Oxytocin  E. Reverse transcription
o Vasopressinase.
o Inhibin 82. From Which structure inhibin is
produced ?
77. What is the blood supply of Foregut
? o Theca cells
 B. Granulosa cells.
o Celiac trunk.
o Superior mesenteric artery  C. Cumulus oophorus
o Inferior mesenteric artery  Corpus leutem
o Abdominal aorta  Lydig cells
o Internal iliac artery
83. At what level ovarian artery arise ?
78. Regarding Standard error of mean [
SEM ] how it is calculated ? o L1
o L2

95 ‫صفحة‬ Dr. MOHAMED YAHIA 0900988745


MRCOG RECALLS

o L3 o Deep perineal pouch


o L4  B. Ischiorectal fossa
o L5
 C. Pudendal canal
84. Which of the following muscles  Superficial perineal pouch
leaves the lesser pelvis through the
greater sciatic foramen?  E. Urogenital diaphragm

89. What do the bottom and top


 A. pubococcygeus
represent on the box-and-whisker plot?
 B. Iliococcygeus
o Mean and mode
 Puborectalis
 B. Mean and median
 D. Piriformis
 Coccygeus  C. Standard deviation and mean
 Least and greatest value
85. To reduce postoperative pain after  Lower and upper quartile
caesarean section, an obstetrician
performs ilioinginaliliohypogastric 90. Calculate the odds of asymmetrical
nerve block (IINB) on his 25-year-old IUGR and symmetrical IUGR for
patient. What is the nerve root of Apgar < 7
ilioinguinal and iliohypogastric nerve?
Assymetrical.
 A. T12 Symmetrical
 T12 and L1 Apgar < 7. 1 1
 L1
 D. L2 Apgar > 7 1 4
 L1 and L2 o 4
o 2.5
86. Following radical hysterectomy and o 16
pelvic lymphadenectomy for Stage I o 1.5
carcinoma of the cervix, 63-year-old
o 1
woman reports weakness of hip
adduction. Which nerve most likely to 91. Which hormone prevent regression
have been injured during the operation? of corpus luteum?
o Obturator nerve o HCG
o Femoral nerve
 B. Progesterone
 C. Inferior gluteal nerve
o Superior gluteal nerve  C. Estrogen
o Sciatic nerve  HPL
 FSH
87. What is the level of dermatome for
epidural at umblical region ? 92. Hormone that regulates apetite and
is important in obesity?
o L1
o Leptin
 B. T12
o Relaxin
 C. T10 o Estrogen
 L3 o Progesterone
 L4
93. In which part of the cell does
88. What is the anatomical location of glycolysis take place?
Bartholin’s gland?
o Cytosol

96 ‫صفحة‬ Dr. MOHAMED YAHIA 0900988745


MRCOG RECALLS

 B. Mitochondrial matrix 98. Botulinum toxin is now used to treat


neurogenic bladder because of its ability
o Mitochondrial cristae
to
o Rough endoplasmic
reticulum o Inhibit the release of
o Smooth endoplasmic acetylcholine
reticulum o Block storage of
acetylcholine
94. A 14-year-old girl is treated with
radiotherapy. Which of following tissues  C. Inhibit acetylcholine ACH
is most radioresistant? synthesis
o Block bladder nicotinic
 A. Intestinal epithelium receptors
 Cerebral nuerons  E. Block bladder muscarinic
 Respiratory epithelium receptors
 Bone marrow
 Skin 99. What is the embryonic origin of the
trigone of urinary bladder?
95. Two months postoperatively, a 55-
year-old woman who is being treated o Mesonephric duct
with a diagnosis of small lung cancer  B. Paramesonephric duct
presents with excessive polyuria and o Ureteric bud
depression and urine showed red cells
 D. Urogenital sinus
+++ what is the electrolyte imbalance?
 E. Caudal of cloaca
o Hypernetremia
 B. Hyponatremia 100. In the development of the oocyte,
when does the second meiotic division
 C. Hyperkalemia
occur?
 Hypokalemia
 E. Hypercalcemia  A. At birth
 B. When sperm penetrates the
96. Which pattern of endometrium do
simple endometrial hyperplasia most zone pellucida
likely resemble?  At Ovulation
 D. When sperm enters the tube
o Proliferative endometrium
 E. Before the sperm enters the
 B. Secretory endometrium
zone pellucida
o Endometritis
o Endometrial polyp 101. A 28-year-old woman has
 E. Endometrial carcinoma undergone emergency hysterectomy due
to obstetric haemorrhage during her
97. Which class I major 3rd pregnancy. The previous
histocompatibility complex human pregnancies were both term C-section
leucocyte antigen (HLA) is expressed deliveries. Examination of the uterus
only in extravillous trophoblast? shows the placenta has not invaded the
myometrium. What is the most likely
 A. HLA-A diagnosis?
 HLA-B o Placenta praevia
 HLA-C o Placenta accreta
 D. HLA-F o Placenta increta
o Invasive mole
 E. HLA-G
o Placenta percreta

97 ‫صفحة‬ Dr. MOHAMED YAHIA 0900988745


MRCOG RECALLS

102. The rate of growth hormone with a CRL of 4 mm. There is no fetal
secretion follows a circardian rhythm. heart action noted. The ovaries and
When is the highest peak of growth adnexa appear normal. What is the
hormone release? most likely diagnosis?

o Early morning o Heterotopic pregnancy


o After exercise  B. Viable intrauterine
o Afternoon
pregnancy
 D. Early sleep o Could be a viable
o Deep sleep pregnancy
 D. Non viable pregnancy
103. The most biologically active form
of vitamin D is  E. Threatened miscarriage

o Calcidiol 106. A publication reports the outcome


 B. Ergosterol of a new therapy in a placebo controlled
 C.7-dehydrocholesterol primary prevention of venous
thromboembolism (VTE). 200 patients
 D. 1, 25- were randomised to receive the new
Dihydroxycholecalciferol therapy and 200 allocated to placebo.
 E. 24, 25- The study was completed over a 2 year
Dihydroxycholecalciferol period. In the placebo group there were
20 have venous thromboembolism and
104. Which muscles form the pelvic in the group treated with the new
diaphragm? therapy there were 10 have venous
thromboembolism. What is the number
o Levator ani and coccygeus needed to treat to prevent one VTE over
 B. Levator ani and piriformis the course of this study?
o Obturator internus and b. 2
coccygeus
o Obturator internus and c. B. 4
levator ani d. C. 10
o Obturator internus and
piriformis e. D. 20
f. E. 40
You are carrying out analysis of a new
test for DVTs and decide to plot a 107. A 23-year-old woman attends the
receiver operating characteristic curve. antenatal clinic in her first pregnancy.
What are the axes of the Booking bloods reveal her Blood group
curve?(MARCH 2016) O and her rhesus status to be cde/cde;
her long-standing partner is also tested
 A. Sensitivity versus Specificity at her request and his blood group A
 Specificity versus Sensitivity with rhesus status CDe/CDe. What is
 Positive Predictive Value versus the expected blood group of her fetus?
Negative Predictive Value
 Sensitivity versus (1-Specificty)  A. A positive
 E. (1-Sensitivity) versus  B. A negative
Specificity  O positive
 O negative
105. A 23-year-old woman who is six
 AB positive
weeks by dates is referred to EPU with
vaginal spotting, which continuous 108. A 32-year-old female presents with
today. Ultrasound shows a fetal pole a two month history of breathlessness,

98 ‫صفحة‬ Dr. MOHAMED YAHIA 0900988745


MRCOG RECALLS

tachycardia and menstrual irregularity.


Her Investigations revealed: Serum
hCG negative Serum T4 178 mmol/l
Serum TSH0.05 mU/L Prolactin400
mmol/l What is the most likely
diagnosis?

o Primary hypothyroidism o Residual volume


o Graves' disease
 B. Expiratory reserve volume
 C. Hashimoto's thyroiditis o Inspiratory reserve
o Multi-nodular goitre volume
o DeQuervain's thyroiditis o Tidal volume
o Vital capacity
109. The probability of sarcomatous
change occurring in fibroid is 113. What electrolyte imbalance can
cause paralytic ileus?
o 0.1%
o 1% Hypernatremia
 C. 5%
 B. Hyperkalemia
 D. 10%
 C. Hypocalcemia
 E. 15%
 D. Hyponatermia
110. What part of the embryo is the
 E. Hypokalemia
origin of primordial germ cells?

o Gonadal ridge 114. Vasopressin act on increasing


water re-absorption at
 B. Yolk sac endothelium
 C. Coelomic epithelum o Proximal tubule
o Loop of henle
 D. Extraembryonic mesoderm
 C. Distal convoluted tubules
 E. Primitive sex cord  collecting duct

111. The greater omentum is derived 115. Placement of cup during ventose
from which of the following embryonic delivery when the neck is flexed
structures?
o Sagittal
o Dorsal mesoduodenum o Coronal
o Dorsal mesogastrium o Temporal
o Pericardioperitoneal canal o Frontal
o Pleuropericardial o Lamdoid
membranes
o Ventral mesentery 116. Schiller Duval Bodies is found in

112. The following is a normal o choriocarcinoma


spirometry of a preoperative woman. o endodermal sinus tumor
Which lung volume is indicated by the o dysgerminoma
arrows on the spirometry tracing? o teratoma
 Gonadoblastoma

117. No of carbons in Estrogen?

o 18
o 19

99 ‫صفحة‬ Dr. MOHAMED YAHIA 0900988745


MRCOG RECALLS

o 20
o 21
o 22

118. Pregnancy related nausea and


vomiting uses PUQE index. What is the o 1
Maximum score? o 2
o 3
o 5  4
 B.11  5
 C.15
 D.20 123. How much in the UK population
has RH negative blood group?

o 5%
o 10%
o 15%
o 20%
o 25 %

124. In Horseshoe Kidney what is the


structure preventing the kidney to
120. Most common endocrine disorder ascend from pelvis to abdomen?
in reproductive age group o superior mesenteric a.
o Diabetes Mellitus o inferior mesenteric a.
o Polycystic Ovarian o bifurcation of aorta
Syndrome o coeliac trunk
o Premature Ovarian Failure  median sacral artery
o Hypothyroidism
125. Muscles found in the scrotum
o Hyperprolactenimia
o Tunica Albuginea
121. Perineal Nerve Supply with
o Dartos Ms.
diagram
o Tunica Vaginalis

126. Angle of the vagina with the


horizontal while standing

o 45
o 60
o 70
o 80
o 90
What is the nerve root? 127. Joint which resist horizontal
rotation of the pelvis
o S2,S3,S4
o L3,L4 o Sacroiliac
o S3,S4 o Sacrospinous
o L1,L2 o Ileolumbar
o L4,L5 o Sacrotuberous
122. Structural diagram to which 128. Two Most common dermatitis in
carbon number base will attach pregnancy

100 ‫صفحة‬ Dr. MOHAMED YAHIA 0900988745


MRCOG RECALLS

o Atopic Dermatitis + o Common Iliac a


Polymorphic (PUPP) o Deep circumflex iliac a.
o Pustulae + Cholestasis o Femoral a.
o Pemphigoid + Choletatis
o Polymorphic (PUPP) + 135. At Term G2 wearing seat belt has
Pemphigoid car accident she has abdominal pain
o Polymorphic (PUPP) + and consults you for anti D
Obstetric Cholestasis
 she doesn’t need
129. Most common protozoan causes  anti D 250iu
zoonotic infection(cerebral calcification,  anti D 500iu
retinitis)  anti D 1500 iu
 perform kleihauer
o Chlamydia test
o Plasmodium
o Toxoplasma 136. Dalteparin a Low molecular wt
o Trichomonas heparin is measured using?
o Gardia lambia
o APTT
130. Paroxetine is teratogenic to which o PT-INR
organ o AntiFactor Xa activity

 A.CVS 137. Congenital conjunctivitis organism.


 CNS Purulent discharge 36 hours after birth
 Craniofacial o Chlamydia trachomatis
 Limb o Neisseria gonorrgea
 Kidneys
138. Piezoelectric used in
131. Which electrolyte is involved in
central pontine mylenosis due to fluid o MRI
overload? o PET
o U/S
o Potassium o XRAY
o Sodium o CT Scan
 C Calcium
o Magnesium 139. Transformation Zone
o Chloride
o Glandular to squamous
132. Vitamin E toxicity metaplasia
o Glandular to squamous
a. dementia, dysplasia
b. hemorrhage.. o Squamous to glandular
c. renal failure, dysplasia
d. cardiomyopathy, o Squamous to glandular
e. lung fibrosis metaplasia
133. Homologue of round ligament in 140. Alternate complement pathway
males initiation
o Vas deferens o Microorganism
o Gubernaculum o C3b
o Rete testis o Protein
o Cell death
134. Origin of obturator artery
141. Prostaglandin is
o Internal Iliac a.
o External Iliac a. o Autocrine

101 ‫صفحة‬ Dr. MOHAMED YAHIA 0900988745


MRCOG RECALLS

o Paracrine o Retrovirus
o Endocrine
o Eccrine 149. First trimester pregnancy with
o Exocrine simple ovarian cyst measuring 5x5cm

142. Infertility case, Male 190 cm wt 60 o Wait no follow up


kg, testicular volume 5 ml. What is the o Follow up at anomaly
possible karyotype scan for size
o First trimester surgery
o 46xy o CS + cystectomy
o 46xxy o Remove immediately
o 46xx
o 46xyy 150. Abnormal LFTs, HbsAg -ve,
HbeAg -ve, HbeAb +ve, HbsAb
143. Target Hgb with oral and IV iron 3miu/hr,HbcAb +ve

o 0.8g/dl/ml o Chronic infection ( high


o 1.2 g/dl/ml infectivity)
o 1.5g/dl/ml o Chronic infection ( low
infectivity)
144. 18 y/o non penetrative intercourse, o Post vaccination
+ painful ulcers in the vagina o Immune to Viral Infection
o Acute infection
o Primary HSV genitalia
o Behchet's disease 151. 25 y/o first papsmear , low grade
o Aphthous ulcer dyskaryosis, next step?
145. Incomplete early miscarriage, o Colposcopy
bleeding PV, refuse surgical o Repeat after 6 months
intervention. o HPV testing
o Misoprostol 152. Cauterization of large area with
o Carboprost appearance of smoke
o Mifepristone
o Ergometrine o Carbonization
o Fulguration
146. HSG is done on what day of cycle o Vaporization
o Coagulation
o 1-5 o Desiccation
o 6-12
o 13-21 154. Pregnancy rate with COCP if
o 22-28 perfectly compliant and non perfectly
o 28-32 compliant
147. In ECG, T wave represents o 0.05 2-4
o 0.1 5-6
o Atrial depolarization o 0.3 10-15
o Ventricular depolarization
o Ventricular repolarization 155. VTE with COCP
o Atrial repolarization
o 2 to 6/10,000
148. Herpes simplex is what type of  B.
virus  C.
o DsDNA 156. INH Metabolism is by
o SsDNA
o DsRNA o Acetylation
o SsRNA

102 ‫صفحة‬ Dr. MOHAMED YAHIA 0900988745


MRCOG RECALLS

o Oxidation
o Reduction
o Conjugation
o Sulfonization

157. Muscles of internal sphincter


derived from?

o Longitudinal Muscle
o Circular Muscle
o Lavatory Ani
o External anal sphincter

158. Cardiac output after delivery 163. Primary Mechanism of shunting


blood away from the pulmonary
o <20% circulation
o 20-40%
o 40-60% o Foramen Ovale
o 60-80% o Ductus Arteriosus
o > 80% o Ductus Venosos
159. Where does the inferior rectal vein 164. Rate of miscarriage in couples with
drain? balanced translocation?
o superior mesentric vein ANSWER: 2-5%
o superior rectal vein
o External pudendal vein 165. What is the most common cause of
o middle rectal vein secondary hyperparathyroidism?
o Internal pudendal vein
o Parathyroid adenoma
160. IUD+ Pregnancy. What is the risk o Multiple myeloma
of Ectopic Pregnancy o Sarcoidosis
o Chronic renal failure
o 1/10 o Multiple endocrine
o 1/50 neoplasia type 1
o 1/20
o 1/5 166. L-arginine - Arachidonic acid
o 1/100
167. Gonadal artery level?

171. Sensitivity of test ( PTL)

ANSWER: 4/6= 67%

172. Lifelong Immunity

o Adaptive
o Innate
o Active
o Natural

173. During the inflammatory phase of


wound healing what is the predominant
cell type

found in the wound during the first 48


hours?

103 ‫صفحة‬ Dr. MOHAMED YAHIA 0900988745


MRCOG RECALLS

o Macrophages 179. 50 yrs old female had hysterectomy


o PMNs 2 hrs ago. The nurse noticed that urine
o Fibroblast output was 35 ml/hr in the past 2 hours.
o Lymphocytes What is the normal rate of urine output
o B-Cells per hour?

174. Mechanism of amino acid transfer o 0.25ml/kg/hr


across the placenta:- o 0.5ml/kg/hr
o 1ml/kg/hr
o active transport o 1.5ml/kg/hr
o simple diffusion o 2ml/kg/hr
o facilitated diffusion
o secondary active transport 180. Precursor of testosterone in theca
 bulk transfer cell

175. Glucose transported across the o Androstenedione


placenta thru? o DHT
o Estriol
o Facilitated Diffusion o Estradiol
o Active Transport o Cholesterol
o Passive Diffusion
181. Tetanus toxoid vaccine
D. Endocytosis
o Toxoided protein
176. Gonads are derived from which o Live attenuated
embryonic layer? o Subunit
o Conjugated
o Endoderm
o Ectoderm
o Intermediate Mesoderm
o Splanchnic Mesoderm

177. Which of the following is true for a


patient whose pre-transfusion blood
type is group O?(Please select 1 option)

o + AB antigen -ve
antibodies
o + A antigen +ve B
antibodies
o + B antigen -ve A
antibodies
o - AB antigen +ve Anti A
and Anti B antibodies
o + AB antigen and +ve AB
antibodies

178. Which immunoglobulin is a key to 185. 80 y/o no subjective complaints


passive neonatal immunity? came in for pessary change then with
profuse vaginal discharge. Culture
A. IgD result moderate beta hemolytic
B. IgE streptococcus with coliform and
C. IgM anaerobic bacteria .
D. IgA
E. IgG o No treatment
o Clindamycin suppository

104 ‫صفحة‬ Dr. MOHAMED YAHIA 0900988745


MRCOG RECALLS

o Metronidazole + on Oxytocin. After 2 hours, Baseline


Clindamycin p.o. X 7 days FHR 120 bpm, Variability < 5bpm, No
acceleration, No deceleration. Next
186. Thromboprophylaxis on patient management?
with hyperemesis
o Category 1 CS
o Until discharge o Category 2 CS
o All throughout antenatal o FBS
period o Wait for 1/2 hours
o Up to first trimester only
o 3 days after 192. 24 y/o suffer from right pelvic pain
commencement with nausea but no vomiting for 1 year.
She stopped OCP 2 years ago. Ca 125=
187. The Probability of sarcomatous 60iu/ml? What is your diagnosis?
change occurring in fibroid is
ANSWER:
o 0.1%
o 1% 193. Diagonal conjugate is the only
o 5% diameter that can be measured
o 10% clinically. How is it measured ?
o 15%
o From upper border of
188. Vertical transmission of HIV symphysis pubis to sacral
infection mostly occurs promontory.
o From Lower border of
o In the first trimester Symphsis pubis to sacral
o In the second trimester promontory.
o In the third trimester o From the back of
o During labor and delivery symphsis pubis to tip of
o In the neonatal period coccyx.
through breastfeeding o From Sacroiliac joint to
opposite iliopectineal line.
189. Primary amenorrhea with absent o It is the distance between
uterus, vagina ends in a blind pouch, ischial tuboristies.
normal breasts and scant pubic hair is
seen in: 194. Identify the muscle in the diagram
o Klinefelter Syndrome ANSWER: Levator Ani Muscle
o Mayer Rokitanski Kusher
Hauser Syndrome 195. Question about probability
o Turner's Syndrome
o Androgen Insensitivity 196. During which phase DNA synthesis
Syndrome and chromosome duplication take place
o Noonan Syndrome
o S phase
190. Why ramifentanyl used in patient o G1
controlled analgesia? o G2
o G0
ANSWER: Short acting easily o Mitosis
metabolized by serum esterase
197. BRCA 1 associated ovarian cancer
191. Patient Hypertensive, Gravida 2 in
labor 4cm dilated, membranes ruptured 198. Endometrial tissue found within
spontaneously clear liquor, CTG the myometrium is
showed Baseline FHR 120-130 bpm,
good beat to beat variability, no o Endometriosis
acceleration, no deceleration. She was o Fibroid

105 ‫صفحة‬ Dr. MOHAMED YAHIA 0900988745


MRCOG RECALLS

o Myoma A. Northern blotting


o Adenomyosis
o Leiomyoma  Southern blotting
 Southwestern blotting
199. Question about trophoblast  Western blotting
invasion  Eastern blottin
200. Which of the following laboratory
techniques is used for RNA analysis?

EXAM 6

106 ‫صفحة‬ Dr. MOHAMED YAHIA 0900988745


MRCOG RECALLS

1. IgG passing through the placenta a. Seretonin,


b. histamine,
a. Active transport, c. muscarinic,
b. passive transport,
c. phagocytosis, 4, Nature of heparin
d. diffusion,
e. Pinocytosis a. Glycoproteins,
b. polypeptide,
2. Usual questions of atosiban, labetalol, c. polysaccharide,
hydralazine, d. protein

3. which of the following Receptors 5. Why oxytocin can't be used orally


Ondansetron act:
a. Will be restricted by gastric juice

107 ‫صفحة‬ Dr. MOHAMED YAHIA 0900988745


MRCOG RECALLS

b. B) will be metabolized by liver 13. What group of drug is heparin


enzymes
c. C) will cause very powerful a. Peptide
contractions b. Glycoprotein
d. D) will be delay Ed in action c. Polypeptide
e. Rapidly des by gastric juice
14. Patient 8 weeks GA with threatened
6. A new diagram for possible miscarriage..which ia an indication for
fertilization through which a complete AntiD Ig
mole can occur and asked to identify a. Surgical intervention
which can form complete mole.. empty b. Complete miscarriage
egg with one sperm or 2 sperms c. vaginal bleeding
7. Another new picture about
15. Picture of direct coombs test
adenomyosis
16. Most common cancer pregnancy
- It also had a leiomyoma
and post partum
- Adenomyosis a. Ca Breast
8. There was another MRI picture of b. Ca cervix
intramural fibroid c. Ca ovary

9. Picture of a Testis..Arrow on 17. Non ergot dopa agoinst for


something looking like Ductus hyperprolactinemia
deferenns and asking of the embryonic - Guinagolide
origin
18. Post lap ligarion as daycare surgery
a. Mullerian duct
pt was admitted for pain management.
b. Mesonephric duct
Given morphine. After 6 hrs pt cmpls of
c. Gartners duct
pain. A cystic swelling present upto
10. MRI Of submucous fibroid umbilicus. Tender with oozing from
port sites.... diagnosis
- Intramural
a. Urine retention
11. Case scenarios of ovarian cyst b. Hemorrhage
4mm.usual options c. Infection

a. Send her back 19. which of the following is a Pentamer


b. Reassure and discharge immunoglobulin?

12. Question about about hirshsprung a. IgM


disease..Aetiology b. IgG
c. IgE
a. Abnormality in circular muscles d. IgD
b. Abnormality in longitudnal e. IgA
muscle
c. Abnormality in parasympathetic 20. Level of common iliac arteries
ganglia
d. Abnormality in sympathetic a. L4
ganglia b. L5

108 ‫صفحة‬ Dr. MOHAMED YAHIA 0900988745


MRCOG RECALLS

c. S1 - Proximal CT
d. S2
e. S3 30. Insensible waterloss

21. Bifurcation of aorta occurs at - 850ml

a. L4 31. What type is zika virus


b. L5 - SsRna
c. S1
d. S2 32 Picture of testes...what is the
e. S3 structure is labelled?

22. Cut section of hysterectomy - Paramesonephric duct the embryonic


specimen of a Uniformly enlarged origin
uterus.. history of painful menses..
Diagnosis 33. The vitamin causing pellagra....

a. Adenomyosis a. niacin,
b. Polyp. b. cobalamin,
c. leiomyoma c. pyridoxine,
d. Adenocarcinoma d. thiamine

23. Most common type of malignancy in 34. What type of epithelium is in the
tube fallopian tubes:

- Adenocarcinoma a. simple columnar,


b. pseudostratified columnar,
24. Pic of fetal circulation to identify DV c. cuboidal
d. squamois stratified
Ductus venosus
35. Picture of placenta and cord... what
25. One question for rmi calculation. type is it? Velamentous, vasa previa,
Postmenopausal bilateral solid ovarian
differentiate etc
T CA 125 100....
- Velamentous or vasaprevia?
- 900
- Velamentous insertion of cord?
26. Root value of ilioinguinal and
iliohypogastric..Repeat question 36. Picture of twin gestation: what type
of chorionicity?
- L1
a. Diamniostic dichorionic,
27. Rfeered pain of ovary felt near b. Monoamniotic monochorionic
umbilicus... level
Twin gestation at eight weeks...
- T 1o
37. Picture of cut section of ovary...what
28. Most common intracellular cation is the structure with arrow?
- Potassium a. Corpus luteum,
29. Absorption of glucose occurs in b. endometrioma,
which part c. albicans

109 ‫صفحة‬ Dr. MOHAMED YAHIA 0900988745


MRCOG RECALLS

38. Woman ?60 years with urgency, c. Internal iliac artery


urge incontinence, no symptoms of
stress incontinence....etc...what is the Calculation of odds ratio
next step? Sensitivity/ 1-specificity is ....
a. Beta sympathometic agonist, a. negative likelihood ratio,
b. beta sympathometic antagonist,
b. positive likelihood ratio
c. antimuscarinic etc c. Positive predictive value
d. Urogram investigation
Derive the null hypothesis from
39. Photo of cervix – type of epithelium
relationship of soymilk and menopause
- Columonar epithelium - No relation between menopause and
40. Immunoglobin g transport soymilk
mechanism Iodine transport in thyroid cell symport
- Pinocytosis a. Na
Piriformis nerve supply b. Ca

- Subcostal Most important role of vit c... collagen..


immunity..
Adenomyosis?
- Answer?? Collagen?
Circumscribed fibroid
- Coenzyme for collagen
Wernicke caused by which vitamin-
Caumarin acts again which vit
- Thiamine
- Vit k
Gnrh is?
Umbilical artery is branch of....
- Decapeptide
- Anterior branch of internal iliac
The only content of lesser sciatic
foramen? Derp circumflec artery is branch of....

a. Tendon of obturator, - Ext iliac


b. piriformis
Structure differentiating ext iliac from
c. Internal pudental vessle
femoral artery
X ray radiation is equivalent to - Ingiinal ligament
- 2.5 dsys to 3 days Which structure differentiates
Gonadal artery is branch of superficial perineal pouch from deep

- Aorta - Perineal membrane

Umbilical artery branch of Bartholins occupy which space. Usual


option
a. deep circumflex iliac
b. Aorta Superficial perineal pouch

110 ‫صفحة‬ Dr. MOHAMED YAHIA 0900988745


MRCOG RECALLS

Vestibule is between 2 -X recessive?

- Minora -X linked dominant?

Ondansteron inhibits what Fetal skull showing parietal bone

- 5ht3 Fetal head is said to be engaged when


the BPD passes through which AP
Question on ecg heart block Diameter measuring?
3 degree heart block? a. 11 cm
Question on CtG picture no h/o read b. 12 cm…
paper on ctg Chromosome 2 diagrams ? Is it
-FBS inversion

- Inversion
-repeat after 30 min

-type 1 cs Hydrogen containing buffer in urine?

-Cont ctg reasses after 30 min Ammonia

- It was reassuring cTg bt bc of meconium Phosphate


v hv to continue. Chloride
HPLC with peaks at A, A2 and S - Phosphate ??
HbF normal Ammonia is synthesised by the action of
Interpretation?? which enzyme?

a. Alfa thalassemia - Glutaminase


b. Beta thalassemia Urea cycle takes place in ?
c. Sickle cell disease
d. Sickle cell trait Liver
e. Normal interpretation
Glycolysis.... in cytosol
Finesterde act on enzyme
Cytosol
- 5 alpha reductase
Microscopic pictuof diplococci
Primary amenorrhea, absent uterus,
FSH 6.0, ovaries normal, karyotype Gonorrhea
46XX Chlamydia
Diagnosis?? Angiotensin 1 to angiotensin 2
- MRKH… convertion at ?

Odd ratio answer - Lung

-4? Teratogenicity of lithium,


aminoglycozide, tetracycline
Pedigree chart

111 ‫صفحة‬ Dr. MOHAMED YAHIA 0900988745


MRCOG RECALLS

- Heart I think 3. The one opposite 29

-Vestibulocochlear nerve Carcinoma of the vulva

-Teeth - Scc

Clue cells of bacterial vaginosis are ?? Most common degeneration in fibroid


??
-Vaginal epithelial
- Hyaline
Treatment of listeria
Most common type of bacteria in
Amoxicillin endometritis
Cefalexin - strept A
Erythromycin CTG of a low risk PG , 39 week had
Azithromycin AROM with meconium stained liquor
>> CTG for 55 minutes
-A
showed decreased variability and
HPV types causing warts, another one variable decelerations
for types causing high grade
a) 30 monitor
intraepithelial dysplesea

High grade neoplasia after vaccination... B) CS I

-33 C) CS II

D) FBS
Velamintous insertion

Epithelium of Fallopian tube ?? E)?

30 min?
Epithelium of distal urethra

stratified squamous Supra umbilical versus suprapubic


incision ... which layer is not
ciliated columnar epithelium encountered in supra pubic incision?

simple columnar Posterior rectus sheath

Chromosome with centeromere towards -First polar body


one side
Maximum radiation exposure in
- Telocentric?
Hystero contrast salpingography
- Acrocentric?
Hysterosalpingogram
Carcinoma with endometriosis
Lap and dye test
-Clear cell
Cristae present in...
There was a question asking to calculate
mitochondria
mode ...

112 ‫صفحة‬ Dr. MOHAMED YAHIA 0900988745


MRCOG RECALLS

Liver and biliary system are Why elevated FSH and normal
differentiated by invagination of what testosterone?
structure?
Testicular failure
Dorsal mesogastrium
MRI Tesla
Ventral mesogastrium…
Site of vassopressin production
- Septum transversum
- Supra and paraventricula
Corpus luteum?
Diathermy burn
Corpus albicans?
High density small return plate
- luteum
Low density earthed
The girls on a hill trek...respiratory
alkalosis Low density small return plate

The acid base balance q: ph 7.2 Similarity growth hormone And HPL or
prolactin can't remember
PCO2 8.2
In a Dexa bone mineral density scan the
HCO3 25 results are reported which two ways?

- Respiratory alkalosis T score (compares the individual to


another individual of the same age and
whch chains joins IgM together to make gender) and Z score (compares the
pentamer: individual to a young adult where normal
J chain is greater than –2.5)

T score (compares the individual to


L chain
another individual of the same age and
Mu chain gender) and Z score (compares the
individual to a young adult where normal
Lambda chain is greater than -1)
Prev 3 sections, placenta now invading Z score (compares the individual to
upto serosa... ans- pl percreta another individual of the same age and
gender) and T score (compares the
Male infertile, small soft testis in
individual to a young adult where normal
scrotum, Fsh high, testosterone normal.
is greater than 0)
Anabolic steroids
Z score (compares the individual to
Congenital absence of Vas another individual of the same age and
gender) and T score (compares the
Testicular failure individual to a young adult where normal
is greater than –1)
Seminoma
2nd common cause of CAH
Vasectomy
- 11 beta hydroxylase
abserbce of vas defernce ? ?

113 ‫صفحة‬ Dr. MOHAMED YAHIA 0900988745


MRCOG RECALLS

Common type of valvular cancer - Treat uti and give intrapartum antibiotic
prophylaxis
Placenta invading myometrium on to
serosa - Treat UTI and test again prior to labour

Primary function of DNA polymerase Nerve from medial border of psoas


muscle?
-added nuclotide
Obturator nerve
-dna repair
Adduction of thigh?
Skeletal muscle embryology from where
except skull -Obturator nerve

Endoderm Bifurcation of aorta ? L1 L2

Exoderm L3 L4

Neural tube - L4

Neural cret Lateral border of ischiorectal fossa

- Paraxial mesoderm - Lateral obturator faschia

-It was skeletal development except - obturator internus


skull....paraxial mesoderm
Letrazol
Which HLA not i extra villous
trophoblast?? - Aromatase inhibitor

- A nd b Antiemetic with involuntary movements


?
CT scan post hysterectomy?? Renal
stone pelvic hematoma or ureteric obst Cyclizine

- Uretric obst Promethazine

Hershsprung disease Metclopromide

Absence of Neurons - Metoclopramide

Which sympathetic or parasympathetic Enzym in granulosa cells not in theca


cells
Pregnant women at 22 weeks with
group b streptococcus in urine more - Aromatase
than 1 lakh ? which nerve passes medially by poas
Give intrapartum gbs treatment major muscle?

Perform c section - Obturator medial

Treat uti - Genitofemoral anterior

Treat uti and repeat culture during term -The rest of lumbar plexus nerve lateral
and if no gbs no treatment

114 ‫صفحة‬ Dr. MOHAMED YAHIA 0900988745


MRCOG RECALLS

- from lumber plexus Thromboxane A2 produced in platelets

androgen secreted from which cell - Platelets


under LH?
anticonvulsant therapy,hb low,which
- Leydig cell aneamia?

Sertoli cells phagocytosis in testis - Folic acid deficiency

mild dykaryosis?which HPV Intense vasospasm before menstruation


reaponsilbe due to...

Histology of endometrium in follicular - Pgf2 alpha


phase with graph of hormones
Coronal suture between which Bones
-Proliferative
-Frontal and parietal
Biopsy if taken at the arrow point what
is the histology.. options proliferative Parietal bones shown in picture
secretory luteal Anteroposterior diameter of pelvic inlet
follicular - 11?
AMH as ovarian reserve marker A primi is informed that engagement
Nerve supply of bladder has occurred. which is the
anteroposterior diameter of the inlet
Greater splanchinic
through which the biparietal diameter
Lesser splanchinic has to cross

Pelvic splanchinic 1.11

Hypogastric 2.12

- Hypo gastric? 3.13

- its sympathetic-inf,hypogastric 4.10

Cycle length 21 to 35 days variability Thromboxane released from platelets


due to ? Variable follicular phase
positive feedback of esrogen High level b hcg what cyst ? Theca
leuutin cyst
Fail toPositive feedback
32 year old with 4 cm simple cyst ?
Free T4 Reassure her

-0.1 percent which nerve passes by medialy by poas


major mucle
Structure passing through lesser sciatic
foramen - Obturator nerve

-Pudendal nerve -,illioinguinal

-Obturator tendon MCH-1 extravilluos ?

115 ‫صفحة‬ Dr. MOHAMED YAHIA 0900988745


MRCOG RECALLS

Absent is HLA-A HLA-B Pregnant lady with rash and fetal ascitis
.. parvovirus
Radiation exposure.. background risk
equal to one CXR , repeat XRay in PE -Parvovirus

3mths, 4mths?? Standard deviation of mean in normally


distuributed population ?
- 3 days?
95 or 99.7 ?
2-3 days
one SD-68
Drug of choice to treat HMB with
history of asthma and wish to start Normal distribution curve ..Wat
family within one year ...? percentage of data is encompassed by 1
std deviation
-Tranexamic acid
1.34
-Mirena
2.68
wish to start family within one year
3.95
Mirnea it says at year of time
4.99
Psammoma bodies contain calcium
Placental transfer of molecules...high
LDH tumour marker in dysgerminoma lipid soluble
Bartholins gland is situated in superficial Structure formed in inner cell mass
perineal pouch which divides it symmetrically and
Lane tall mechanism of action ? gastrulation occurs in future ?

Listeria what is treatment ?? Primitive streak

- Amoxacillin Notocord …

- Primitive streak
Anal sphincter layers

-Internal deep external superficial - Amniotic cavity?


subcutaneous Haldane effect
Which hormone maintains corpus - Deoxygenated Hb carries more CO2
letueum in early pregnancy ?
Ureteric bud from mesonephric or
Hcg wolfian duct
Progesterone - mesonephric
-a Graph showing fetal weight between 10
Test to compare unpaired non th and 50 th centile ?
parametric data?..chi squared or Mann -Normal
Whitney ?
yes-10%above means normal
- Mann whitney

116 ‫صفحة‬ Dr. MOHAMED YAHIA 0900988745


MRCOG RECALLS

chickn pox immunization ICF which ion is reamin more-k+

- Not exposed to chicken pox -K

vdrl test which layer ablated for mennorrhegia-


functional layer of endometrium
- Biological false positive
- Basal
grb B (+) neonate delivery after konw
sawb 4days +,-info m neonatologist -but basal is not upto stop bleeding dear,its
on functional i mean upto serosa
- Give antibiotics ?
Perinatal mortality by WHO..22 wks plus
-no reffer to neonatologist 7 days early neonatal mortality
Twins diagram... -it was 7days or 4wks upto?i forgot,not
Biopsy on day 7 to 14 of menstrual sure
cycle...proliferative or follicular -It's 7 days
-Histologically proliferative HPV type for high grade cervical
- Arrow one was secretory lesions...16 and 18

Phase 3 clinical setting shows efficacy of 6 and 11 warts.


rct or new drugs only Questions was 90% genital warts formed
- Randomised trial with other gold by
standard drug 1.6 and 11…
POP only pill most common side effect Molluscum contagiosum..poxvirus
- erratic bleed Smear taken from a patient with vaginal
discharge..Gram stain shows following
Cell free Fetal DNA source
picture..Identify the
- placntal trophoblast
organism
DMPA mechanism of action
Umbilical cord section showing umbilical
- inhibition of ovulation vein

medalzzolam act on-gaba receptor caput medusa

labetalol Repeat question..Which embryological


remnant results in this
- Nonselective alpha and beta antagonisy
1.umbilical artery
-selective alpha ,non selective beta bloker
2.umbilcal vein
Hyperemesis which fluid to avoid 5%
dextrose Caput medusae formed by opening of
embryological remnants of umbilical vein
-Hyperemesis which fluid to avoid 5%
dextrose

117 ‫صفحة‬ Dr. MOHAMED YAHIA 0900988745


MRCOG RECALLS

Fetal circulation diagram showing ductus - I think lobular hyperplasia


venosus
Pain scale as varianc
What abt anti d is given in ?
-Analysis of variance ??
I wrote surgical abortion
no ordinary
i wrote threatened.
-Mann whitney???
its threatedn
There was one q on discrete/nominal ?
in threatened you should not give anti D
till it is established bleeding - Ordinal

Upto 9 wks H ducrey causes chanchroid

but it was writtn in oxfrd revision ?? Condyloma lata secondary syphillis

threatened means she bled bt os is closed. Placenta formation..lacunae develop in...

Ans what is the true answer inyermediate trplophoblast

And me surgical abottion but dont know -Syncitiotrophoblast .sure


true or false Nerve root for ilionguinal and
Less than 12weeks of gestation no need of iliohypogastric nerve s
anti d in painless single episode of - L1
bleeding
Nerve injury following hysterectomy with
But when surgical intervention plannd failure of adduction of the thigh
then 250 anti d given
- Obturator
Source pass mrcog
Nerve from the medical border of psoas
It uterine vein drains into int iliac vein
Sensory nerve of ovary
Left renal vein
- t10
RCOG trick
There was q 20% transport ? Albumin
The question was left uterine vein drains ?shbg ?
into..So ans is left int iliac vein
- Oestrogen transport
Maintainance of lactation through ductal
dilatation, lobar hyperplasia, stromal -Oestrogen in menopause.. estrone
hyperplasia, fat atrophy
-albumin binds to 20%,estrone-in
- Lobule one postmenopouse

- Ithink stromal because he said when she -also qet about -pre menoposal estrogen
was pregnancy not after delivery soucer -granulosa cell

She is lactating since last one year What is the term used for the anticancer
agent methotrexate?

118 ‫صفحة‬ Dr. MOHAMED YAHIA 0900988745


MRCOG RECALLS

plant alkaloid - Adenocarcinoma

Antimetabolite Benign tumor histology shows Spindle


shaped cells?
Alkylating agent
-Fibroma
Chain cutting agent
-History suggestive of meighs
Intercalating agent syndrome..Mild pleural effusion
-Antimetabolite or cytotoxic...Not sure Present..So ans fibroma..( repeat)
which one is correct Type of radiotherapy inside vagina
-antimetabolite - Brachytherapy
Commonest type of fibriod degeneration. - Repeat question this was..
Hyaline ,red
Image of twins on ultrasound
- hyaline
A pregnant patient is more than happy
Q64 ) mechanism of drug crossing the to see twin viable gestation..Wat is the
placental barrier. chorionicity
-high lipid 1.dcda
Q65) mechanism of action of methotraxate 2.mcda
- Antimetabolite 3.chorionicity can't be assessed
67) regarding magnesium metabolism 4.mcma
- Doesn't chng DCDA?
-Its was abt excretion ? Mcda
-Yes magnesium excretion... Unchanged Mcda, there is no delta/lambda sign
nature of vincristine characteristic for dcda

- Tubule Inner cell mass?

- Attatches - Amniotic cavity

Q 68 landfall is alpha and beta agonist q Skeletal system?


69 ) letrazole is aromatase inhibitor q70) - Paraaxial mesoderm
nature of heparin
Which is major cell for adaptive
paper 2 immunity
Vdrl+ bt others test negative management? - Lymphocyte
- It was repeat question..vdrl +ve and tpha Krukenberg tumour from GIT
neg..So false positive was the answer
Macrophage from monocyte
Most common type of ca in fallopian tube?

119 ‫صفحة‬ Dr. MOHAMED YAHIA 0900988745


MRCOG RECALLS

Which phase of Cell cycle immediately Labetolol is?


precedes Mitosis?
Ans: alpha & beta antagonist
- G1
Antimetabolite?
- G2
Ans: Methotrexate
Steroid secreted by fetal adrenal and
placenta which hastens lung maturity Finestrate?

- Ans: 5 alpha reductase

Cortisol Letrozole?

Post sterilization has received mprphine fr Ans: Aromatse inhibitor


pain relief inpatient fr 24hrs. Nw has pain Treatment for hyperprolactinaemia?
on exam cystic
Ans: Quinagolide (all options started with
mass upto umbilicus tender to touch, some ‘Q’...bastards)
oozing frm portsite
Lithium toxicity affects which system?
- Urinary retention
Options: CNS, CVS, GI...
-Potr site infection recall
Oxytocin can’t be given orally because?
-Another option was post bleeding I think
Ans: destroyed by gastric juice
Null hypothesis Q ?
Main defect of tetracycline
Type 2 error?
administration?
The hypothesis was falsely accepted.. Options: included teeth and limbs
Options Cranial nerve risk with aminoglycosides?
Type 1
CN 8
Type 2 Type of genital mutilation after
Formula of positive likelihood ratio clitorectomy

Which enzyme is in granulosa and - Repeat question.. clitoris excised..Labia


lacking in thecal cells normal..No cauterisation..So answer type
1
-Aromatase
A women aged --35--45yrs with right
Sertoli cells (cleaning)cells adnexal cyst with torsion like symptoms
diagnosis?
Nature psammona
A.cystadenocarcinoma
-Calcium
B.dermoid cyst
- Nature of clue cells
C.peritoneal pseudocyst
Épithélial cells

120 ‫صفحة‬ Dr. MOHAMED YAHIA 0900988745


MRCOG RECALLS

D.fimbrial cyst remember the whole details of the


question
E..........
- Lateral umbilical folds contain which
Plasma volume increases in pregnancy structure
by?
-Inferior epigastric vessels
A.60-90%
Q was medial ligament not lateral
B.30-50%
What is the mechanism during
C.10-20% breastfeeding which increases the flow
D.15_30% and keeps it going for exclusive

A case of difficult hysterectomy breastfeeding???? Something like that


followed by abd pain , MRI showed A.hyperplasia of ducts
unilateral distended kd and mass in
B.I don't remember rest options.......can
the pelvis , this one I was confused someone recall????
between ureteric ligation and harmatoma.
Please help me guys can't - Lobular hyperplasia

EXAM 7

121 ‫صفحة‬ Dr. MOHAMED YAHIA 0900988745


MRCOG RECALLS

7. Pic of trichomonas viginalis

1. Percentage of Brenner tumor of ovary


are malignant--- 5%.
2. Diagram of arcuate line
3. Diagram of rectus muscles below
umbilicus asking for during
suprapubic incision after cutting
rectus sheath u will encounter ………
answer was fascia transversalis.
4. Diagram of epidural space 8. Semen analysis report ….. it was
normal semen report.
9. Woman with 28days menstrual cycle
when to perform
hysterosalpingography………on day
6-12.
10. Decrease in amplitude of ultrasound
5. due to absorption, scattering,
6. Complication with retroverted reflection is called as…..attenuation
uterus……… Artifact

122 ‫صفحة‬ Dr. MOHAMED YAHIA 0900988745


MRCOG RECALLS
11. Appendiceal artery is a branch of ? B. Teratoma
a. Ileocolic artery
b. inferior mesenteric a. C. Nephroblastoma
c. left colic a.
d. middle colic a. D. Sarcococcygeal teratoma
e. right colic a.
18.A neoplasm of the female genital tract
12. What is the most common cause of the occurring in an 18-year-old girl whose mother
female urethral caruncle? A. Trauma wastreated with diethylstilbestrol during the
pregnancy is likely to be
B. Tumour
A. Brenner cell tumor of ovary
C. Infection
B. Teratoma of ovary
D. Hypoestrogenism
C. Sarcoma botryoides of vagina
E. Congenital
D. Clear cell carcinoma of vagina
13. 10. Which of the following is precursor of
Serotonin? E. Squamous cell carcinoma of vulva

A) Histidine.

B) Glycine.

C) Arginine.

D) Tyrosine.

E. Tryptophan

14. For overactive bladder a new drug called 19.


Mirabegron is used. Which is the
singlemechanism of action for this drug? 20.Which of the following Antineoplastic
drugs causing hemorrhagic cystitis?
A. Selective serotonin re-uptake inhibitor
A. Cisplatin
B. Muscarinic receptors agonist
B. Methotrexate
C. Selective Beta-1 agonist
C. Paclitaxel
D. Selective Beta-3 agonist
D. Cyclophosphamide
E. Selective Beta-3 antagonist
E. Vincristine
15. A 47 years old women with absent periods
for 8 months, has bilateral ovarian mass with 21. Mechanism of action of ulipristal acetate ?
solid areas and ascites on scan and CA 125 of
A. Partial progesterone receptor blockage
250. What is her RMI score?
B. Selective Estrogen receptor modulator
A.750
C. Prostaglandin analogue.
B.2250
D. Selective progesterone receptor modulator
16. woman having chancroid causative
organism E. Oxytocin Antagonist
Hemophilus ducyri. 22.Clomiphene citrate mechanism of action ?
17.Which is common malignant tumour in A. Androgenic steroid
newborn?
B. Estrogenic steroid
A. Retinoblastoma

123 ‫صفحة‬ Dr. MOHAMED YAHIA 0900988745


MRCOG RECALLS
C. Progestogenic steroid C. Rubella

D. Gonadotropin analogue D. Parvovirus B19

E. Selective estrogen receptor modulator E. Cytomegalovirus

23.Ductus venosus is completely obliterated 28. Which HPV causes Genital Warts? It was
after birth to form what adult structure? scenario on anogenital warts.

A. Ligamentum venosum A. HPV 6 and 11

B. Ligamentum arteriosum B. HPV 16 and 18

C. Ligamentum teres C. HPV 16 and 45

D. Medial umbilical ligament D. HPV 18 and 45


E. Median umbilical ligament
E. HPV 31 and 33
24. ligamentum arteriosus is completely
obliterated after birth to form what adult 29.A Women with 150cm height and 90 kgs
structure? weight, What is her category of BMI?

A. truncus arteriosus A. Normal

B.4th aortic arch B. Overweight

C. 6th aortic arch C. Obesity Class I

D. dorsal aorta D. Obesity Class II

25. A 60-year-old with erythematous erosive E. Obesity Class III


lichen planus on the vulva. What type of
canceris linked to lichen planus? 30.A woman is 8 weeks pregnant attends for
genetic counseling as she worried about CF,
A. Squamous cell carcinoma hersister has a child with cystic fibrosis. She is
a carrier and her partner is Caucasian with a
B. Basal cell carcinoma rateof 1/25 What is the risk of the child having
cystic fibrosis? A. 1%B. 12.5%C. 25%D.
C. Merkel cell carcinoma 75%E. 100%

D. Malignant melanoma 31. Scenario on Location of Bartholin glands


A. superficial perineal pouch
E. Adenocarcinoma
32.Anatomical location of external anal
26. Which of the following organisms sphincter A. deep perineal pouch
Colonize copper IUD?
33. mechanism of action of apixaban A. factor
A. Lactobacillus. Xa B. thrombin inhibitor
B. Actinomyces 34.
C. Mobil incus

D. Ureaplasma

E. Staphylococcus aureus

27. The most common in Utero infection


causing fetal anemia in the UK is

A. Syphilis

B. Toxoplasmosis 35.What is affected in Horner syndrome?

124 ‫صفحة‬ Dr. MOHAMED YAHIA 0900988745


MRCOG RECALLS
A. Loss of Motor supply 43.A Pregnant lady 11 week , presented by
UTI , what the best course of action ?with
B. Excessive Sympathetic stimulation urine cs report showing enterococcus sensitive
to nitrofurantoin and trimethoprim and
C. Excessive Parasympathetic stimulation resistant to cephalosporin. Repeated urine c/s
report is same.
D. absence of Parasympathetic supply
A. give no treatment.
E. absence of Sympathetic supply
B. repeat urine c/s
36.Post-delivery the registrar notes that baby's
HR 150bpm, pink body and blue extremities, C. cephalosporins
Grimace, some flexion & friable cry. What is
the apgar score? D. Nitrofurantoin.

a. 3 b. 4 c. 6 d. 8 e. 10 E. Trimethoprim

37.What is the most common cause of active 44.What do the bottom and top represent on
prolactin secreting tumor? the box-and-whisker plot?
A. Mean and mode
A. Adenoma B. Sarcoma C. carcinoma D. B. Mean and median
hyperplasia E. Iatrogenic Steroid C. Standard deviation and mean
Administration D. Least and greatest value
E. Lower and upper quartile
38.A town with a total number of live birth is
250,000, and maternal death is 750, number 45.Radiation could be transmitted or decay
ofstillbirth is 250, what is the Maternal
mortality ratio? SI unit of radiation is

a. 300 b. 600c. 900d. 200e. 750 A. becquerel

39.A Women after delivery found to be in B. gray


lithotomy position for 2 hours , now she
develop drop foot and loss of sensation. what C. rad
nerve most likely to be the cause ?
D. sievert
A. Tibial nerve. B. Femoral nerve. C. Sciatic
nerve. D. Common peroneal nerve E. Hertz

40.The Progesterone receptor is example of 46.A scenario on Botulinum toxin is now used
which type of receptor ? to treat neurogenic bladder because of its
ability to
A. Guanylate cyclase receptor B. Nuclear A. Inhibit the release of acetylcholine
transcription factor C. Tyrosine kinase nuclear B. Block storage of acetylcholine
receptor D. Ligand-gated ion channel in the C. Inhibit acetylcholine ACH synthesis
cell membrane E. G protein coupled receptor D. Block bladder nicotinic receptors
on the Golgi complex E. Block bladder muscarinic receptors

41.Which of the following drugs cause 47.What electrolyte imbalance can cause
Kernicterus ? paralytic ileus?
A. bicarbonate
A. Penicillin B. Gentamycin C. Sulphonamide B. hydrogen
D. Methotrexate. E. Erythromycin C. chloride
D. sodium
E. potassium

48.Schiller Duval Bodies is found in


A. choriocarcinoma
B. endodermal sinus tumor
C. dysgerminoma
D. teratoma
42. E. Gonadoblastoma

125 ‫صفحة‬ Dr. MOHAMED YAHIA 0900988745


MRCOG RECALLS
49. bHCG tumor marker for d. no-hemolysis

Choriocarcinoma 61. Most common cause of secondary


hyperparathyroidism
50.Rubella type of vaccination
A. chronic renal failure
live vaccination
B. Vitamin D deficiency
51. 20 years old lady menarche since age of 12
years, complaining of heavy menstrual C. Increased calcium
bleeding since menarche. CBC done. Physical
examination not remarkable. What to do next? D. liver cirrhosis

A. no any test to do 62. Innate and adaptive system are linked


together via
B. thyroid profile
A. Leukotrienes
C. serum ferritin
B. Cytokines
D.Clottingtest
C. TNF
E. FSH
63. TRH
52. Mycoplasma
genitalium….flask shaped and do not have a A. tripeptide
cell wall
B. Glycoprotein
53. Chlamydia trachomatous …. Gram
negative cocco-bacillus C. Polysaccharide

54. Ovarian surface epithelium….. cuboidal 64. delta cells of pancreas secrets

55. Fallopian tube most common A. Somatostatin


tumor….Papillary serous adenocarcinomas
B. glucagon
56. most common site of endometriosis…..
ovary C. Insulin

57. Arias Stella phenomenon effect on…. D. Pancreatic polypeptide


Endometrium
E. Cholecystokinin
58. Transverse colon blood supply….. Middle
65. Electrolyte disturbance caused by Conns
colic artery
syndrome
59. postmenopausal bleeding with granulosa
A. Metabolic alkalosis
cell tumor of ovary effect on endometrium
B. Respiratory alkalosis
A. Proliferative
66. Oestrogen type secreted by placenta
B. Secretory
A. estriol
C. Atrophic
67. HPL increased in which trimester
D. Inflammatory
A. 3rd trimester
60. Group A streptococcus on Lancefield
shows B. 2nd trimester
A. B-hemolytic C. Labor
B. a-hemolytic D. 1st trimester
C. $-hemolytic E. after delivery

126 ‫صفحة‬ Dr. MOHAMED YAHIA 0900988745


MRCOG RECALLS
68. alpha subunit of LH…. Similar to alpha 93.
subunit of FSH, TSH, Bhcg

69. prolactin with PCOs… due to increased


unopposed estrogen

70. iodine with sodium

71. OGTT: FBS- 5.8 2hr 7.9 gestational


diabetes

72. aldosterone… angiotensin II

73. carboprost … f2a


It was thalassemia major
74. delayed puberty… constituational
94.Chronsdisease , history of gut resection ,
75. adrenal medulla… acetylcholine
investigations show MCV increase , cause
76.duchenne muscular dystrophy… X linked vitamin b12 defeciency
recessive
95. misopristol diarrhea
77. pedigree of X linked dominant
96.lithium ebstein
78.mitrochondial DNA genes… 37genes
97.cystic fibrosis congenital absence of vas
79. DNA GCAT…. CGTA deifference

80. transcription requires …. RNA polymerase 98. VIN III by HPV 16

81. median 1 99. complete mole 46 XX

82. Calculate standard error of mean answer


0.05

83. allantosis endoderm

84.carbohydrate I breast milk …. Lactose

85. dimer igA

86. mucosal immunity igA

87.Internal anal sphincter… continuation of


circular rectal muscles.

88. Nevirapine…. Noncompetitively inhibit


reverse transcriptase
100. vitamin d unchanged
89. only hRT breast cancer…. Low risk or no
risk 101. DMPA 12 weeks

90. Female , early pregnancy , doesn’t know 102. ACE… kidneys


about her vaccination status for VZV , she may
103.cardiac output in first stage of labour
have immunity against it ?
a.<10 b.40 c 60-80 d.>80 A. 20% B. 30% C. 10%
91. perinatal mortality 100/1000
104. fetal cardiac output to fetal kidney
92. burner cells found in duodenum.
A. 10-15% B 20- 25% C.35%

105. antenatal anti d at 28 – 34 weeks

127 ‫صفحة‬ Dr. MOHAMED YAHIA 0900988745


MRCOG RECALLS
106. Remnant of patent umbilical artery .. Pudendal nerve
superior vesical artery
Ilioinguinal nerve
107.A scenarios of 15years girls wants
contraception 123.fetal kidneys mature to form urine at

Follow Fraser law 10 weeks

12weeks

124.test done on pregnant patients serum x is


positive in a patient.How likely she may
develop preeclampsia

80%

60%

125.placenta previa early confirmed at what


gestational age

24 weeks

108.This cyst is derivative of 28 weeks

Mesonephric duct 32 weeks

109. type 1 mutilation 126.calculate the sensitivity 80%

110.type of necrosis in brain .. liquefactive 127.odds ratio having apgar score less than
7…. 4
111.Ventose applied on… sagittal suture
128.which cell cycle precedes mitosis
112.postnatal depression methyldopa
G2
113.PUPQmaximum score 15
G1
114. pt with severe nausea and vomiting
thromboprophylaxis given till 129.CRISPR/Cas9 indicate what in molecular
biology
LMWH until discharge
A transcription factor and an enzyme
115.Hrt … increases thyroid binding globulin
A DNA and an enzyme
116.heat stable enzyme in PCR taq polymerase
130.
117.ND YAG laser…. Solid

118.MRI …. Hydrogen ion

119.grastulation formation of three germ layer

120.Which organism produce exotoxin that


causes toxic shock syndrome

Staphylococcus aureus

121.ratio of t4 to t3 in blood 7:1

122.After pelvic surgery of sacrospinous


fixation patient complains of perineal and Identify the muscle …obturator internus
valval pain that exaggerate on sitting which muscle
nerve is most likely to be damaged ……

128 ‫صفحة‬ Dr. MOHAMED YAHIA 0900988745


MRCOG RECALLS
131.monochorionic and diamniotic occurs at Mature teratoma
which stage…. Morula
138.one spontaneous mutation in human over
132.appendix testis derivative of years

Paramesonephric duct One in one million years

133.A scenario about patient developed caput One in one crore years
medusae that is connected to embryological
remnant of…….Umbilical veins One in 10million years

134.ovaries contain enzyme for androgen 139.during the cell cycle second meiotic
production division occurs at which stage

5alpha reductase Primary oocyte

11 b hydroxylase Secondary oocyte

3β-hydroxysteroid dehydrogenase 140.senerio of 47XXY

141.senerio of 45XO

142.oocyte originate from

Yolk sac

143.warfarin causes embryopathy at what


gestational age

6-9 weeks

144.Ebola virus is

135. Single standard RNA virus


Identify the artery arrow was on internal
pudendal artery 145.chiken pox virus is

136.in PKU there is deficiency of enzyme Herpes virus


known as hydroxylase. This enzyme catalyze
146.pregnant woman undergoes routine
the reaction and converting phenylalanine to
antennal test reports shows
Tyrosine
Treponema palladium IgG Positive EIA
Tryptophan
Biologically false positive
137.
Latent syphilis

Secondary syphilis

147.Herpes simplex virus 1

Double stranded DNA virus

148.Vitamin K source

Leafy vegetables

149. Pelvic diaphragm is formed by

A. Levator ani and coccygeus and its fascia


Identify the specimen B. Levator ani and its fascia

129 ‫صفحة‬ Dr. MOHAMED YAHIA 0900988745


MRCOG RECALLS
150.Tidal volume defines as Anti A, Anti B, Anti O

A. The amount of air inspired or expired at Anti A, Anti B


each respiration
161. 24 years woma had LMP 12days back.
151.Non pregnant with weakness general Presented 96 hours after unprotected sexual
Malaise intercourse. Which contraceptive advise u will
Labs high alkaline phosphatase offer
A. Vit d deficiency
Levonorgestrel, copper IUD
B. Gall bladder disease
Levonorgestrel, copper iud, ullipristril acetate
152.increase in caloric requirements after first
trimester Ullipristril acetate, copper iud

A.200 k calories B. 300k calories C.400 k 162.senerio of pregnancy of uncertain viability


calories
163. scenario of normal rise of hcg in singleton
153.diagram of urodynamic studies pregnancy

Detrusor overactivity 164.mechanism of action of POP implants…


inhibition of ovulation
154. A patient came to general practitioner
complaining of numbness and muscle spasm. 165.drug that cannot be given in third trimester
ECG done. There was ECG pic. That indicates to cause hemolytic anemia

Hypocalcemia Nitrofurantoin

Hypokalemia 166.sheehan syndrome

155. conn disease … metabolic alkalosis Anterior pituitary

156. cushing disease 167.mechanisum of action of methotrexate

Increase ACTH, increase Cortisol, Increase Dihydrofolate reductase inhibitor


GH
168.oxybutynin common side effect

Dry mouth

Blurring of vision

169.BMI 40kg/m2 loss 30kg for IVF

170.forest plot … confidence interval


157. 171.abdominal mass ca125
Identify the muscle involved in episiotomy. 172.bicarbonate PCT
Arrow was on bulbospongiosus
173.round ligament blood supply inferior
158.senerio of woman presented with epigastric artery
galactorrhea. Labs were advised which shows
high TSH 174.secondary cartilageous joint
Hypothyroidism 175.free fatty acid are transported via
159.muscle not innervated on perineal body Albumin
Ishiocavernosus 176.patient on estrogen only HRT since 5years
risk of breast cancer
160.patient blood group O positive. Antibodies
present would be Low risk or no risk

130 ‫صفحة‬ Dr. MOHAMED YAHIA 0900988745


MRCOG RECALLS
177.pic of complete mole 46XX 190.no of patients with BMI 19-25 --- 34%

178.hepatocytic hematopoiesis commence at 191.itching and rashes… pup


which week
192.B1
6 week
193.Question was 80 women died in which
8week bmi was checked as a risk factor so how many
died coz of obesity…80
179.Azoospermia … steroid overuse
194.senerio of vitamin b12
180.TVS frequency 7-9 hz deficiencybicytopenia

181.shoulder dystocia first maneuremcroberts 195.most common cause of hyperthyroidism in


pregnancy…..graves
182.menstrual cycle of 25days .. ovulate at 11
days 196. number needed to treat…… 4

183.5 weeks bhcg…2400, 4800 197. albumin synthesized in hepatocytes.

184.inhibin synthesis in granulosa cells

185.antiemetic causes extrapyramidal effects


… metoclopramide

186.vitamin c promotes hydroxylation in


protein synthesis

187.Northern blotting for rna

188.level of evidence 2a, 2b

189.Lichen planus symptomatic?

Erosive
Hyperpigmented

EXAM 8

131 ‫صفحة‬ Dr. MOHAMED YAHIA 0900988745


MRCOG RECALLS

1-woman with Bhcg 155 and after 48 protein S :decreased


hours Bhcg is 270, no C/O lower
abdominal pain, no gestational sac seen, Anticardiolipin antibodies: -ve
no adnexal mass, no fluid in pouch of A- normal pregnancy changes
douglas: B- congenital thrombophilia
a. Normal singleton pregnancy C- acquired thrombophilia
b. Molar pregnancy
3-Convex outline of uterus, lap and dye
c. C-Ectopic pregnancy
shows two uterine cavities:
d. D-Non-viable pregnancy
A- A-septate
2-A woman have got stillbirth at 34 B- bicornuate
weeks, her thrombophilia screening C- C-didelphus
shows:
4-Identify the type of placenta;
Hb:10.1

Platelates:135

Protein C:normal

132 ‫صفحة‬ Dr. MOHAMED YAHIA 0900988745


MRCOG RECALLS

A- succenturiate
B- circumvallate
b. A-Polyp
C- velamentous
c. B-Adenomyosis
5-patient have normal menstrual cycles d. C-Leiomyoma
but now she have amenorrhea for 8
8-Pt have mild lower abdominal
months,she is taking metoclopramide
pain,have H/O irregular menstrual
and valproate for epilepsy, she have no
cycles,not taking ocps,these ultrasound
visual symptoms,no hirsutism, her
findings indicate:
serum prolactin is 1000,your further
management?

A- A-stop metoclopramide and repeat


test
B- B-stop metoclopramide and
review from neurosurgeon
C- C -MRI brain
D- stop valproate and review from
neurophysician

6-patient have IUCD in place since 8


months and now she have C/O vaginal
discharge ,your management a. A-Corneal ectopic pregnancy
b. B-Endometrioma

9-Minor PPh is called:

a. A-500 blood loss


b. B-500-1000 with no signs of
shock
c. C - >1000 with no signs of shock
d. D-1500 with no signs 0f shock

10-A 25 years old with chronic left


lower abdominal pain for 3 months,she
a. Don’t remove iucd and take have previously received treatment for
vaginal swabs and treat chlamydia,her ultrasound revealed a
accordingly 4cm diffuse homogenous hypoechoic
focal lesion with little fluid in pouch of
7-Patient with heavy menstrual
bleeding: identify the picture douglas:

133 ‫صفحة‬ Dr. MOHAMED YAHIA 0900988745


MRCOG RECALLS

a. A-mucocele appendix
b. B-hydrosalpinx
c. C-Endometrioma

11-Direc most common cause of


maternal death according to 2017
guideline:

A-PPH B-Thromoembolism

12-Patient having severe pre- eclampsia


symptoms, BP 160/110,blurring of
vision, severe headache, pulse 85,she is a. A-Reasure
known case of asthma, drug you will b. B-Ultrasound 2 weekly
give to decrease her BP: D- FKC charting at home

A- I/v hydralazine 15-Patient pregnant , PID due to


B- I/v labetalol gonorrhea, appropriate treatment:
C- methyldopa a. A-Ceftriazone,doxycycline
13-placentation at site A is called: b. B-Ceftriazone, azithromycin

16- MRI picture of intramural fibroid


(Same picture as came in July 2018)

17-Patient postpartum and is breast


feeding, have CT scan due to some
reason ,regarding breast feeding you
will advise

a. A-Encourage breast feeding


normally
b. B-Encourage breast feeding but
send baby’s UCE after 24 hours
a. A-accreta
b. B-Normal placentation 18-A woman have multiple fibroids ,is
c. C-Abruption D-Praevia planned for hysterectomy, she is on
ulipristal acetate prior to have surgery
14-patient is low risk and she came for which additional contraception you will
antenatal visit,her symphysiofundal advise:
height shown by this graph, what you
will advise after looking this graph ? Barrier contraception

19- identify picture of complete mole:

134 ‫صفحة‬ Dr. MOHAMED YAHIA 0900988745


MRCOG RECALLS

b. B-24 hours

27-Identify the muscle;

20-Combined oral contraception can be a. A ischiocavernous


used in woman had GTD: After Bhcg b. B bulbospongiosus
titer return to normal, not rising at all. c. C sphincter urethrae
d. D-Deep transverse perineal
21- Senario of ovarian cyst of 4 cm;
muscle
Reassure no follow up
28-Which structure devide the
22-Drug not given in 3 rd trimester to
paravesical and para rectal pouches
treat UTI: Nitrofurantoin

23-patient came with pv bleeding at


home,now pv bleeding is controlled and
all signs clued about complete
miscarriage,what will be your further
management:

A- PT after 2 weeks
B- D and E a. A-ureter
C- Ultrasound after 2 weeks b. B-uterine artery
c. C-external iliac artery
24- 30 years old woman having HMB
d. D internal iliac artery
that is disturbing her daily life
routine,she is not going to plain family 29-Which structure does not contain
soon,your choice of treatment: paracrine glands:
Mirena a. A-Mons pubis
b. B-Labia majora
25-Woman is 22 weeks pregnant,asked
c. C-Labia minora
about alchohol intake during
pregnancy: 30-Difference between jejunum and
ileum:
a. A-Should not be taken at all
b. B-she can take 14 units/ day JEJUNUM ILEUM
c. D-Can take 18 units/ day
a. A Thin walled, longer, long
26-when does the prophylactic dose of arcades Thicker, shorter, short
LMWH is stopped? arcades
a. A-12 hours

135 ‫صفحة‬ Dr. MOHAMED YAHIA 0900988745


MRCOG RECALLS

b. B Thin walled, longer, short 35-Which of the following structure


arcades Thick walled, shorter, passes immediately anterior to that
long arcades marked in x ray: ( Arrow was marked
c. C Thick walled, shorter, long over sacro iliac joint)
arcades Thin walled, longer, short
arcades

31-Which line indicates the junction


between hindgut and lower anal canal:
It was asked about dentate line.

a. A-Ureter
b. B-Uterine artery

36-Sacrotuberous and sacrospinous


32-sacral plexus situated behind which
ligaments prevent from:
muscle :
a. A-Anterior rotation at horizontal
a. A-Obturator internus
line of sacrum
b. B-Pyriformis
b. B-Posterior rotation at horizontal
33- During laparoscpy surgeons have line of sacrum
limitization of lymphectomy on lateral c. C-downward displacement of
abdominal wall due this structure: ilium and ischium

37- Which artery of internal iliac is


mostely absent:

Obturator

38-Uterine artery supplies what


percentage of blood supply to ovary:

a. A-10 %
b. B-20 %
a. A-Inferior Myenteric c. C-30%
b. B-Obliterated umblical artery d. D-40 %
c. C-Inferior epigastric artery
39- hepatic artery is a branch of:
34-The expected angle between brim of
pelvis and horizontal line that is also the Celiac trunck
angle of vagina: 60 degrees

136 ‫صفحة‬ Dr. MOHAMED YAHIA 0900988745


MRCOG RECALLS

40-Rectus sheath is formed by: internal c. C-placenta


oblique, external oblique, transversus
abdominus apponeurosis. 50- NO derived from : L-Arginine

41- Sacrospinous fixation, which artery 51-Which vitamin given to prevent from
is close to it and is prone to be ligated? wernickes encephalopathy: B1

Internal pudendal Artery 52:Essential Fatty acid OMEGA 3:


Linolenic acid
42-Difference between inguinal and
femoral hernia: 53-Bending used for Cytogenic analysis:

INGUINAL FEMORAL a. A-G Banding B-M banding


b. C-D banding D-A banding
A- Superior and medial to
pubic tubercle inferior and 54-DNA replicates: S phase
lateral to pubic tubercle
55-CRISP/Cas : DNA+Enzyme
B- Inferior and lateral to
pubic tubercle Superior 56-Biological active form of vit d: 1,25
and medial to pubic Dihydrocholechalciferol
tubercle
C- Superior and lateral to 57-Bile salts produced by liver: Cholic
pubic tubercle Inferior and and deoxycholic
Medial to pubic tubercle
58-Which of the following metabolic
43- Left uterine vein drain into : processes produce the most energy
needed for uterine contraction:
Internal iliac vein
Oxidative phosphorylation
44- Rate limiting step in urea synthesis:
59 -Diagnostic test for megaloblatic
N acetylglutamate dehydrogenase anaemia : serum B12

45-Important function of glucagon : 60-At 4 weeks primordial germ cells are


within what structure:
Gluconeogenesis (glycogenolysis not in
options) Yolk sac

46- RNA analysis: Northern blotting Genital ridge

47- Prostaglandins are synthesized 61-Major contribution to amniotic fluid


from: by fetal urine:

a. A-Saturated fatty acids A -16 weeks


b. B-Unsaturated fatty acids
B -18 weeks
48-Which organel can attach to tRNA
and can synthesise protein:Ribosomes 62-Monozygotic dichorionic twins is
formed at:
49 -Source of cholesterol in term fetus:
A-morula
a. A-fetal kidney
b. B-Fetal liver B-Blastocyst

137 ‫صفحة‬ Dr. MOHAMED YAHIA 0900988745


MRCOG RECALLS

C-zygote( 2 cell stage) B-60%

D-Bilaminar disc C-80%

63-In lateral wall of vagina remnant of 73-Histogram shows which variable:


mesonephric ducts: Garteners duct
A-Time variable
64-Anterior pituitary start to develop
from: B- Continuous variable

A-6-7 weeks C-Ordinal

B-8-9 weeks C-11-12 weeks 74-Type 1 error: Rejecting null


hypothesis when there is difference
65-Fetus start to produce urine: A- 10
weeks B-12 weeks 75-SEM definition

66-Ductus arteriosus is between which 76-Statistical heterogeneity indicate:


structures: Aorta and pulmonary artery Difference between studies outcome

67-Lower 2/3 of vagina develop from: 77-Gestational age is:


urogenital sinus A-Interval
68-Posterior fontanelle closes by: 3 B-Ratio
months
78-Odds ratio calculation answer was :3
69-New drug 0.8, placebo drug 0.65,
calculate absolute risk reduction: 0.15 79--A couple visited fertility clinic,
woman have normal investigations and
70-Laparoscopy sensitivity 90 examination, the picture showing
%,specificity 90 %,what will be false kelinfelters
positive rate?
features, asked about what will be
A-90 %
results in man:
B 10%
TESTOSTERONE FSH LH SHBG
C5% A- Dec inc inc inc
71-when does the Odds ratio B- Inc dec dec inc
approximate risk ratio:
C- Dec inc inc dec
When the disease being studied is rare
80-Throid profile in pregnancy is
72-A tumor marker is revolutionized for checked by:
detection of endometrial cancer,60
women included in study, however A triiodothyronine
confirmatory test shows only 40 have ca
endometrium,what is the sensitivity of B TSH
test? (false negative not given in
C Thyroid binding globulin
scenario)

A-20%

138 ‫صفحة‬ Dr. MOHAMED YAHIA 0900988745


MRCOG RECALLS

81-which hormone decrease osteoclast 91-Hormone raised in post menopausal


activity and inc cartilage deposition at woman : FSH
the end of long bones in term fetus:
92-In pregnancy normal physiological
A -calcitonin changes;

B -Growth hormone A-Inc plasma volume,dec haematocrit,inc


blood volume
82-In girls puberty indicated by:
B-Inc plasma volume,inc haematocrit,inc
Growth spurt, Thelarchae, adrenarchae, blood volume
menarchae
93-How many days it takes resting
83-Progesterone is increased maximum follicle stage to ovulation: 300 days
up to:
94-Which type of cell junctions help in
A-At term transmission of small molecules from
B-6 weeks cell to cell in placenta:

C-12 weeks A-Gap junctions

B-Tight junctions
84-Half life of LH:

A-30 minutes C-Hemidesmosomes

B-6 hours D-Desmosomes

C-12 hours 95-Free fatty acids transferred from


maternal to fetal circulation: passive
85-GnRH other than brain is diffusion
synthesized in which abdominal
96-Correct sequence of sperm
structure?
peneterates the oocyte:
a. A-Kidney
b. B-liver Corrona radiata, zona pallucida,
c. C-pancreas perivitelline space, plasma membrane
d. D-Duedenum 97-What is the primary haemoglobin at
e. E-Gall bladder 32 weeks of gestation: HbF
86-Which hormone leads to nutrient 98-What is the component of placental
transfer from mother to fetus: HPL barrier at term:
87-primary aldosteronism features: synchtrophoblast+endothelium
Hypernatraemia,hypertension,hypokala 99-Amino Acids tranfer through
emia placenta by: Active transport
88-Calcitonin produced by:thyroid 100-left side deviation of Oxygen
89-Percentage of T3 free: 1% dissociation curve due to ; dec 2,3 DPG

90-FNN act through : Integrin 101-volume dec in pregnancy: ERV

102-identify the picture:

139 ‫صفحة‬ Dr. MOHAMED YAHIA 0900988745


MRCOG RECALLS

d. D-45x0

110-Edwards male karyotype: 47XY

111-Oligohydroamnios,low set ears,limb


hypoplasia,renal,agenesis;

A-Potter syndrome

A-Endocrine B-pataue syndrome

B-Paracrine 112 -X linked recessive pedigree

C-Autocrine

103- O blood group: Anti A and B


positive

104-Miscarriage occur due to withdraw


of this hormone: Progesterone

105-PTH function : dec calcium


excretion
113-Autosomal recessive pedigree
106-Inc milk production during
114-woman with flu like symptoms and
pregnancy: HPL+Prolactin.
bachachae, delivered baby after 1 week,
107-Sex differentiation occur due to: baby got jaundiced, have

Short arm of Y chromosome hepatosplenomegaly, rashes over body


cause?
108 -BRCA 1 associated with:
A-listeriosis
A-Ca breast 75% and ca ovary 35%
B-GBS
109 -POF may have karyotype or
mutant gene: 115-Bacterial vaginosis diagnosis
criteria according to amsel criteria:

A-Clue cells, fishy smell,Ph > 4.5

116-woman having parvo virus infection


,what is her risk 0f fetal loss if she got
infection in 1 st trimester:

a. A-5%
b. B-10%
c. C-20%
d. D-50 %

117-Patient have history of ca


a. A-46xx colon,ureter is fixed in colon,sigmoid
b. B-46xxi audit is performed,shunts are left
c. C-47xxy inside,don’t

140 ‫صفحة‬ Dr. MOHAMED YAHIA 0900988745


MRCOG RECALLS

remember complete thread,was asked 127--Benign ovarian tumor: dermoid


about gram negative bacilli in ureter: (teratoma)

a. A-psuedomonas aurginosa 128-Red degeneration of fibroid occur


b. B-s.aureus due to: Ischemia
c. C-E.coli
d. D-Faecalis 129-Most common fibroid degeneration
:hyaline
118-woman with hepatis C test,her PCR
is negative and HCV antibody 130-Ovarian ca associated with
positive:indicate mutation of mismatch repair genes:

Previous infection a. A-Clear cell carcinoma


b. B-serous ca
119-neonates with CMV symptomatic at c. C-Mucinous
birth: 10-15 % d. D-papillary

120- Woman with VDRL positive : 131-Pt Operated for ca cervix and
(repeated )Likely biological false TAH+BSO is performed, Ovarian
positive histopath shows which type of
carcinoma:
121-MMR: Live attenuated vaccine

122-Lancified grouping of streptococci


is based on:

a. A-Antigen present in their cell


wall
b. B-colour on AGAR
c. C-Sugar fragmentation

123-Malaria common: p.Falciparum

124-Which vulval disease is associated


mostely with autoimmune diseases:
A-Metastatic carcinoma B-Leimyoma C-
a. A-Lichen sclerosis Lymphoma
b. B-Eczema
c. C-lichen planus 132-24 years old sexual contact with her
new boyfriend,she is concerned about
125-Which of the following is non development of new lesions on vulva.On
epithelial carcinoma of vagina:
examination you note pearly white ,non
a. A-Rhabdomysarcoma tender,dome shaped ulcers: Molloscum
b. B-Leimyosarcoma Contagiosum
c. C-Lymphoma
133-BRCA is: tumor suppressor
126-Postmenopausal women having
typical complex hyperplasia: progress 134-Brenner tumor malignant :5%
to ca endometrium < 5 % 135-unmarried, history of non
peneterative intercourse, C/O painfull
ulcers over vulva :

141 ‫صفحة‬ Dr. MOHAMED YAHIA 0900988745


MRCOG RECALLS

A-Behchets disease 145-Radiation exposure with ventilation


perfusion scan:
B-herpes
A-3 days
136-Lynch syndrome occur due to:
mutation of genes involved in DNA b-3 months
mismatch repair.
C-6 months
137-Symptomatic lichen planus with
dysuria and some vulval lesions ( A very D-12 months
big scenario): 146-ultrasound resolution maximum at;
138-which of the following cell types A focal region
lyses cells that have been infected with
virus: B interface

A-CD4+T cells B-CD8+T cells C gray

139-The tissue in which most D axial


lymphocytes are produced: Bone
Marrow 147-Type of TVS probe used in obese
woman;
140-What percentage of lymphocytes is
present in blood: 20-40% a. A low frequency curvilinear probe
b. B high frequency curvilinear
141-Patient have recurrent miscarriage c. C low frequency linear probe
due to inadequate placentation,what d. D high frequency linear probe
Type of HLA is deficient in each
pregnancy: HLA-G 148-radiation dosing in:

142-What in body secretions such as msv (mili sievert)


cervical secretions attack bacteria
149-In electro surgery long loop is used
lipoprotein:
for:
A-Lysozymes
a. A-To earth the patient
143-cell lysis by: b. B To complete circuit High
density current
c. C-To complete circuit Low
density current

150-CTG shows baseline heart rate of


155, no acceleration, baseline variability
8, Variable D cells for 2 hours, pretend
CTG:
A-C3
a. A:Suspicious
B-C5 b. B:Pathological
c. C: Non-reassuring
144-Complemt system synthesized in:
Liver 151-CTG paper frequency

142 ‫صفحة‬ Dr. MOHAMED YAHIA 0900988745


MRCOG RECALLS

A- 1cm B-2cm C-3cm c. C-Fetal pole of 4mm with no heart


beat shows viable pregnancy
152- A small square of ECG: d. D-gastational sac of 20 mm is
a-0.02 sec appropriate for gestational age

B- 0.04 sec 157- In a study comparison of singleton


and twin pregnancy is made:
153-Inferior myocardial infarction
,leads show it:

A-II aVF

B-III aVF C-aVF

154-Normal ECG changes in pregnancy


:Q waves in lead III

155- Partograph, what this arrowed line


indicate? (Left arrow) A-If cervical length less than 20mm then
double chances of preterm before 34
weeks in twins than singletons

B-If cervical lenth Less than 30 mm then


double chances of preterm before 34
weeks in twins than singletons

158-Identify the Diagram,,a scenario


was given and was asked about the
cause:

a. A action line
b. B alert line
c. C active stage
d. D second stage

156- 6weeks gestasional amenorrhea


A-Normal voiding urogram
,gestational sac of 20 mm, fetal pole of
4mm,no fetal heart beat seen ,what does B-Bladder overactivity
it
159-Haemolytic disease in newborn is
indicate? detected by: direct coombs test
a. A-fetal pole 4mm with no fetal 160-FBS 7.23 your action : repeat after
heart beat is appropriate for 30 minutes
gestational age
b. B-Fetal pole of 4mm with no fetal 161-In Emergency department with
heart beat shows non viable tachypenia,ABGs show following
pregnancy results:

143 ‫صفحة‬ Dr. MOHAMED YAHIA 0900988745


MRCOG RECALLS

PH:7.25 pC02 normal range,p02 normal c. C-5%


range,HCO3: dec
168-Mechanism of action of
A-Mixed respiratory/metabolic acidosis clindamycin :Inhibit protein synthesis
by binding to 50s subunit
B-Metabolic acidosis
169-Mechanism of action of
162-Pt with headache on day of metronidazole: covalentely bind DNA
hysterectomy,ABGs shows following
results: 170-Mechanism of action of fluconazole:
Inhibit cell wall synthesis by binding to
Ph:7.5 Pc02 low HCO3 normal 14 a demethylase
Interpret: Hyperventilation 171-Mechanism of action of
163- Normal level of fibrinogen in neostigmine: Anti cholinesterase
blood:( 1-2 gm That is what I did ,don’t 172-Trimethoprim is: antimetabolite
remember other options) (affects DNA synthsis)
164-woman have history of eclampsia in 173-Diazepam can cross placenta
previous pregnancy,now she is 12 weeks because of : Lipid Solublity
pregnant,what is her risk of pre
174-Mechanism of action of vincristine:
eclampsia in current pregnancy: Inhibit fuction of microtubules
a. A-10 % 175-Mechanism of action of
b. B-15-20 % Indomethacin: cyclo oxygenase
c. c-30 % inhibitor
d. D-55%
176-Midazolam acts over:GABA
165-woman have history of receptors
salpingectomy due to ectopic
pregnancy,recurrent risk in next 177-Ulipristal type of drug: Selective
pregnancy of ectopic is : progesterone receptor modulator

a. A-1-3 % 178-Depoprovera :Inhibition of


b. B-5-10% ovulation
c. C-15-20%
179-Oxytocin cant be administered
166-patient having GDM,what is her life through oral route because:Is destroyed
time risk of diabetes type II: by gastric juice enzymes

a. A-5-10% 180-Calcium channel blocker:


b. B-15-20% Nifidepine
c. C-25-35%
d. D-45-55% 181-Drosperinone is:

167-Risk of miscarriage with a. A-Mineralocorticoid


amniocentesis: b. B-Progestogenic
c. C-Androgenic
a. A-1%
b. B-2% 182-Second generation progestogen is:
Levonorgestril

144 ‫صفحة‬ Dr. MOHAMED YAHIA 0900988745


MRCOG RECALLS

183-Non ergot dopamine agonist used


after stillbirth :quinagolide

184-Drug excretion dec in pregnancy


due to:

A-Dec plasma volume

B-Dec albumin and binding proteins

185-Indirect action of warfarin :


prothrombin II

BEST OF LUCK
‫ محمد يحيى أبوالقاسم ختم‬.‫د‬

145 ‫صفحة‬ Dr. MOHAMED YAHIA 0900988745

You might also like